Mint StatiCA
Sign Up on PracticeMock for Free Tests, General Awareness, Current Affairs, Exam Notifications and Updates

This segment aims to finetune your preparation by asking static questions based on the topics covered in Mint newspaper. It has been observed that even static questions in Phase II (Economic and Social Issues, and Finance) of RBI Grade B and both phases of NABARD Grade A are hugely influenced by the topics in news. On that note, please attempt the quizzes and evaluate where you stand.

RBI Grade B Free Mock Test 2023

Below we will be providing a set of 5 question answers on a daily basis for the aspirants to keep up their exam preparation.

Mint StatiCA April 2023

Mint StatiCA MCQs – March 31, 2023

Last week, the UN’s Intergovernmental Panel on Climate Change (IPCC) released its report that detailed the devastating consequences of greenhouse gas emissions and warned that the world needs to change course to avoid irreversible damage. The IPCC is the UN body for assessing the science related to climate change. It was set up in (A) by the (B) and United Nations Environment Programme to provide policymakers with regular assessments of the scientific basis of climate change, its impacts and future risks, and options for adaptation and mitigation.
Identify (A) and (B) respectively.
(1) 1980; UNDP
(2) 1988; UNDP
(3) 1980; WMO
(4) 1988; WMO
(5) 1990; UNDP

Ans: 4
Solution:
The IPCC is the UN body for assessing the science related to climate change. It was set up in 1988 by the World Meteorological Organization and United Nations Environment Programme to provide policymakers with regular assessments of the scientific basis of climate change, its impacts and future risks, and options for adaptation and mitigation.

2.) A ₹33,000-crore backstop proposed by the market regulator to support debt mutual funds at times of liquidity freeze is only an initial corpus, and it could be topped up in future, said a key executive at the fund that will manage this facility.
Identify the market regulator being talked about in the above excerpt.
(1) RBI
(2) SEBI
(3) IRDAI
(4) NABARD
(5) PFRDA

Ans: 2
Solution:
SEBI in India is the policymaker that regulates the mutual fund industry. The SEBI guidelines for mutual funds protect the investor’s interest. These guidelines help investors to make more established and informed investment decisions. SEBI regulates and promotes the securities of the Indian market.

3.) The __(A)__’s Directorate General of Trade Remedies (DGTR) recommended a duty of $0.512/kg on these imports in December as part of a government initiative to enhance the quality of textiles.
Identify the ministry (A).
(1) Ministry of External Affairs
(2) Ministry of Heavy Industries
(3) Ministry of Commerce and Industry
(4) Ministry of Finance
(5) None of these

Ans: 3
Solution:
The commerce ministry’s Directorate General of Trade Remedies (DGTR) recommended a duty of $0.512/kg on these imports in December as part of a government initiative to enhance the quality of textiles.

4.) India participated in the ‘Bretton Woods Conference’ in 1944 as a British Colony and thus, has been the founding member of the Bretton Wood Twins.
Identify the Bretton Wood Twins from the options given below:
(1) IBRD; ILO
(2) IMF; ILO
(3) IBRD; IMF
(4) GATT; IMF
(5) None of these

Ans: 3
Solution:
India at the Bretton Woods Conference
India participated in the Conference in 1944 as a British Colony and thus, has been the founding member of the Bretton Wood Twins.
The Indian delegation was represented by the then Reserve Bank Governor Sir C.D. Deshmukh along with prominent Indians like Shanmukham Chetty, A.D. Shroff and B.K. Madan.

5.) The ____shows net receipts on account of trade in services, (or what are also called invisibles).
(1) Balance of Payments
(2) Balance of Exports
(3) Balance of Imports
(4) Balance of Trade
(5) Balance of Trade in Services

Ans: 5
Solution:
Balance of Trade in Services (BOS): The BOS shows net receipts on account of trade in services, (or what are also called invisibles). We can broadly classify invisibles into five groups, viz., (i) services, such as banking, insurance, shipping civil aviation, royalty, consultancy services, postal services, etc. (ii) investment income, which includes profits and dividends on direct, portfolio and other investments as well as interest charges on bilateral and multilateral loans. (ii) travel both business and tourist, (iv) government transfers, and (v) private transfers.

Mint StatiCA MCQs – March 29, 2023

1.)India’s state-run retirement fund manager raised the interest rate on employees’ provident fund to 8.15% for FY23, a development that will benefit more than 60 million subscribers of the Employees’ Provident Fund Organization (EPFO). EPFO is one of the World’s largest Social Security Organisations in terms of clientele and the volume of financial transactions undertaken. At present, it maintains ___(A)____accounts (Annual Report 2021-22) pertaining to its members. The Employees’ Provident Fund came into existence with the promulgation of the Employees’ Provident Funds Ordinance on the 15th November, 1951. It was replaced by the Employees’ Provident Funds Act, ___(B)___.
Identify (A) and (B) respectively in the above passage.
(1) 25.74 Crores; 1951
(2) 26.74 Crores; 1951
(3) 27.74 Crores; 1952
(4) 28.74 Crores; 1952
(5) None of these

Ans: 3
Solution:
EPFO is one of the World’s largest Social Security Organisations in terms of clientele and the volume of financial transactions undertaken. At present it maintains 27.74 crore accounts (Annual Report 2021-22) pertaining to its members.
The Employees’ Provident Fund came into existence with the promulgation of the Employees’ Provident Funds Ordinance on the 15th November, 1951. It was replaced by the Employees’ Provident Funds Act, 1952.

2.) Reliance Industries Ltd, Tata Power Solar, and ReNew are among companies chosen to receive ₹14,007 crore worth of government incentives to encourage the local manufacturing of solar modules under its production-linked incentive scheme. The Union Cabinet has cleared Production Linked incentive (PLI) of Rs _ to incentivise manufacturing of domestic solar cell modules.
(1) Rs. 19500 Crores
(2) Rs. 20050 Crores
(3) Rs. 22600 Crores
(4) Rs. 23500 Crores
(5) None of these

Ans: 1
Solution:
The Union Cabinet has cleared Production Linked incentive (PLI) of Rs 19,500 crore to incentivise manufacturing of domestic solar cell modules.

3.) Government of India launched the Ujwal DISCOM Assurance Yojana (UDAY) in November (A). It was designed to turn around the precarious financial position of state power distribution companies (discoms). Discoms are responsible for buying electricity from the generation companies and selling them to consumers.
(1) 2014
(2) 2015
(3) 2016
(4) 2017
(5) None of these

Ans: 2
Solution:
Government of India launched the Ujwal DISCOM Assurance Yojana (UDAY) in November 2015. It was designed to turn around the precarious financial position of state power distribution companies (discoms). Discoms are responsible for buying electricity from the generation companies and selling them to consumers.

4.) Representatives from Japanese businesses and multilateral institutions, including the Asian Development Bank, are slated to visit the Northeast in April to explore significant investments as part of a wider manufacturing push, according to people aware of the matter. The delegates will attend a conference to advance discussions around the “Bay of Bengal-Northeast Industrial Value Chain” in (A)_.
Identify the city (A).
(1) Guwahati
(2) Agartala
(3) Shillong
(4) Kohima
(5) Imphal

Ans: 2
Solution:
Representatives from Japanese businesses and multilateral institutions, including the Asian Development Bank, are slated to visit the Northeast in April to explore significant investments as part of a wider manufacturing push, according to people aware of the matter. The delegates will attend a conference to advance discussions around the “Bay of Bengal-Northeast Industrial Value Chain” in Agartala.

5.) China’s stagnant steel demand and rising exports despite the lifting of pandemic curbs could depress prices worldwide, posing trouble for Indian steel exporters. National Steel Policy which was released in (A) aims to attain a steel production capacity in India of (B) by 2030.
Identify (A) and (B) respectively.
(1) 2015; 300 MT
(2) 2016; 500 MT
(3) 2017; 300 MT
(4) 2018; 500 MT
(5) None of these

Ans: 3
Solution:
National Steel Policy which was released in 2017 aims to attain a steel production capacity in India of 300 MT by 2030. It has a long-term vision to enhance domestic consumption, produce high-quality steel and make the sector globally competitive.

Mint StatiCA MCQs – March 28, 2023

1.) Identify the correct statements about the Insolvency and Bankruptcy Code.
A. Bankruptcy is a situation where individuals or companies are unable to repay their outstanding debt.
B. Insolvency, on the other hand, is a situation whereby a court of competent jurisdiction has declared a person or other entity insolvent, having passed appropriate orders to resolve it and protect the rights of the creditors. It is a legal declaration of one’s inability to pay off debts.
C. Insolvency professional agencies develop professional standards, code of ethics and be first level regulator for insolvency professionals members leading to development of a competitive industry for such professionals.
(1) Only A and B
(2) Only B and C
(3) Only A and C
(4) Only C
(5) A, B and C

Ans: 4
Solution:
Insolvency is a situation where individuals or companies are unable to repay their outstanding debt.
Bankruptcy, on the other hand, is a situation whereby a court of competent jurisdiction has declared a person or other entity insolvent, having passed appropriate orders to resolve it and protect the rights of the creditors. It is a legal declaration of one’s inability to pay off debts.
Insolvency professional agencies develop professional standards, code of ethics and be first level regulator for insolvency professionals members leading to development of a competitive industry for such professionals.

2.) In India, the disinvestment process is conducted by the DIPAM, which comes under the Ministry of Finance.
What is ‘A’ in “DIPAM”?
(1) Account
(2) Asset
(3) Agreement
(4) Absolute
(5) Accumulated

Ans: 2
Solution:
In India, the disinvestment process is conducted by the Department of Investment and Public Asset Management (DIPAM), which comes under the Ministry of Finance.

3.) Government had constituted the National Investment Fund (NIF) in _ into which the proceeds from disinvestment of Central Public Sector Enterprises were to be channelized.
(1) 1999
(2) 2001
(3) 2003
(4) 2005
(5) 2008

Ans: 4
Solution:
Government had constituted the National Investment Fund (NIF) in 2005 into which the proceeds from disinvestment of Central Public Sector Enterprises were to be channelized.

4.) The National Action Plan on Climate Change (NAPCC) was launched in __ by the Prime Minister’s Council on Climate Change.
(1) 2003
(2) 2005
(3) 2008
(4) 2010
(5) 2012

Ans: 3
Solution:
The National Action Plan on Climate Change (NAPCC) was launched in 2008 by the Prime Minister’s Council on Climate Change.
There are 8 national missions forming the core of the NAPCC which represent multi-pronged, long term and integrated strategies for achieving key goals in climate change. These are-
• National Solar Mission
• National Mission for Enhanced Energy Efficiency
• National Mission on Sustainable Habitat
• National Water Mission
• National Mission for Sustaining the Himalayan Ecosystem
• National Mission for A Green India
• National Mission for Sustainable Agriculture
• National Mission on Strategic Knowledge for Climate Change

5.) With India having set a target of achieving net zero carbon emission by _, its actions matter globally and they could help in shaping an “incredibly important” model for other countries, said Manish Bapna, president and chief executive officer of the US-based National Resources Defence Council (NRDC).
(1) 2030
(2) 2050
(3) 2060
(4) 2070
(5) 2100

Ans: 4
Solution:
With India having set a target of achieving net zero carbon emission by 2070, its actions matter globally and they could help in shaping an “incredibly important” model for other countries, said Manish Bapna, president and chief executive officer of the US-based National Resources Defence Council (NRDC).

Mint StatiCA MCQs – March 27, 2023

1.) Choose the correct statements about LIC.
A. LIC is an Indian state-owned insurance group and investment corporation owned by the Government of India.
B. It was founded in 1958 when the Parliament of India passed the Life Insurance of India Act that nationalized the insurance industry in India.
C. Over 245 insurance companies and provident societies were merged to create the state-owned LIC.
(1) Only A and B
(2) Only A and C
(3) Only B and C
(4) Only B
(5) A, B and C

Ans: 2
Solution:
LIC is an Indian state-owned insurance group and investment corporation owned by the Government of India. It was founded in 1956 when the Parliament of India passed the Life Insurance of India Act that nationalized the insurance industry in India. Over 245 insurance companies and provident societies were merged to create the state-owned LIC.

2.) ____ tax is levied not on the income, profit, or revenue but on goods and services used by the taxpayer.
(1) Direct Tax
(2) Revenue Tax
(3) Primary Tax
(4) Indirect Tax
(5) None of these

Ans: 4
Solution:
Indirect Tax
The tax is levied not on the income, profit, or revenue but on goods and services used by the taxpayer. Unlike direct taxes, indirect taxes can be shifted from one individual to another.

3.) The Government of India today announced the sites for setting up of 7 PM Mega Integrated Textile Regions and Apparel (PM MITRA) Parks for the Textile industry. The Parks will come up in Tamil Nadu, Telangana, Gujarat, Karnataka, Madhya Pradesh, Uttar Pradesh and _(A).
Identify (A).
(1) Assam
(2) Jammu and Kashmir
(3) Maharashtra
(4) Odisha
(5) None of these

Ans: 3
Solution:
The Government of India today announced the sites for setting up of 7 PM Mega Integrated Textile Regions and Apparel (PM MITRA) Parks for the Textile industry. The Parks will come up in Tamil Nadu, Telangana, Gujarat, Karnataka, Madhya Pradesh, Uttar Pradesh and Maharashtra.

4.____ is the inflation that takes place when supply falls drastically and demand remains at the same level.
(1) Structural Inflation
(2) Aggregate Supply Inflation
(3) Demand Pull Inflation
(4) Cost Push Inflation
(5) Bottleneck Inflation

Ans: 5
Solution:
Bottleneck inflation is the inflation that takes place when supply falls drastically and demand remains at the same level. This creates excess demand in the economy as the supply cannot match it and thus prices rise Such situations arise due to supply side accidents, hazards, or mismanagement.

5.) In its Report (2001), the National Statistical Commission (NSC), led by __, proposed the compilation of CPI for rural and urban areas.
(1) Bimal Jalan
(2) Montek Singh Ahluwalia
(3) C.Rangarajan
(4) Suresh Tendulkar
(5) None of these

Ans: 3
Solution:
In its Report (2001), the National Statistical Commission (NSC), led by Dr. C. Rangarajan, proposed the compilation of CPI for rural and urban areas.

Mint StatiCA MCQs – March 23, 2023

1.) India, the world’s ___(A)___largest coal producer and a net importer of the fuel, is expected to begin exports by FY26 as the country expects output to exceed local demand over the next two years when supplies from new coal mines enter the market, coal secretary Amrit Lal Meena said.
Identify (A).
(1) 2nd
(2) 3rd
(3) 4th
(4) 5th
(5) 6th

Ans: 1
Solution:
India, the world’s second-largest coal producer and a net importer of the fuel, is expected to begin exports by FY26 as the country expects output to exceed local demand over the next two years when supplies from new coal mines enter the market, coal secretary Amrit Lal Meena said.

2.) Though water supply metrics show some improvement over the last one year, only ___(A)___% of Indian households drink piped water received from their local bodies without purification, according to a survey conducted by LocalCircles in ___(B) ___districts of India.
Identify (A) and (B) respectively.
(1) 10 %; 350
(2) 20 %; 450
(3) 10 %; 450
(4) 20 %; 350
(5) None of these

Ans: 5
Solution:
Though water supply metrics show some improvement over the last one year, only 3% of Indian households drink piped water received from their local bodies without purification, according to a survey conducted by Local Circles in 305 districts of India.
The survey that received over 26,000 responses at a pan-India level shows that supply of piped water to households has increased to 44% this year from 35% last year because of more community involvement at the district and panchayat level and as more quality testing is being conducted under the Jal Jeevan Mission.
To provide safe and adequate drinking water through individual household tap connections by 2024 to all households in rural India, the department of drinking water and sanitation of the Jal Shakti ministry launched the Har Ghar Jal Mission in 2019.

3.) The EU’s Carbon Border Adjustment Mechanism (CBAM)—a tax on the bloc’s imports of iron and steel, cement, fertilisers, aluminium and electricity—comes into force on ____.
(1) October 1, 2023
(2) December 1, 2023
(3) October 1, 2024
(4) December 1, 2024
(5) None of these

Ans: 1
Solution:
The EU’s Carbon Border Adjustment Mechanism (CBAM)—a tax on the bloc’s imports of iron and steel, cement, fertilisers, aluminium and electricity—comes into force on 1 October 2023.

4.____ is the inflation that takes place when supply falls drastically and demand remains at the same level.
(1) Structural Inflation
(2) Aggregate Supply Inflation
(3) Demand Pull Inflation
(4) Cost Push Inflation
(5) Bottleneck Inflation

Ans: 5
Solution:
Bottleneck inflation is the inflation that takes place when supply falls drastically, and demand remains at the same level. This creates excess demand in the economy as the supply cannot match it and thus prices rise Such situations arise due to supply side accidents, hazards, or mismanagement.

5.) The Bretton Woods Conference, officially known as the United Nations Monetary and Financial Conference, was a gathering of delegates from 44 nations that met from July 1 to 22, ___(A)___ in Bretton Woods, New Hampshire (USA), to agree upon a series of new rules for international financial and monetary order after the conclusion of World War II.
Identify (A) in the above passage.
(1) 1944
(2) 1945
(3) 1946
(4) 1950
(5) None of these

Ans: 1
Solution:
The Bretton Woods Conference, officially known as the United Nations Monetary and Financial Conference, was a gathering of delegates from 44 nations that met from July 1 to 22, 1944 in Bretton Woods, New Hampshire (USA), to agree upon a series of new rules for international financial and monetary order after the conclusion of World War II.
The two major accomplishments of the conference were the creation of the International Bank for Reconstruction and Development (IBRD) and International Monetary Fund (IMF).

Mint StatiCA MCQs – March 22, 2023

1.) Section 45ZB of the RBI Act provides for the constitution of a __-member Monetary Policy Committee (MPC) to determine the policy rate required to achieve the inflation target.
(1) 3
(2) 6
(3) 9
(4) 12
(5) 15

Ans: 2
Solution:
Section 45ZB of the RBI Act provides for the constitution of a six-member Monetary Policy Committee (MPC) to determine the policy rate required to achieve the inflation target.

2.) The European Union is willing to collaborate with India in easing the administrative burden for businesses while enforcing its Carbon Border Adjustment Mechanism (CBAM), which seeks to impose tariffs on imports with a high carbon footprint entering the __-member bloc.
(1) 25
(2) 27
(3) 29
(4) 31
(5) 33

Ans: 2
Solution:
The European Union is willing to collaborate with India in easing the administrative burden for businesses while enforcing its Carbon Border Adjustment Mechanism (CBAM), which seeks to impose tariffs on imports with a high carbon footprint entering the 27-member bloc.

3.) Ministry of Textiles, has issued a notification to set up _ Mega Integrated Textile Region and Apparel (PM MITRA) Parks with a total outlay of Rs. 4,445 crores.
(1) 3
(2) 5
(3) 7
(4) 9
(5) 13

Ans: 3
Solution:
Ministry of Textiles has issued a notification to set up 7 Mega Integrated Textile Region and Apparel (PM MITRA) Parks with a total outlay of Rs. 4,445 crores.
The Parks will come up in Tamil Nadu, Telangana, Gujarat, Karnataka, Madhya Pradesh, Uttar Pradesh, and Maharashtra.

4.) The International Solar Alliance (ISA) is an action-oriented, member-driven, collaborative platform for the increased deployment of solar energy technologies. The ISA was conceived as a joint effort by India and _(A) to mobilize efforts against climate change through the deployment of solar energy solutions.
Identify the country (A).
(1) Sweden
(2) UK
(3) France
(4) Japan
(5) USA

Ans: 3
Solution:
The ISA was conceived as a joint effort by India and France to mobilize efforts against climate change through the deployment of solar energy solutions.

5. Which of the following statements is/are not true about Shares?
A. Share is a type of security, which signifies ownership in a corporation and represents a claim on the part of the corporation’s assets and earnings.
B. Profits of companies are sometimes paid in the form of dividends. A higher proportion of shares in a company signifies a higher stake in the profits also.
C. Another extremely important feature of a share is its unlimited liability.
Choose the correct option:
(1) Only B and C
(2) Only B
(3) Only C
(4) Only A and C
(5) Only A and B

Ans: 3
Solution:
Share is a type of security, which signifies ownership in a corporation and represents a claim on the part of the corporation’s assets and earnings.
Profits of companies are sometimes paid in the form of dividends. A higher proportion of shares in a company signifies a higher stake in the profits also.
Another extremely important feature of a share is its limited liability, which means that, as an owner of a share, you are not personally liable if the company is not able to pay its debts.

Mint StatiCA MCQs – March 21, 2023

1.) Starting 1 October, iron, steel, and aluminium exports to European Union countries from across the world will face added scrutiny under the bloc’s CBAM, or carbon tax.
What is ‘A’ in CBAM?
(1) Arrangement
(2) Adjustment
(3) Asset
(4) Association
(5) None of these

Ans: 2
Solution:
Starting 1 October, iron, steel, and aluminium exports to European Union countries from across the world will face added scrutiny under the bloc’s Carbon Border Adjustment Mechanism (CBAM), or carbon tax. Suppliers will have to report greenhouse gas emissions ‘embedded’ in their consignments before taxes are levied from 1 January 2026.

2.) The Supreme Court on Monday directed the Securities and Exchange Board of India (Sebi) to refund ₹300 crore deposited by the National Stock Exchange (NSE) as part of a disgorgement order. The National Stock Exchange is the largest financial market in India established in 1992. The Securities Contracts (Regulation) Act, __, designated it as a stock exchange, and it began operations in 1994.
(1) 1947
(2) 1956
(3) 1962
(4) 1992
(5) None of these

Ans: 2
Solution:
The National Stock Exchange is the largest financial market in India established in 1992. The Securities Contracts (Regulation) Act, 1956, designated it as a stock exchange, and it began operations in 1994.

3.) The Balance of Trade in Services shows net receipts on account of trade in services, (or what are also called invisibles). We can broadly classify invisibles into five groups.
Which of the following is not one of the five groups?
A. Services, such as banking, insurance, shipping civil aviation, royalty, consultancy services, postal services, etc.
B. Investment income, which includes profits and dividends on direct, portfolio and other investments as well as interest charges on bilateral and multilateral loans.
C. Travel both business and tourist
(1) Only A and B
(2) Only B and C
(3) Only A and C
(4) Only B
(5) None of these

Ans: 5
Solution:
Balance of Trade in Services (BOS): The BOS shows net receipts on account of trade in services, (or what are also called invisibles). We can broadly classify invisibles into five groups, viz., (i) services, such as banking, insurance, shipping civil aviation, royalty, consultancy services, postal services, etc. (ii) investment income, which includes profits and dividends on direct, portfolio and other investments as well as interest charges on bilateral and multilateral loans. (iii) travel both business and tourist, (iv) government transfers, and (v) private transfers.

4.) The SHG Bank Linkage Project launched by ___(A)___ in 1992 has blossomed into the world’s largest microfinance project.
Identify the entity (A) in the above excerpt.
(1) RBI
(2) Ministry of Agriculture and Farmers Welfare
(3) NABARD
(4) NAFED
(5) None of these

Ans: 3
Solution:
The SHG Bank Linkage Project launched by NABARD in 1992 has blossomed into the world’s largest microfinance project.

To impart liquidity to money market instruments and help the development of the secondary market in such instruments, the Discount and Finance House of India (DFHI) was set up as a money market institution jointly by the ___(A)___, public sector banks and financial institutions in ___(B)__.
Identify (A) and (B) respectively in the above excerpt.
(1) LIC; 1980
(2) RBI; 1980
(3) LIC; 1988
(4) RBI; 1988
(5) None of these

Ans: 4
Solution:
In order to impart liquidity to money market instruments and help the development of the secondary market in such instruments, the Discount and Finance House of India (DFHI) was set up as a money market institution jointly by the Reserve Bank of India, public sector banks and financial institutions in 1988.

Mint StatiCA MCQs – March 20, 2023

____(A)___often known as pasta wheat or macaroni wheat is also one of the best quality wheat variety in India.
Identify (A) in the above excerpt.
(1) Durum Wheat
(2) Spelt Wheat
(3) Khorasan Wheat
(4) Einkorn Wheat
(5) None of these

Ans: 1
Solution:
Wheat is a Rabi Crop grown between September and December and harvested between February and May.
The total area under the crop is about 29.8 million hectares in the country. Indian wheat is largely a soft/medium-hard, medium protein, white bread wheat, somewhat similar to U.S. hard white wheat.
Durum wheat, often known as pasta wheat or macaroni wheat is also one of the best-quality wheat variety in India.

The Central government has designed a scheme for rewarding states that push
organic farming in a measure to cut ballooning subsidies for chemical fertilizers.
___(A)___ became the first State in India to become fully organic in ___(B)___.
Identify (A) and (B) respectively.
(1) Sikkim; 2015
(2) Madhya Pradesh; 2016
(3) Sikkim; 2016
(4) Madhya Pradesh; 2015
(5) Sikkim; 2017

Ans: 3
Solution:
Sikkim became the first State in India to become fully organic in 2016. Northeast India has traditionally been organic, and the consumption of chemicals is far less than the rest of the country. Similarly, the tribal and island territories have been traditionally practicing organic farming. The major organic exports from India have been flax seeds, sesame, soybean, tea, medicinal plants, rice, and pulses.

3.) With reference to ‘Bitcoins’, sometimes seen in the news which of the following
statements is/are correct?
A. Bitcoins are tracked by the Central Banks of the countries.
B. Anyone with a Bitcoin address can send and receive Bitcoins from anyone else
with a Bitcoin address.
C. Online payments can be sent without either side knowing the identity of the
other.
Select the correct answer using the code given below:
(1) A and B only
(2) B and C only
(3) C only
(4) A, B and C
(5) A and C Only

Ans: 2
Solution:
Bitcoins are not tracked by the Central Banks of the countries. Bitcoin is decentralized thus: Bitcoin does not have a central authority. Anyone with a Bitcoin address can send and receive Bitcoins from anyone else with
a Bitcoin address. Online payments can be sent without either side knowing the identity of the other.

4.) After Paytm’s IPO fiasco, Sebi has turned cautious while giving clearance to the
initial share sales as it has returned the preliminary papers of half a dozen
companies, including Oravel Stays, which runs hospitality chain Oyo, in over two
months. In order to protect investors, SEBI has laid down rules that require
companies to meet certain criteria before they can go to the public to raise funds.
Among other conditions, the company must have net tangible assets of at least
_ in each of the preceding three full years.
(1) Rs 1 Crore
(2) Rs 3 Crores
(3) Rs 5 Crores
(4) Rs 7 Crores
(5) None of these

Ans: 2
Solution:
In order to protect investors, SEBI has laid down rules that require companies to meet certain criteria before they can go to the public to raise funds. Among other conditions, the company must have net tangible assets of at least Rs 3 crore, and a net worth of Rs 1 crore in each of the preceding three full years,
and it must have a minimum average pre-tax profit of Rs 15 crore in at least three of the immediately preceding five years.

5.)____ involves the use of government spending, taxation, and borrowing to
influence both the pattern of economic activity and the level of growth of aggregate
demand, output, and employment.
(1) Fiscal Policy
(2) Monetary Policy
(3) Exchange Rate Policy
(4) Industrial Policy
(5) None of these

Ans: 1
Solution:
Fiscal policy involves the use of government spending, taxation, and borrowing to influence both the pattern of economic activity and the level of growth of aggregate demand, output, and employment.

Mint StatiCA MCQs – March 17, 2023

1. Government had constituted the National Investment Fund (NIF) in ____(A)___ into which the proceeds from the disinvestment of Central Public Sector Enterprises were to be channelized.

Identify (A).

(1) 1995

(2) 1999

(3) 2003

(4) 2007

(5) None of these

Ans: 5

Solution:

Government had constituted the National Investment Fund (NIF) in 2005 into which the proceeds from disinvestment of Central Public Sector Enterprises were to be channelized.

2. These are decentralized markets where buyers and sellers can interact with one another and engage in the trade of customized products, as per their requirements.

Which type of market has been depicted in the above excerpt?

(1) Secondary Market

(2) Cash Market

(3) Futures Market

(4) Exchange Traded Market

(5) Over the Counter Market

Ans: 5

Solution:

Over-the-counter market

These are decentralized markets where buyers and sellers can interact with one another and engage in the trade of customized products, as per their requirements.

3. Which of the following statements is/are correct?

A. National Income is defined as the net value of all economic goods and services produced within the domestic territory of a country in an accounting year plus the net factor income from abroad.

B. According to the Central Statistical Office (CSO) ‘National income is the sum total of factor incomes generated by the normal residents of a country in the form of wages, rent, interest and profit in an accounting year’.

C. CSO was merged with NSSO to form National Statistical Office in 2018.

Choose the correct option:

(1) Only A and B

(2) Only B and C

(3) Only A and C

(4) Only B

(5) A, B and C

Ans: 1

Solution:

National Income is defined as the net value of all economic goods and services produced within the domestic territory of a country in an accounting year plus the net factor income from abroad.

According to the Central Statistical Office (CSO) ‘National income is the sum total of factor incomes generated by the normal residents of a country in the form of wages, rent, interest and profit in an accounting year’.

CSO was merged with NSSO to form National Statistical Office in 2019.

4. Competition Commission of India (CCI) is a statutory body of the Government of India responsible for enforcing the Competition Act, 2002, it was duly constituted in March ____(A)___.

Identify (A).

(1) 2005

(2) 2006

(3) 2007

(4) 2008

(5) 2009

Ans: 5

Solution:

5. Which section of the Companies Act,2013 deals with the qualification of the President and Members of the National Company Law Tribunal (NCLT)?

(1) Section 407

(2) Section 408

(3) Section 409

(4) Section 410

(5) None of these

Ans: 3

Solution:

Section 409 – Qualification of the president and members of the tribunal

Mint StatiCA MCQs – March 16, 2023

1. The __ comprises credit and debit transactions under non-produced non-financial assets and capital transfers between residents and non-residents.
(1) BoP Account
(2) Current Account
(3) Capital Account
(4) Transfer Account
(5) None of these

Ans: 3
Solution:
The capital account comprises credit and debit transactions under non-produced non-financial assets and capital transfers between residents and non-residents. Thus, acquisitions and disposals of non-produced non- financial assets, such as land sold to embassies and sales of leases and licences, as well as transfers which are capital in nature, are recorded under this account.

2. Which of the following statements is/are true about the Production Linked Incentive (PLI) Scheme?
A. The scheme was first introduced in March 2020.
B. In her 2021-2022 Budget speech Finance Minister Nirmala Sitharaman announced an outlay of Rs 1.97 lakh crore for the PLI scheme.
C. The scheme is aimed at boosting domestic manufacturing under the Atmanirbhar Bharat initiative of the government.
Choose the correct option:
(1) Only A and B
(2) Only B and C
(3) Only A and C
(4) Only B
(5) A, B and C

Ans: 5
Solution:
The scheme was first introduced in March 2020 and in her 2021-2022 Budget speech Finance Minister Nirmala Sitharaman announced an outlay of Rs 1.97 lakh crore for the PLI scheme. The scheme is aimed at boosting domestic manufacturing under the Atmanirbhar Bharat initiative of the government. According to the Commerce Ministry, the PLI scheme is expected to boost production worth an estimated $500 billion in five years since its introduction.

3. Following the balance of payments crisis of __, the government commenced on a path of economic liberalisation whereby the economy was opened up to foreign investment and trade, the private sector was encouraged, and the system of quotas and licences was dismantled. Fiscal policy was re-oriented to cohere with these changes.
(1) 1980
(2) 1991
(3) 1997
(4) 2001
(5) None of these

Ans: 2
Solution:
Following the balance of payments crisis of 1991, the government commenced on a path of economic liberalisation whereby the economy was opened up to foreign investment and trade, the private sector was encouraged, and the system of quotas and licences was dismantled. Fiscal policy was re-oriented to cohere with these changes.

4. The global financial crisis that erupted around September __ saw Indian fiscal policy being tested to its limits. The policymakers had to grapple with the impact of the crisis that was affecting the Indian economy through three channels; contagion risks to the financial sector; the negative impact on exports; and the effect on exchange rates.
(1) 2002
(2) 2004
(3) 2006
(4) 2008
(5) None of these


Ans: 4
Solution:
The global financial crisis that erupted around September 2008 saw Indian fiscal policy being tested to its limits. The policymakers had to grapple with the impact of the crisis that was affecting the Indian economy through three channels; contagion risks to the financial sector; the negative impact on exports; and the effect on exchange rates.

5. __ is implemented to allow the money supply to rise in line with national income and the demand for money.
(1) Accommodative Monetary Policy
(2) Easy Monetary Policy
(3) Hawkish Monetary Policy
(4) Neutral Monetary Policy
(5) Both 1 and 2

Ans: 5
Solution:
Accommodative Monetary Policy
It is implemented to allow the money supply to rise in line with national income and the demand for money This is also known as “easy monetary policy” as the policy rates are decreased to encourage more credit.

Mint StatiCA MCQs – March 15, 2023

1. In March last year, just weeks after Moscow’s invasion of Ukraine, seven major Russian banks were removed from the SWIFT international payments system.
What is ‘T’ in ‘SWIFT’?
(1) Transaction
(2) Transfer
(3) Translation
(4) Telecommunication
(5) None of these

Ans: 4
Solution:
SWIFT
It is formally known as the Society for Worldwide Interbank Financial Telecommunication (SWIFT).
It is a trusted messaging system for banks and other financial institutions around the world.
It doesn’t settle any money itself but provides instruction messages for just how to give and receive specific funds.
It is controlled by the central banks of the G10 countries, the European Central Bank, and the National Bank of Belgium.
It was established in 1973 and is based in Belgium.
The Group of Ten is made up of eleven industrial countries (Belgium, Canada, France, Germany, Italy, Japan, the Netherlands, Sweden, Switzerland, the United Kingdom and the United States) which consult and co-operate on economic, monetary and financial matters.

2. The basket’s contents of the Wholesale Price Index (WPI) have been raised from 676 to __ items.
(1) 687
(2) 693
(3) 697
(4) 703
(5) 722

Ans: 3
Solution:
In May 2017, WPI’s base year was updated from 2004-05 to 2011-12.
WPI with Base 2011-12 has a number of key new features:
The basket’s contents have been raised from 676 to 697 items.
To ensure thorough coverage and representativeness, the number of quotations has been increased from 5842 to 8331.
To eliminate the impact of fiscal policy, the new definition of the wholesale price index excludes taxes. This also aligns the new WPI series with worldwide practices by bringing it closer to the Producer Price Index.
In comparison to the Arithmetic mean used in the WPI 2004-05 series, the item level indices are compiled using a statistically robust Geometric mean.
A distinct WPI Food Index has been launched as part of the updated WPI series.
The WPI food index tracks changes in the pricing of food commodities at the producer level.
Food inflation in India might be efficiently monitored using the WPI Food Index and the CPI Food Price Index.

3. What is the weightage of Primary Articles in the Wholesale Price Index (WPI)?
(1) 13.15 %
(2) 22.62 %
(3) 64.23 %
(4) 43.34 %
(5) 33.45 %

Ans: 2
Solution:
The WPI is divided into three sections. The different sections along with their weightage are:
Primary Articles (e.g.- Food Articles, Vegetables, Milk, Minerals, etc.) – 22.62%
Fuel and Power (e.g.- LPG, Petrol, etc.) – 13.15%
Manufactured Goods (e.g.- manufacture of food products, sugar, manufacture of textiles, etc.) – 64.23%

4. Trade Related Aspects of Intellectual Property Rights (TRIPS) is an agreement on international IP rights. TRIPS came into force in _, as part of the agreement that established the World Trade Organisation (WTO).
(1) 1986
(2) 1995
(3) 2000
(4) 2003
(5) None of these

Ans: 2
Solution:
Trade Related Aspects of Intellectual Property Right (TRIPS) is an agreement on international IP rights. TRIPS came into force in 1995, as part of the agreement that established the World Trade Organisation (WTO).

5. It is an overallotment mechanism. __ is an option to allocate shares in excess of the shares which have already been issued to the public. It is a price stability mechanism to provide post-listing price stability to an initial public offering.
Fill in the Blanks.
(1) Call Option
(2) Green Shoe Option
(3) Put Option
(4) American Style Option
(5) European Style Option

Ans: 2
Solution:
Green Shoe Option
It is an overallotment mechanism. Green Shoe Option is an option to allocate shares in excess of the shares which have already been issued to the public. It is a price stability mechanism to provide post-listing price stability to an initial public offering.

Mint StatiCA MCQs – March 14, 2023

1. Stocks extended their losing streak for the third day as concerns about the US banking crisis continued to weigh on investor sentiment. The benchmark Nifty and Sensex indices closed 1.49% and 1.52% lower on Monday, marking their lowest levels in seven months. The National Stock Exchange introduced the NIFTY market index. It is a combination of the words National Stock Exchange and Fifty, which was coined by the NSE on April 21, _.
(1) 1992
(2) 1996
(3) 1998
(4) 2000
(5) 2002

Ans: 2
Solution:
The National Stock Exchange introduced the NIFTY market index. It is a combination of the words National Stock Exchange and Fifty, which was coined by the NSE on April 21, 1996. The NIFTY 50 is a benchmark-based index and the NSE’s flagship, showcasing the top 50 equity stocks listed on the stock exchange.

2. Retail inflation stayed above the Reserve Bank of India’s upper tolerance level of 6% for the second consecutive month in February on costlier cereals, milk, and spices, putting pressure on the central bank to raise interest rates further. Which of the following entities has the highest weightage in the Consumer Price Index (Combined)?
(1) Food and Beverage
(2) Housing
(3) Fuel and Light
(4) Clothing and Footwear
(5) Pan, tobacco, and intoxicants

Ans: 1
Solution:
The following are the primary components of CPI (C): (along with their weights)
• Food and Beverage – 45.86
• Housing – 10.07
• Fuel and Light – 6.84
• Clothing and Footwear – 6.53
• Pan, tobacco, and intoxicants – 2.38
• Miscellaneous – 28.32

3. What is the weightage of Manufactured Goods in the Wholesale Price Index (WPI)?
(1) 13.15 %
(2) 22.62 %
(3) 64.23 %
(4) 43.34 %
(5) 33.45 %

Ans: 3
Solution:
The WPI is divided into three sections. The different sections along with their weightage are:
Primary Articles (e.g.- Food Articles, Vegetables, Milk, Minerals, etc) – 22.62%
Fuel and Power (e.g.- LPG, Petrol etc) – 13.15%
Manufactured Goods (e.g.- manufacture of food products, sugar, manufacture of textiles, etc) – 64.23%

4. The Union Cabinet has approved the new National Education Policy (NEP), 2020 with an aim to introduce several changes in the Indian education system – from the school to college level. Universalization of education from preschool to secondary level with (A) Gross Enrolment Ratio (GER) in school education by (B)__.
Identify (A) and (B) respectively.
(1) 50 %; 2030
(2) 50 %; 2035
(3) 100 %; 2030
(4) 100 %; 2035
(5) None of these

Ans: 3
Solution:
The Union Cabinet has approved the new National Education Policy (NEP), 2020 with an aim to introduce several changes in the Indian education system – from the school to college level.
Universalization of education from preschool to secondary level with 100% Gross Enrolment Ratio (GER) in school education by 2030.

5. __ is an organization that holds securities (like shares, debentures, bonds, government securities, mutual fund units, etc.) of investors in electronic form at the request of the investors.
Fill in the Blanks.
(1) Stockbroker
(2) Depository
(3) Underwriter
(4) Merchant Banker
(5) None of these

Ans: 2
Solution:
A depository is an organization that holds securities (like shares, debentures, bonds, government securities, mutual fund units etc.) of investors in electronic form at the request of the investors through a registered Depository Participant. They also provide the safekeeping of securities. They also help in other functions like pledges, hypothecation, stock lending and borrowing etc. In India, there are two depositories namely National Securities Depository Limited (NSDL) and Central Depository Services (India) Limited (CDSL).

Mint StatiCA MCQs – March 13, 2023

1. At least three Indian banks—Axis Bank, Kotak Mahindra Bank, and ICICI Bank—along with the Indian unit of HSBC, assembled crack teams to work through the weekend to assist startups affected by the collapse of Silicon Valley Bank open new dollar accounts in Gujarat’s Gift City, which caters to non-residents and offshore entities. _ is the chairman of the International Financial Services Centre (IFSC) at GIFT City (Gujarat International Finance Tec-City).
(1) Injeti Srinivas
(2) T.V. Somanathan
(3) Amitabh Kant
(4) Urjit Patel
(5) None of these

Ans: 1
Solution:
The Appointments Committee of the Cabinet approved the appointment of Injeti Srinivas as chairman of the International Financial Services Centres Authority (IFSCA).
The former corporate affairs secretary and 1983-batch IAS officer of the Odisha cadre will head the IFSCA for a period of three years.

2. Competition Commission of India (CCI) is a statutory body of the Government of India responsible for enforcing the Competition Act, _(A), it was duly constituted in March 2009.
Identify (A).
(1) 1995
(2) 2002
(3) 2005
(4) 2008
(5) None of these

Ans: 2
Solution:
Competition Commission of India (CCI) is a statutory body of the Government of India responsible for enforcing the Competition Act, 2002, it was duly constituted in March 2009.
The Monopolies and Restrictive Trade Practices Act, 1969 (MRTP Act) was repealed and replaced by the Competition Act, 2002, on the recommendations of the Raghavan committee.

3. The Unified Payments Interface (UPI) payments system has cut across the Indian socio-economic digital divide amazingly fast. It is an advanced version of IMPS- round–the–clock funds transfer service to make cashless payments faster, easier, and smoother.
What is ‘S’ in IMPS?
(1) Standard
(2) Simplified
(3) Statement
(4) Service
(5) None of these

Ans: 4
Solution:
Unified Payments Interface (UPI)
It is an advanced version of Immediate Payment Service (IMPS)- round–the-clock funds transfer service to make cashless payments faster, easier, and smoother.
UPI is a system that powers multiple bank accounts into a single mobile application (of any participating bank), merging several banking features, seamless fund routing & merchant payments into one hood.

4. As per the regulator IRDAI’s mandate, the minimum solvency ratio insurance companies must maintain is __ to lower risks.
(1) 1
(2) 1.5
(3) 2
(4) 2.5
(5) None of these

Ans: 2
Solution:
As per the regulator IRDAI’s mandate, the minimum solvency ratio insurance companies must maintain is 1.5 to lower risks. In terms of solvency margin, the required value is 150%.

5. The government is set to appoint a full-time chairman at Life Insurance Corporation of India (LIC) by June, a finance ministry official said, ending speculation over the vacant top job in the state-owned insurer. It was set up in __.
(1) 1952
(2) 1956
(3) 1960
(4) 1966
(5) None of these

Ans: 2
Solution:
Life Insurance Corporation of India
LIC is a government-owned insurance and investment company arising from the Life Insurance Act of India.
It is a statutory body.
Established in 1956.
HQ: Mumbai
Its aim is to provide citizens with a higher return on economic security through services and products than most other investment players on the market, thereby helping them build a particular quality of life and providing economic development.

Mint StatiCA MCQs – March 10, 2023

1. India is poised to transition to a new domestic gas pricing regime based on crude oil prices starting 1 April, as suggested by the _.
(1) Hardeep S Puri Panel
(2) Rameswar Teli Panel
(3) Kirit Parikh Panel
(4) Manoj Soni Panel
(5) None of these

Ans: 3
Solution:
India is poised to transition to a new domestic gas pricing regime based on crude oil prices starting 1 April, as suggested by the Kirit Parikh panel.
In November, the Kirit Parikh committee made several recommendations to the petroleum ministry, including pricing locally produced natural gas at 10% of crude oil prices and implementing a floor price of $4 per mmBtu and a ceiling price of $6.5 per mmBtu.

2. India and the US are set to sign an accord on semiconductor cooperation, US commerce secretary Gina Raimondo told reporters, adding the two countries had held a dialogue about coordinating semiconductor incentive schemes to prevent a subsidy race and glut in the supply of this vital technology. In 2021, India announced its roughly (A) Production-Linked Incentive (PLI) scheme to encourage semiconductor and display manufacturing in the country.
Identify (A) in the above excerpt.
(1) $ 1 bn
(2) $ 2 bn
(3) $ 5 bn
(4) $ 10 bn
(5) $ 12 bn

Ans: 4
Solution:
In 2021, India announced its roughly $10 billion dollar Production-Linked Incentive (PLI) scheme to encourage semiconductor and display manufacturing in the country.

3. The Minister of Commerce and Industry, Minster of Consumer Affairs, Food and Public Distribution, and Minister of Textiles, Government of India, Shri Piyush Goyal, and Minister for Trade, Tourism and Investment, Government of Australia, Mr. Dan Tehan, MP, signed the India-Australia ECTA in 2022, in a Virtual ceremony in presence of the Prime Minister of India, Shri Narendra Modi and Prime Minister of Australia, Mr. Scott Morrison.
What is ‘C’ in ‘ECTA’?
(1) Commerce
(2) Cooperation
(3) Council
(4) Conference
(5) Collaboration

Ans: 2
Solution:
The Minister of Commerce and Industry, Minster of Consumer Affairs, Food and Public Distribution, and Minister of Textiles, Government of India, Shri Piyush Goyal, and Minister for Trade, Tourism and Investment, Government of Australia, Mr. Dan Tehan, MP, signed the India-Australia Economic Cooperation and Trade Agreement (ECTA) in 2022, in a Virtual ceremony in presence of the Prime Minister of India, Shri Narendra Modi and Prime Minister of Australia, Mr. Scott Morrison.
The India-Australia ECTA is the first trade agreement of India with a developed country after more than a decade. The Agreement encompasses cooperation across the entire gamut of bilateral economic and commercial relations between the two friendly countries and covers areas like Trade in Goods, Rules of Origin, Trade in Services, Technical Barriers to Trade (TBT), Sanitary and Phytosanitary (SPS) measures, Dispute Settlement, Movement of Natural Persons, Telecom, Customs Procedures, Pharmaceutical products, and Cooperation in other Areas. Eight subject-specific side letters covering various aspects of bilateral economic cooperation were also concluded as part of the Agreement.

4. Considering the importance of availability of labour force data at more frequent time intervals, National Statistical Office (NSO) launched Periodic Labour Force Survey (PLFS) in April __.
(1) 2015
(2) 2016
(3) 2017
(4) 2018
(5) None of these

Ans: 3
Solution:
Considering the importance of availability of labour force data at more frequent time intervals, National Statistical Office (NSO) launched Periodic Labour Force Survey (PLFS) in April 2017.
The objective of PLFS is primarily twofold:
to estimate the key employment and unemployment indicators (viz. Worker Population Ratio, Labour Force Participation Rate, Unemployment Rate) in the short time interval of three months for the urban areas only in the ‘Current Weekly Status’ (CWS).
to estimate employment and unemployment indicators in both ‘Usual Status’ (ps+ss) and CWS in both rural and urban areas annually.

5. With the objective of ensuring greater financial inclusion and increasing outreach of the banking sector, RBI, in January _ permitted banks to use intermediaries as Business Facilitators (BF) or Business Correspondents (BC) for providing financial and banking services. Scheduled Commercial Banks (SCBs) including Regional Rural Banks (RRBs), Local Area Banks (LABs), the recently licensed Payments and Small Finance Banks have been permitted to use the services of Business Facilitator / Business Correspondents.
(1) 1999
(2) 2003
(3) 2006
(4) 2008
(5) 2010

Ans: 3
Solution:
With the objective of ensuring greater financial inclusion and increasing outreach of the banking sector, RBI, in January 2006 permitted banks to use intermediaries as Business Facilitators (BF) or Business Correspondents (BC) for providing financial and banking services. Scheduled Commercial Banks (SCBs) including Regional Rural Banks (RRBs), Local Area Banks (LABs), the recently licensed Payments and Small Finance Banks have been permitted to use the services of Business Facilitator / Business Correspondents.

Mint StatiCA MCQs – March 6, 2023

1. In (A), the Securities and Exchange Board of India (Sebi) changed the rules of AT-I bonds, which are bonds without a fixed maturity date or perpetuals, to be considered as having a maturity of _(B) beginning April 2023, starting from their sale date.
Identify (A) and (B) respectively.
(1) 2019; 50 Years
(2) 2021; 50 Years
(3) 2019; 100 Years
(4) 2021; 100 Years
(5) None of these

Ans: 4
Solution:
In 2021, the Securities and Exchange Board of India (Sebi) changed the rules of AT-I bonds, which are bonds without a fixed maturity date or perpetuals, to be considered as having a maturity of 100 years beginning April 2023, starting from their sale date.
AT-1 bonds are a type of unsecured, perpetual bonds that banks issue to shore up their core capital base to meet the Basel-III norms.

2. The Insolvency and Bankruptcy Board of India (IBBI) has sent a strong message to professionals who have allegedly erred while steering sinking companies through the bankruptcy resolution process by taking action against a dozen such officials since January. The Insolvency and Bankruptcy Board of India was established on __ under the Insolvency and Bankruptcy Code.
(1) 1st October 2015
(2) 1st October 2016
(3) 1st January 2015
(4) 1st January 2016
(5) None of these

Ans: 2
Solution:
The Insolvency and Bankruptcy Board of India was established on 1st October, 2016 under the Insolvency and Bankruptcy Code, 2016. It is a key pillar of the ecosystem responsible for implementation of the Code that consolidates and amends the laws relating to reorganization and insolvency resolution of corporate persons, partnership firms and individuals in a time bound manner for maximization of the value of assets of such persons, to promote entrepreneurship, availability of credit and balance the interests of all the stakeholders.
It is a unique regulator: regulates a profession as well as processes. It has regulatory oversight over the Insolvency Professionals, Insolvency Professional Agencies, Insolvency Professional Entities and Information Utilities. It writes and enforces rules for processes, namely, corporate insolvency resolution, corporate liquidation, individual insolvency resolution and individual bankruptcy under the Code. It has recently been tasked to promote the development of, and regulate, the working and practices of, insolvency professionals, insolvency professional agencies and information utilities and other institutions, in furtherance of the purposes of the Code. It has also been designated as the ‘Authority’ under the Companies (Registered Valuers and Valuation Rules), 2017 for regulation and development of the profession of valuers in the country.

3. __ is defined as a bank account that is created by any commercial bank of a given country in some other foreign country with the currency of a foreign country.
(1) Nostro Account
(2) Loro Account
(3) Vostro Account
(4) Foreign Investment Account
(5) None of these


Ans: 1
Solution:
Nostro Account is defined as a bank account that is created by any commercial bank of a given country in some other foreign country with the currency of a foreign country. It is derived from the Italian word Nostro account meaning ‘Our.’ Simply, the money that is in your bank is in your country’s currency. The interest rate is not given to this type of account. If a case arises where there is an overdrawn amount or there is an insufficient amount of funds in the Nostro account, some fee is charged from these accounts.
The account of the State bank of India was held with the Bank of America in dollars. In simple words, if there is any bank in India that opens an account to deal with transactions with a foreign bank, it will be called a Nostro Account. A Nostro account is used as a specialized, additional feature that is mostly found in the countries with the facility of convertible currencies. Banks usually tie up with third parties to enable transactions like these. The banks with Nostro facilities are also called facilitator banks. These banks are not at all similar to deposit accounts, and they demand that the deposit accounts are kept in the same currency as a bank.

4. Prominent Economist like, C N Vakil and P R Brahmananda advocated __ for the development of the Indian economy and Industrialisation.
(1) Mahalonobis Model
(2) Harrod–Domar model
(3) Rostov Model
(4) Aggregate Demand Model
(5) None of these

Ans: 5
Solution:
The Wage Good Model: Prominent Economist like, C N Vakil and P R Brahmananda advocated Wage Good model for the development of the Indian economy and Industrialisation. Vakil and Brahamanda differed from the Mahalanobis strategy as they believe “At the low level of consumption (this was the situation in India) the productivity of the workers depends on how much they consumed. According to them, if people were undernourished, they will lose their productivity and become less efficient, at this juncture it is necessary to feed them to increase their productivity. But this is not true for all consumer good; so they differentiated between Wage Good (whose consumption increases workers’ productivity) and Non-Wage Good (whose consumption did not increase the same).

5. India’s path to becoming a $7 trillion economy by 2030 is dependent on two key areas of growth: manufacturing and infrastructure creation, the two fundamental pillars of economic development in a fast-evolving nation like India, poised to soon become the fastest growing economy in the world, according to a panel of key industry stakeholders from the infrastructure and manufacturing sectors at the Mint Zetwerk Smart Manufacturing Summit. Make in India is a national initiative launched by the Government of India in __. It is aimed at transforming India into a global design and manufacturing hub.
(1) 2012
(2) 2014
(3) 2015
(4) 2016
(5) None of these

Ans: 2
Solution:
Make in India is a national initiative launched by the Government of India in 2014. It is aimed at transforming India into a global design and manufacturing hub.

Mint StatiCA MCQs – March 3, 2023

1. The Supreme Court set up a _____member panel to investigate allegations of regulatory failure and breach of laws against the Adani group and to suggest steps to bolster the statutory and supervisory regime following a bombshell report from US short-seller Hindenburg Research that wiped more than $140 billion in Adani group stocks’ market value.
(1) 3
(2) 5
(3) 7
(4) 9
(5) None of these

Ans: 5
Solution:
The Supreme Court set up a six-member panel to investigate allegations of regulatory failure and breach of laws against the Adani group and to suggest steps to bolster the statutory and supervisory regime following a bombshell report from US short-seller Hindenburg Research that wiped more than $140 billion in Adani group stocks’ market value.

2. The Reserve Bank has taken proactive policy measures to promote green finance. It has joined the Central Banks and Supervisors NGFS in April 2021.
What is ‘N’ in ‘NGFS’?
(1) Nominal
(2) Notional
(3) National
(4) Notified
(5) None of these

Ans: 5
Solution:
The Reserve Bank has taken proactive policy measures to promote green finance. It has joined the Central Banks and Supervisors Network for Greening the Financial System (NGFS) in April 2021.

3. The government is considering overhauling India’s patent rules to spur innovation and research and development (R&D), union minister for commerce and industry Piyush Goyal said, as he sought suggestions from industry and stakeholders to revamp the country’s legal framework. Patents Act, 1970, this principal law for patenting system in India came into force in the year __. It replaced the Indian Patents and Designs Act 1911.
(1) 1971
(2) 1972
(3) 1975
(4) 1977
(5) None of these


Ans: 2
Solution:
Patents Act, 1970: This principal law for patenting system in India came into force in the year 1972. It replaced the Indian Patents and Designs Act 1911.

4. __ is implemented to allow the money supply to rise in line with national income and the demand for money.
(1) Accommodative Monetary Policy
(2) Easy Monetary Policy
(3) Hawkish Monetary Policy
(4) Neutral Monetary Policy
(5) Both 1 and 2

Ans: 5
Solution:
Accommodative Monetary Policy
It is implemented to allow the money supply to rise in line with national income and the demand for money This is also known as “easy monetary policy” as the policy rates are decreased to encourage more credit.

5. Section _ of RBI Act,1934 says every non-banking financial company shall create a reserve fund and transfer therein a sum not less than twenty percent of its net profit every year as disclosed in the profit and loss account and before any dividend is declared.
(1) 45 ZA
(2) 45 ZB
(3) 45 IC
(4) 45 IA
(5) None of these

Ans: 3
Solution:
Section 45-IC of RBI Act,1934 says every non-banking financial company shall create a reserve fund and transfer therein a sum not less than twenty percent of its net profit every year as disclosed in the profit and loss account and before any dividend is declared.

Mint StatiCA MCQs – March 2, 2023

1. The Policy statement intended to bring about the integrated development of the small and large sector, thus, promoting economic federalism in the country. It states: “It will be the Government’s endeavour to reverse the trends of the last three years towards creating artificial divisions between small and large-scale industry under the misconception that their interests are essentially conflicting. While making all efforts towards integrated industrial development it is proposed to promote the concept of economic federalism with the setting up of a few nucleus plants in each district, identified as industrially backward, to generate as many ancillaries and small and cottage units as possible.”
Which industrial policy has been depicted in the above passage?
(1) Industrial Policy, 1956
(2) Industrial Policy, 1977
(3) Industrial Policy, 1980
(4) Industrial Policy, 1991
(5) None of these

Ans: 3
Solution:
The Policy statement of the Industrial Policy, 1980 intended to bring about the integrated development of the small and large sector, thus, promoting economic federalism in the country. It states: “It will be the Government’s endeavour to reverse the trends of the last three years towards creating artificial divisions between small and large-scale industry under the misconception that their interests are essentially conflicting. While making all efforts towards integrated industrial development it is proposed to promote the concept of economic federalism with the setting up of a few nucleus plants in each district, identified as industrially backward, to generate as many ancillaries and small and cottage units as possible.”

2. Which of the following is incorrect about the objectives of the National Manufacturing Policy, 2011?
A. Increase manufacturing sector growth to 12-14% over the medium term to make it the engine of growth for the economy. The 2 to 4 % differential over the medium-term growth rate of the overall economy will enable manufacturing to contribute at least 25% of the National GDP by 2025.
B. Increase the rate of job creation in manufacturing to create 100 million additional jobs by 2025.
C. Creation of appropriate skill sets among the rural migrant and urban poor to make growth inclusive.
(1) Only A and B
(2) Only B and C
(3) Only A and C
(4) Only B
(5) A, B and C

Ans: 1
Solution:
Objectives of National Manufacturing Policy
The government of India decided to bring out the National Manufacturing Policy to bring about a quantitative and qualitative change with the following six objectives:
Increase manufacturing sector growth to 12-14% over the medium term to make it the engine of growth for the economy. The 2 to 4 % differential over the medium-term growth rate of the overall economy will enable manufacturing to contribute at least 25% of the National GDP by 2022.
Increase the rate of job creation in manufacturing to create 100 million additional jobs by 2022.
Creation of appropriate skill sets among the rural migrant and urban poor to make growth inclusive.
Increase domestic value addition and technological ‘depth’ in manufacturing.
Enhance global competitiveness of Indian manufacturing through appropriate policy support.
Ensure sustainability of growth, particularly with regard to the environment including energy efficiency, optimal utilization of natural resources and restoration of damaged/ degraded eco-systems

3. India’s manufacturing sector expanded at the slowest pace in four months in February but remained relatively robust amid strong underlying domestic demand despite inflationary pressures, according to a survey by S&P Global.
Identify the correct statements about S&P Global India Manufacturing Purchasing Managers’ Index (PMI).
A. It is a survey-based measure that asks the respondents about changes in their perception about key business variables as compared with the previous month.
B. The PMI is a number from 0 to 100.
C. A print above 50 means expansion, while a score below that denotes contraction.
(1) Only A and B
(2) Only B and C
(3) Only A and C
(4) Only B
(5) A, B and C


Ans: 5
Solution:
Purchasing Managers Index
It is a survey-based measure that asks the respondents about changes in their perception about key business variables as compared with the previous month.
The purpose of the PMI is to provide information about current and future business conditions to company decision makers, analysts, and investors.
It is calculated separately for the manufacturing and services sectors and then a composite index is also constructed.
The PMI is a number from 0 to 100.
A print above 50 means expansion, while a score below that denotes contraction.
A reading of 50 indicates no change.

4. Which of the following countries is not a G 20 Country?
(1) Iran
(2) India
(3) Australia
(4) South Africa
(5) Brazil

Ans: 1
Solution:
The G20 was formed in 1999 in the backdrop of the financial crisis of the late 1990s that hit East Asia and Southeast Asia in particular.
It aims to secure global financial stability by involving middle-income countries.
Together, the G20 countries include 60% of the world’s population, 80% of global GDP, and 75% of global trade.
Members:
Argentina, Australia, Brazil, Canada, China, France, Germany, India, Indonesia, Italy, Japan, Republic of Korea, Mexico, Russia, Saudi Arabia, South Africa, Turkey, the United Kingdom, the United States and the EU.

5. _ are the receipts which the residents of a country get for ‘free’, without having to provide any goods or services in return. They consist of gifts, remittances and grants. They could be given by the government or by private citizens living abroad.
(1) Capital Account
(2) Transfer Payments
(3) Balance of Payments
(4) Current Account
(5) None of these

Ans: 2
Solution:
Transfer payments are the receipts which the residents of a country get for ‘free’, without having to provide any goods or services in return. They consist of gifts, remittances and grants. They could be given by the government or by private citizens living abroad.

Mint StatiCA MCQs – March 1, 2023

1. Identify the correct statements about India’s GDP Growth.
A. India’s economic growth rate slowed to 3.4% in the December quarter.
B. Slowed growth is because of rising interest rates and elevated raw material prices which rippled through the economy while global trade languished.
C. Data from the ministry of statistics and programme implementation showed that manufacturing output contracted by 1.1% in the December quarter, the second consecutive quarter after a 3.6% contraction seen in the preceding three months.
(1) Only A and B
(2) Only B and C
(3) Only A and C
(4) Only B
(5) A, B and C

Ans: 2
Solution:
India’s economic growth rate slowed to 4.4% in the December quarter.
Slowed growth is because of rising interest rates and elevated raw material prices which rippled through the economy while global trade languished.
Data from the ministry of statistics and programme implementation showed that manufacturing output contracted by 1.1% in the December quarter, the second consecutive quarter after a 3.6% contraction seen in the preceding three months.

2. The Reserve Bank of India (RBI) is examining the utilization of sanctioned bank loans by the Adani group to determine if the company is relying excessively on domestic lenders to fund its operations and expansion plans.
Identify the correct statements about the Reserve Bank of India.
A. The Reserve Bank of India was established on April 1, 1934 in accordance with the provisions of the Reserve Bank of India Act, 1934.
B. The Central Office of the Reserve Bank was initially established in Calcutta but was permanently moved to Mumbai in 1937. The Central Office is where the Governor sits and where policies are formulated.
C. Though originally privately owned, since nationalization in 1950, the Reserve Bank is fully owned by the Government of India.
(1) Only A and B
(2) Only B and C
(3) Only A and C
(4) Only B
(5) A, B and C

Ans: 4
Solution:
The Reserve Bank of India was established on April 1, 1935 in accordance with the provisions of the Reserve Bank of India Act, 1934.
The Central Office of the Reserve Bank was initially established in Calcutta but was permanently moved to Mumbai in 1937. The Central Office is where the Governor sits and where policies are formulated.
Though originally privately owned, since nationalization in 1949, the Reserve Bank is fully owned by the Government of India.

3. Driven by a strong jump in coal, electricity and fertilizer production, core sector output nearly doubled in January compared with the previous year, data released by the commerce ministry showed.
Which of the following has the highest weightage in the Index of Eight Core Industries?
(1) Fertilizer
(2) Coal
(3) Crude Oil
(4) Electricity
(5) None of these

Ans: 5
Solution:
The Eight Core Industries include Natural Gas, Coal, Refinery Products, Crude Oil, Cement, Electricity, Steel, and Fertilizers. These industries are termed as the core industries owing to their strong impact on general economic activities and other industrial activities.
These industries comprise a total of 40.27% of the total weight of the overall items that are included in the Index of Industrial Production (IIP). The Index of Eight Core Industries highest weight is currently possessed by the Refinery Products Industry. Earlier the highest weightage was given to the Electricity Industry. In the decreasing order of the weightage of these industries, the list is stated as below:
Refinery Products Industry
Electricity Industry
Steel Industry
Coal Industry
Crude Oil Industry
Natural Gas Industry
Cement Industry
Fertilizers Industry

4. What is the weightage of Primary Articles in the Wholesale Price Index (WPI)?
(1) 13.15 %
(2) 22.62 %
(3) 64.23 %
(4) 43.34 %

Ans: 2
Solution:
The WPI is divided into three sections. The different sections along with their weightage are:
Primary Articles (e.g.- Food Articles, Vegetables, Milk, Minerals, etc.) – 22.62%
Fuel and Power (e.g.- LPG, Petrol, etc.) – 13.15%
Manufactured Goods (e.g.- manufacture of food products, sugar, manufacture of textiles, etc.) – 64.23%

5. __ is the difference between fiscal deficit and interest payments.
(1) Primary Deficit
(2) Fiscal Deficit
(3) Revenue Deficit
(4) Tax Deficit
(5) Effective Revenue Deficit

Ans: 1
Solution:
Primary deficit is the difference between fiscal deficit and interest payments.

Mint StatiCA MCQs – Feb 28, 2023

1. Following the recommendations of the _ report, in 1999, the Insurance Regulatory and Development Authority (IRDA) was constituted as an autonomous body to regulate and develop the insurance industry. The IRDA was incorporated as a statutory body in April, 2000. The key objectives of the IRDA include promotion of competition so as to enhance customer satisfaction through increased consumer choice and lower premiums, while ensuring the financial security of the insurance market.
(1) Manmohan Singh Committee
(2) Narasimham Committee
(3) Malhotra Committee
(4) Tarapore Committee
(5) None of these

Ans: 3
Solution:
Following the recommendations of the Malhotra Committee report, in 1999, the Insurance Regulatory and Development Authority (IRDA) was constituted as an autonomous body to regulate and develop the insurance industry. The IRDA was incorporated as a statutory body in April, 2000. The key objectives of the IRDA include promotion of competition so as to enhance customer satisfaction through increased consumer choice and lower premiums, while ensuring the financial security of the insurance market.

2. Concept of “effective revenue deficit” was introduced in the Budget _.
(1) 2008-09
(2) 2011-12
(3) 2013-14
(4) 2015-16
(5) None of these

Ans: 2
Solution:
Effective revenue deficit is a difference between grants for capital assets creation and revenue deficit. It aims to deduct the assets exhausted from borrowing to finance capital expenditure.
The concept of “effective revenue deficit” was introduced in the Budget 2011-12.

3. To provide the basis for this assignment and give medium term guidance on fiscal matters, the Constitution provides for the formation of a Finance Commission (FC) every __ years.
(1) 2
(2) 3
(3) 5
(4) 7
(5) None of these

Ans: 3
Solution:
To provide the basis for this assignment and give medium term guidance on fiscal matters, the Constitution provides for the formation of a Finance Commission (FC) every five years.

4. According to the IMF, “an institutional unit is resident in an economic territory when there exists, within the economic territory, some location, dwelling, place of production, or other premises on which or from which the unit engages and intends to continue engaging, either indefinitely or over a finite but long period of time, in economic activities and transactions on a significant scale”. The actual or intended location for _(A) or more is used as an operational criterion for the purpose of determining residence.
Identify (A) in the above passage.
(1) 1 Year
(2) 2 Years
(3) 4 Years
(4) 5 Years
(5) None of these

Ans: 1
Solution:
According to the IMF, “an institutional unit is resident in an economic territory when there exists, within the economic territory, some location, dwelling, place of production, or other premises on which or from which the unit engages and intends to continue engaging, either indefinitely or over a finite but long period of time, in economic activities and transactions on a significant scale”. The actual or intended location for one year or more is used as an operational criterion for the purpose of determining residence. Based on the above definition, each institutional unit is a resident of one and only one economic territory determined by its centre of predominant economic interest.

5. India has a _ share of the global trade in textiles and apparel.
(1) 2 %
(2) 4 %
(3) 6 %
(4) 10 %
(5) 12 %

Ans: 2
Solution:
India has a 4% share of the global trade in textiles and apparel.

Mint StatiCA MCQs – Feb 24, 2023

1. The Bharatmala Pariyojana is a project introduced for the construction of a better road transport system in India. This project was launched on __, by Mr Nitin Gadkari who was the Minister of Road Transport and Highways of India under Prime Minister Narendra Modi to improve the road network in the country.
(1) January 25, 2017
(2) October 25, 2017
(3) October 25, 2018
(4) December 25, 2018
(5) None of these

Ans: 2
Solution:
The Bharatmala Pariyojana is a project introduced for the construction of a better road transport system in India. This project was launched on October 25, 2017, by Mr Nitin Gadkari who was the Minister of Road Transport and Highways of India under Prime Minister Narendra Modi to improve the road network in the country.

2. US president Joe Biden nominated Indian-American business executive Ajay Banga to become president of the World Bank, lauding his experience forging public-private partnerships to address financial inclusion and climate change.
Identify the correct statements about the World Bank Group?
A. Founded in 1944, the International Bank for Reconstruction and Development (IBRD) — soon called the World Bank — has expanded to a closely associated group of five development institutions.
B. International Centre for Settlement of Investment Disputes (ICSID) founded in 1966 settles investment disputes between investors and countries.
C. To become a member of the World Bank, under the IBRD Articles of Agreement, a country must first join the United Nations General Assembly (UNGA).
(1) Only A and B
(2) Only B and C
(3) Only A and C
(4) Only B
(5) A, B and C

Ans: 1
Solution:
Founded in 1944, the International Bank for Reconstruction and Development (IBRD) — soon called the World Bank — has expanded to a closely associated group of five development institutions.
The International Centre for Settlement of Investment Disputes (ICSID) founded in 1966 settles investment disputes between investors and countries.
To become a member of the Bank, under the IBRD Articles of Agreement, a country must first join the International Monetary Fund (IMF).

3. Finance minister Nirmala Sitharaman held bilateral discussions with her counterparts ahead of the G20 finance and central bank governors meeting in _ on key issues like a regulatory framework for crypto assets, preparedness for future pandemic-like shocks and strengthening multilateral development banks.
(1) New Delhi
(2) Mumbai
(3) Chennai
(4) Kolkata
(5) None of these

Ans: 5
Solution:
Finance minister Nirmala Sitharaman held bilateral discussions with her counterparts ahead of the G20 finance and central bank governors meeting in Bengaluru on key issues like a regulatory framework for crypto assets, preparedness for future pandemic-like shocks, and strengthening multilateral development banks.

4. (A)_ depicts temporary slowing of the pace of price inflation and is used to describe instances when the inflation rate has reduced marginally over the short term.
Identify (A).
(1) Stagflation
(2) Deflation
(3) Disinflation
(4) Hyperinflation
(5) None of these

Ans: 3
Solution:
Disinflation
Temporary slowing of the pace of price inflation and is used to describe instances when the inflation rate has reduced marginally over the short term.

5. What is the weightage of Manufactured Goods in the Wholesale Price Index (WPI)?
(1) 13.15 %
(2) 22.62 %
(3) 64.23 %
(4) 43.34 %
(5) 33.45 %

Ans: 3
Solution:
The WPI is divided into three sections. The different sections along with their weightage are:
Primary Articles (e.g.- Food Articles, Vegetables, Milk, Minerals, etc.) – 22.62%
Fuel and Power (e.g.- LPG, Petrol, etc.) – 13.15%
Manufactured Goods (e.g.- manufacture of food products, sugar, manufacture of textiles, etc.) – 64.23%

Mint StatiCA MCQs – Feb 23, 2023

1. Indian stocks saw a sharp decline, mirroring a similar trend in the US markets, with the benchmark indices Sensex and Nifty each falling more than 1.5% as investor sentiment soured because of the continued uncertainty over future rate hike decisions by the US Federal Reserve.
Who is the present chairperson of the US Federal Reserve?
(1) Jerome Powell
(2) Janet Yellen
(3) Michael Barr
(4) Miki Bowman
(5) None of these

Ans: 1
Solution:
Jerome Hayden “Jay” Powell (born February 4, 1953) is an American attorney and investment banker who has served as the 16th chair of the Federal Reserve since 2018.
The Federal Reserve System is the central banking system of the United States. It was created on December 23, 1913, with the enactment of the Federal Reserve Act, after a series of financial panics led to the desire for central control of the monetary system in order to alleviate financial crises.

2. The Mumbai bench of the National Company Law Tribunal (NCLT) admitted an insolvency petition against homegrown media company Zee Entertainment Enterprises Ltd (ZEE) filed by its financial creditor IndusInd Bank, under __ of the Insolvency and Bankruptcy Code (IBC).
(1) Section 2
(2) Section 7
(3) Section 13
(4) Section 17
(5) None of these

Ans: 2
Solution:
The Mumbai bench of the National Company Law Tribunal (NCLT) admitted an insolvency petition against homegrown media company Zee Entertainment Enterprises Ltd (ZEE) filed by its financial creditor IndusInd Bank, under Section 7 of the Insolvency and Bankruptcy Code (IBC).
Section 7 of IBC – Insolvency and Bankruptcy Code, 2016: Initiation of corporate insolvency resolution process by financial creditor.

3. Members of the Reserve Bank of India’s monetary policy committee (MPC) agreed to raise the repo rate by 25 basis points in February as it felt a pause would be premature in its fight against runaway prices, showed minutes of the meeting.
What is the quorum for the MPC meeting?
(1) 2
(2) 4
(3) 6
(4) 8
(5) None of these

Ans: 2
Solution:
The quorum for the meeting of the MPC is four members. Each member of the MPC has one vote, and in the event of an equality of votes, the Governor has a second or casting vote.

4. (A) was the first country to embrace inflation targeting, and since then, a large number of nations, including India, have chosen it as their primary monetary policy tool.
Identify the country (A).
(1) Australia
(2) USA
(3) Germany
(4) Japan
(5) None of these

Ans: 5
Solution:
New Zealand was the first country to embrace inflation targeting, and since then, a large number of nations, including India, have chosen it as their primary monetary policy tool.

5. Finance minister Nirmala Sitharaman will hold bilateral discussions with representatives of over 10 countries on the sidelines of the two-day G20 finance and central bank governors meeting in Bengaluru.
Which of the following is not a member of G 20?
(1) Russia
(2) USA
(3) Indonesia
(4) South Africa
(5) None of these

Ans: 5
Solution:
The G20 comprises Argentina, Australia, Brazil, Canada, China, EU, France, Germany, India, Indonesia, Italy, Japan, Mexico, Russia, Saudi Arabia, South Africa, South Korea, Turkey, UK, and USA. The G20 Countries together represent around 90% of global GDP, 80% of global trade, and two-thirds of the world’s population.

Mint StatiCA MCQs – Feb 22, 2023

1. The total road network in India, _______largest in the world, spans over six million km. This comprises all categories — national and state highways as well as urban and rural roads.
(1) 2nd
(2) 3rd
(3) 4th
(4) 5th
(5) None of these

Ans: 1
Solution:
The total road network in India, second largest in the world, spans over six million km. This comprises all categories — national and state highways as well as urban and rural roads. Construction of national highways (NHs) has risen consistently. There are 663 NHs and their total length has increased from about 91,287 km in March 2014 to about 144,983 km at present. Bharatmala Pariyojana, India’s largest infrastructure programme, envisioned in 2017, aims to develop 34,800 km of NH corridors at an estimated cost of ₹5.35 trillion. As of now, 11,789 km has been completed in the project.

2. The government is poised to invite bids for the auction of the newly discovered lithium reserves in __ district of Jammu and Kashmir as early as the June quarter, a move that will give India access to the strategic mineral that helps power everything from mobile phones to electric vehicles.
(1) Bandipora
(2) Samba
(3) Doda
(4) Reasi
(5) None of these

Ans: 4
Solution:
The Geological Survey of India estimates it has discovered 5.9 million tons of lithium resources in the mountainous Salal-Haimana area of the Reasi district in the union territory of Jammu and Kashmir.

3. India and (A) linked their quick payment systems, Unified Payments Interface (UPI) and PayNow, easing the process of fund transfers between individuals in the two Asian nations.
Identify (A).
(1) Thailand
(2) Bhutan
(3) Indonesia
(4) China
(5) None of these

Ans: 5
Solution:
India and Singapore linked their quick payment systems, Unified Payments Interface (UPI) and PayNow, easing the process of fund transfers between individuals in the two Asian nations.

4. Headquarters of ICAR- IIWBR – Indian Institute of Wheat and Barley Research is located at __.
(1) Lucknow
(2) Varanasi
(3) Karnal
(4) Chandigarh
(5) None of these

Ans: 3
Solution:
In India, the wheat research started in an organized manner exactly hundred years ago during British period after joining of Sir Howards as the Imperial Botanist at Pusa (Bihar) in 1905. Later on with the establishment of the Indian Council of Agricultural Research in 1929, it became the main funding agency and promoter of wheat research in India and the Indian Agricultural Research Institute (IARI) played an important role for the wheat Research related activities. An important milestone in this process was the establishment of the All India Coordinated Wheat Improvement Project (AICWIP) in 1965 by the ICAR. The AICWIP was elevated to the status of the Directorate of Wheat Research in 1978 and in 1990 it moved from IARI, New Delhi, to its present location at Karnal, 130 km north of Delhi along with two regional stations located at Flowerdale, Shimla and Dalang Maidan (Lahaul valley). In 2014, it became an institute, ICAR-Indian Institute of Wheat and Barley Research. By doing so, the ICAR formalised the estab­lishment of a distinct institutional identity for the second most important cereal crop of the country.

5. The government has proposed that from FY24, maturity proceeds from life insurance policies with aggregate annual premium of over ₹5 lakh will be taxed. This excludes ULIPs.
What is ‘L’ in ‘ULIP’?
(1) Leverage
(2) Litigation
(3) Linked
(4) Listed
(5) None of these

Ans: 3
Solution:
The government has proposed that from FY24, maturity proceeds from life insurance policies with aggregate annual premium of over ₹5 lakh will be taxed. This excludes unit linked insurance plans (ULIPs).
A unit-linked insurance plan is a product offered by insurance companies that, unlike a pure insurance policy, gives investors both insurance and investment under a single integrated plan.

Mint StatiCA MCQs – Feb 17, 2023

1. The Hindenburg impact led to the National Stock Exchange and BSE, in consultation with Sebi, placing additional surveillance measures on certain Adani stocks. BSE is India’s first and largest stock market and was founded in (A) as the Native Equity and Stockbroker Association. It is headquartered in _(B)_.
Identify (A) and (B) respectively.
(1) 1875; Mumbai
(2) 1925; Kolkata
(3) 1925; Mumbai
(4) 1875; Kolkata
(5) None of these

Ans: 1
Solution:
The Bombay Stock Exchange (BSE) is India’s first and largest stock market and was founded in 1875 as the Native Equity and Stockbroker Association. It is headquartered in Mumbai, India.

2. A unicorn is any privately owned firm with a market capitalization of more than _.
(1) Rs. 1 billion
(2) USD 1 billion
(3) Rs. 1 million
(4) USD 1 million
(5) None of these

Ans: 2
Solution:
A unicorn is any privately owned firm with a market capitalization of more than USD 1 billion.

3. Which of the following is/are true about Wheat?
A. Wheat is a Kharif Crop grown between September and December and harvested between February and May.
B. The wheat crop has wide adaptability. It can be grown not only in the tropical and sub-tropical zones but also in the temperate zone and the cold tracts of the far north, beyond even the 60 degrees north latitude.
C. Durum wheat, often known as pasta wheat or macaroni wheat is also one of the best quality wheat variety in India.
Choose the correct option:
(1) Only A and B
(2) Only B and C
(3) Only A and C
(4) Only B
(5) A, B and C

Ans: 2
Solution:
Wheat is a Rabi Crop grown between September and December and harvested between February and May.
The wheat crop has wide adaptability. It can be grown not only in the tropical and sub-tropical zones but also in the temperate zone and the cold tracts of the far north, beyond even the 60 degrees north latitude.
Durum wheat, often known as pasta wheat or macaroni wheat is also one of the best quality wheat variety in India.

4. The __ shows redistribution of income between resident and non-residents, i.e., when resources for current purposes are provided without economic value being exchanged in return (transfers).
(1) BoP Account
(2) Secondary Account
(3) Capital Account
(4) Transfer Account
(5) Primary Account

Ans: 2
Solution:
The secondary income account shows redistribution of income between resident and non-residents, i.e., when resources for current purposes are provided without economic value being exchanged in return (transfers).

5. Japan’s trade deficit surged to a record in January, as one-off factors including the lunar new year holidays dragged on exports amid a backdrop of a slowing global economy. The net balance in the __ will show the monetary value of the difference in exports (X) and imports (M) of a country.
(1) Balance of Payments
(2) Balance of Exports
(3) Balance of Imports
(4) Balance of Trade
(5) None of these

Ans: 4
Solution:
Balance of Trade (BOT): The BOT deals only with exports and imports of merchandise (or visible items). The net balance in the BOT will show the monetary value of the difference in exports (X) and imports (M) of a country.

Mint StatiCA MCQs – Feb 16, 2023

1. The union cabinet took a raft of decisions to strengthen border security, including approving (A) for the Vibrant Villages Programme covering (B) states and a union territory for FY2022-23 to 2025-26.
Identify (A) and (B) respectively.
(1) ₹4,800 crore; 4
(2) ₹2,800 crore; 4
(3) ₹4,800 crore; 6
(4) ₹2,800 crore; 6
(5) None of these

Ans: 1
Solution:
The Union Cabinet, chaired by the Hon’ble Prime Minister Shri Narendra Modi, has approved Centrally Sponsored Scheme- “Vibrant Villages Programme” (VVP) for the Financial Years 2022-23 to 2025-26 with financial allocation of Rs. 4800 Crore.
The scheme will provide funds for development of essential infrastructure and creation of livelihood opportunities in 19 Districts and 46 Border blocks 4 states and 1 UT along the northern land border of the country which will help in achieving inclusive growth and retaining the populatiion in the border areas. In the first phase 663 Villages will be taken up in the programme.

2. India’s merchandise exports declined for the second straight month in January by around 6%, the third decline in the current fiscal, led by a demand slowdown across major markets including the US and Europe. Which of the following best describes the term ‘import cover’, sometimes seen in the news?
(1) It is the ratio of value of imports to the Gross Domestic Product of a country
(2) It is the total value of imports of a country in a year
(3) It is the ratio between the value of exports and that of imports between two countries
(4) It is the number of months of imports that could be paid for by a country’s international reserves
(5) None of these

Ans: 4
Solution:
Import Cover measures the number of months of imports that can be covered with foreign exchange reserves available with the central bank of the country.

3. A _ is an account that a domestic bank holds for a foreign bank in the domestic bank’s currency.
(1) Nostro Account
(2) LORO Account
(3) Vostro Account
(4) LIFO Account
(5) None of these

Ans: 3
Solution:
A Nostro Account is said to be a record of deposits held by a bank with a foreign bank in the currency of the country holding the funds.
A Vostro Account is one that is managed by a correspondent bank on another bank’s behalf.
The difference between the two is that it is described from the point of view of the two different banks i.e., depositor and holder.

4. Nifty oscillated 180 points from the lows of 17,853.80 to 18034.10, but at the end of the day the Nifty settled at over 18,000, up 0.48%, or with gains of 86 points. The NIFTY is a benchmark Indian stock market index that represents the weighted average of (A)__of the largest Indian companies listed on the (B).
Identify (A) and (B) respectively.
(1) 50; NSE
(2) 30; NSE
(3) 50; BSE
(4) 30; BSE
(5) None of these

Ans: 1
Solution:
The NIFTY 50 is a benchmark Indian stock market index that represents the weighted average of 50 of the largest Indian companies listed on the National Stock Exchange.

5. The Reserve Bank of India has granted “in-principle approval” to 32 entities to function as online payment aggregators, according to a list released by the central bank.
Which of the following statements about the Payment Aggregators is/are true?
(1) A Payment Aggregator allows one to have multitudes of options for payment.
(2) Payment Aggregators facilitate e-commerce sites and merchants in accepting payment instruments from the customers for completion of their payment obligations without the need for merchants to create a separate payment integration system of their own.
(3) Payment Aggregators are entities that provide technology infrastructure to route and facilitate processing of an online payment transaction without any involvement in handling of funds.
(4) A Payment Aggreagtor allows the merchants to deal in a specific payment option put on the portal only.
(5) Both 1 and 2

Ans: 5
Solution:
Payment Aggregators facilitate e-commerce sites and merchants in accepting payment instruments from the customers for completion of their payment obligations without the need for merchants to create a separate payment integration system of their own. Example: Billdesk.
Payment Gateways are entities that provide technology infrastructure to route and facilitate processing of an online payment transaction without any involvement in handling of funds. PGs in India mainly include banks.
A Payment Gateway allows the merchants to deal in a specific payment option put on the portal, whereas a Payment Aggregator allows one to have multitudes of options for payment. Thus, a Payment Aggregator covers a payment gateway in its ambit.

Mint StatiCA MCQs – Feb 15, 2023

1. The government is looking at buybacks and OFS of central public sector enterprises as potential options to meet the current year’s disinvestment target as uncertain market conditions and legal hurdles pose challenges to ongoing privatization transactions.
What is ‘O’ in “OFS”?
(1) Original
(2) Offer
(3) Ordinary
(4) Ordinance
(5) None of these

Ans: 2
Solution:
Under the offer for sale method, securities are not issued directly to the public but are offered for sale through intermediaries like issuing houses or stock brokers.
In this case, a company sells securities enbloc at an agreed price to brokers who, in turn, resell them to the investing public.

2. Which of the following statements is/are true about Wholesale Price Index (WPI)?
A. It measures the changes in the prices of goods sold and traded in bulk by wholesale businesses to other businesses.
B. Published by the Office of Economic Adviser, Ministry of Commerce and Industry.
C. The base year of All-India WPI has been revised from 2004-05 to 2011-12 in 2017.
Choose the correct option:
(1) Only A and B
(2) Only B and C
(3) Only A and C
(4) Only B
(5) A, B and C

Ans: 5
Solution:
Wholesale Price Index
It measures the changes in the prices of goods sold and traded in bulk by wholesale businesses to other businesses.
Published by the Office of Economic Adviser, Ministry of Commerce and Industry.
Major criticism for this index is that the general public does not buy products at wholesale price.
The base year of All-India WPI has been revised from 2004-05 to 2011-12 in 2017.

3. National Highways Infrastructure Trust (NHIT), an autonomous government body sponsored by highway developer NHAI, plans to raise around ₹7,000 crores through a mix of debt and equity including non-convertible debentures (NCD). The minimum investment amount in an InvIT Initial Public Offering (IPO) is Rs __, therefore, InvITs are suitable for high-net-worth individuals, and institutional and non-institutional investors.
(1) Rs. 5 lakh
(2) Rs. 10 lakh
(3) Rs. 20 lakh
(4) Rs. 40 lakh
(5) None of these

Ans: 2
Solution:
The minimum investment amount in an InvIT Initial Public Offering (IPO) is Rs 10 lakh, therefore, InvITs are suitable for high networth individuals, institutional and non-institutional investors.

4. Which of the following statements is/are true about NAIRU?
A. NAIRU means Non-accelerating inflation rate of unemployment.
B. When unemployment is equal to NAIRU there will be stability in the rate of inflation.
C. When unemployment departs from NAIRU, there is acceleration or deceleration in inflation rate.
(1) Only A and B
(2) Only B and C
(3) Only A and C
(4) Only B
(5) A, B and C

Ans: 5
Solution:
NAIRU means non-accelerating inflation rate of unemployment.
When unemployment is equal to NAIRU there will be stability in the rate of inflation.
When unemployment departs from NAIRU, there is acceleration or deceleration in inflation rate.

5. _(A) shows that there is an inverse relationship between the rate of unemployment and the rate of increase in nominal wages.
Identify (A).
(1) Engel Curve
(2) Kuznets Curve
(3) Laffer Curve
(4) Philips Curve
(5) None of these

Ans: 4
Solution:
Phillips Curve shows that there is an inverse relationship between the rate of unemployment and the rate of increase in nominal wages. A.W. Phillips has come up with this concept that there is a trade-off between wage inflation and unemployment. In other words, a lower rate of unemployment is associated with higher wage rate or inflation and vice versa.

Mint StatiCA MCQs – Feb 14, 2023

1. Stocks such as Adani Enterprises, Adani Total Gas, Adani Green, and Adani Transmission are under NSE’s ASM framework, which clamps extra margins for day trades.
What is ‘S’ in the “ASM” framework?
(1) Standard
(2) Supervision
(3) Surveillance
(4) Substitution
(5) None of these

Ans: 3
Solution:
Stocks such as Adani Enterprises, Adani Total Gas, Adani Green and Adani Transmission are under NSE’s additional surveillance measures (ASM) framework, which clamps extra margins for day trades.
NSE’s ASM framework, under which Adani Enterprises, Adani Ports and Ambuja Cements have been placed, is an additional surveillance initiative by the Exchanges and the Securities and Exchange Board of India (SEBI) to enhance ‘market integrity and safeguard the interest of investors.

2. Retail inflation in January accelerated to the fastest in three months, exceeding the Reserve Bank of India’s upper tolerance band of ________defying hopes that price pressures would decelerate.
(1) 4 %
(2) 5 %
(3) 6 %
(4) 7 %
(5) 8 %

Ans: 3
Solution:
Retail inflation in January accelerated to the fastest in three months, exceeding the Reserve Bank of India’s upper tolerance band of 6%, defying hopes that price pressures would decelerate.
Under Section 45ZA, the Central Government, in consultation with the RBI, determines the inflation target in terms of the Consumer Price Index (CPI), once in five years and notifies it in the Official Gazette. Accordingly, on August 5, 2016, the Central Government notified in the Official Gazette 4 per cent Consumer Price Index (CPI) inflation as the target for the period from August 5, 2016 to March 31, 2021 with the upper tolerance limit of 6 per cent and the lower tolerance limit of 2 per cent. On March 31, 2021, the Central Government retained the inflation target and the tolerance band for the next 5-year period – April 1, 2021 to March 31, 2026.

3. Once in every _, the Reserve Bank publishes the Monetary Policy Report.
(1) 1 month
(2) 6 months
(3) 1 Year
(4) 2 Years
(5) None of these

Ans: 2
Solution:
Once in every six months, the Reserve Bank publishes the Monetary Policy Report containing the following elements:
Explanation of inflation dynamics in the last six months and the near term inflation outlook;
Projections of inflation and growth and the balance of risks;
An assessment of the state of the economy, covering the real economy, financial markets and stability, fiscal situation, and the external sector, which may entail a bearing on monetary policy decisions;
An updated review of the operating procedure of monetary policy; and
An assessment of projection performance.

4. The health ministry proposes to include video streaming platforms under the Cigarettes and Other Tobacco Products Act (COTPA), _(A), to ensure they carry public health warnings related to smoking and alcohol use, similar to those shown on TV programmes and movies.
Identify (A).
(1) 1999
(2) 2003
(3) 2005
(4) 2009
(5) None of these

Ans: 2
Solution:
The health ministry proposes to include video streaming platforms under the Cigarettes and Other Tobacco Products Act (COTPA), 2003, to ensure they carry public health warnings related to smoking and alcohol use, similar to those shown on TV programmes and movies.

5. The ministry of corporate affairs will remove over 38,000 companies from official records for not meeting the statutory filing obligations, to keep up the pressure on the corporate sector so that they do not default on filing their annual returns.
Who is the present minister of corporate affairs?
(1) Amit Shah
(2) Piyush Goyal
(3) Smriti Irani
(4) Kiren Rijiju
(5) None of these

Ans: 5
Solution:
The Ministry of Corporate Affairs is an Indian government ministry primarily concerned with administration of the Companies Act 2013, the Companies Act 1956, the Limited Liability Partnership Act, 2008, and the Insolvency and Bankruptcy Code, 2016.
The current minister is Nirmala Sitaraman.

Mint StatiCA MCQs – Feb 13, 2023

1. Revealing remedial measures meant to contain inflation and shared with the government after it failed to meet the flexible target last year, could lead to market disruptions and cause financial market volatility, the Reserve Bank of India (RBI) said in response to an appeal under the Right to Information (RTI) Act. The right to information is a fundamental right under Article 19 (1) of the Indian Constitution. In _(A), in the Raj Narain vs the State of Uttar Pradesh case, the Supreme Court ruled that Right to information will be treated as a fundamental right under article 19. The Supreme Court held that in Indian democracy, people are the masters, and they have the right to know about the working of the government.
Identify (A) in the above excerpt.
(1) 1952
(2) 1964
(3) 1976
(4) 1988
(5) None of these

Ans: 3
Solution:
The right to information is a fundamental right under Article 19 (1) of the Indian Constitution. In 1976, in the Raj Narain vs the State of Uttar Pradesh case, the Supreme Court ruled that Right to information will be treated as a fundamental right under article 19. The Supreme Court held that in Indian democracy, people are the masters, and they have the right to know about the working of the government.
Thus, the government enacted the Right to Information act in 2005 which provides machinery for exercising this fundamental right.

2. Under __ of the amended (in 2016) RBI Act, 1934, the central government is empowered to constitute a six-member Monetary Policy Committee (MPC).
(1) Section 28
(2) Section 45 ZA
(3) Section 56
(4) Section 45 ZB
(5) None of these

Ans: 4
Solution:
Under Section 45ZB of the amended (in 2016) RBI Act, 1934, the central government is empowered to constitute a six-member Monetary Policy Committee (MPC).

3. Foreign portfolio investors (FPIs) doubled down on their bearish stance since the Adani Group crisis surfaced, selling not only shares but also the highest number of Nifty and Bank Nifty (index) futures contracts in four months.
Which of the following statements is/are true about Futures?
A. Future trading involves an obligation on the buyer and seller to follow through on their agreement and complete the transaction on a defined date and price.
B. Futures contracts can be freely traded on exchanges since they are not standardized in terms of expiration dates and contract sizes.
C. RBI regulates futures trading in India to guarantee that it runs smoothly.
(1) Only A and B
(2) Only B and C
(3) Only A and C
(4) Only A
(5) A, B and C

Ans: 4
Solution:
Future trading involves an obligation on the buyer and seller to follow through on their agreement and complete the transaction on a defined date and price.
Futures contracts can be freely traded on exchanges since they are standardized in terms of expiration dates and contract sizes.
SEBI regulates futures trading in India to guarantee that it runs smoothly.

4. __ happens when the seller sells the transaction at a fixed price at a later date.
(1) Long Call
(2) Short Call
(3) Short Put
(4) Long Put
(5) None of these

Ans: 2
Solution:
Long call
The buyer purchases a transaction at a previously agreed-upon price at a later date.
Short call
When the seller sells the transaction at a fixed price at a later date.
Short put
The seller is obligated to sell in this method, but the buyer has the opportunity to exercise the option contract. When the stock’s value has risen over the predetermined price, the buyer usually does this.
Long put
The buyer can purchase the transaction for a fixed price, which the seller must accept.

5. The Reserve Bank of India established the Committee on Capital Account Convertibility (CAC) or (A)_ to propose a roadmap for full convertibility of the rupee on the capital account. In May (B)_, the committee submitted its report.
Identify (A) and (B) respectively in the above excerpt.
(1) Tarapore Committee; 1997
(2) Bimal Jalan Committee; 1997
(3) Tarapore Committee; 2000
(4) Khan Committee; 1997
(5) None of these

Ans: 1
Solution:
The Reserve Bank of India established the Committee on Capital Account Convertibility (CAC) or Tarapore Committee to propose a roadmap for full convertibility of the rupee on the capital account. In May 1997, the committee submitted its report.

Mint StatiCA MCQs – Feb 10, 2023

1. The Adani group stocks erased nearly all their two-day gains after MSCI flagged the uncertain “investor characteristics” of some investors in group stocks which could impact how the index provider calculates the free float in them and potentially lead to weighting cuts and foreign investor outflows from funds that track the indices. MSCI is headquartered at _.
(1) New York, USA
(2) Frankfurt, Germany
(3) London, UK
(4) Brussels, Belgium
(5) None of these

Ans: 1
Solution:
MSCI is a leading provider of critical decision support tools and services for the global investment community. MSCI has its headquarters in New York, USA.

2. “Rapid Financing Instrument” and “Rapid Credit Facility” are related to the provisions of lending by which of the following:
(1) Asian Development Bank
(2) International Monetary Fund
(3) United Nations Environment Programme Finance Initiative
(4) World Bank
(5) None of these

Ans: 2
Solution:
“Rapid Financing Instrument” and “Rapid Credit Facility” are related to the provisions of lending by the IMF.
The Rapid Financing Instrument (RFI) provides rapid financial assistance to all member countries facing an urgent balance of payments need. The RFI was created as part of a broader reform to make the IMF’s financial support more flexible to address the diverse needs of member countries. The RFI has replaced the IMF’s emergency assistance policy and can be used in a wide range of circumstances.
The Rapid Credit Facility (RCF) provides rapid concessional financial assistance to low-income countries (LICs) facing an urgent balance of payments need with no ex-post conditionality where a full-fledged economic program is neither necessary nor feasible. The RCF was created under the Poverty Reduction and Growth Trust (PRGT) as part of a broader reform to make the Fund’s financial support more flexible and better tailored to the diverse needs of LICs, including in times of crisis.

3. _ are direct investments in shares that possess high risk in long term investments but obtain an optimum return in the long run.
(1) Hybrid Mutual Fund
(2) Debt Mutual Fund
(3) Equity Mutual Fund
(4) Fixed Income Fund
(5) None of these

Ans: 3
Solution:
Equity Funds
These are direct investments in shares that possess high risk in long term investments but obtain an optimum return in the long run. Experts further divide it into small, mid and large-cap based on a company’s size. Investors with a high-risk appetite usually opt for these types of investments.

4. In India, which one of the following is responsible for maintaining price stability by controlling inflation?
(1) Department of Consumer Affairs
(2) Expenditure Management Commission
(3) Financial Stability and Development Council
(4) Reserve Bank of India
(5) None of these

Ans: 4
Solution:
RBI is responsible for maintaining price stability by controlling inflation.

5. Which of the following best describes the term ‘import cover’, sometimes seen in the news?
(1) It is the ratio of value of imports to the Gross Domestic Product of a country
(2) It is the total value of imports of a country in a year
(3) It is the ratio between the value of exports and that of imports between two countries
(4) It is the number of months of imports that could be paid for by a country’s international reserves
(5) None of these

Ans: 4
Solution:
Import Cover measures the number of months of imports that can be covered with foreign exchange reserves available with the central bank of the country.

Mint StatiCA MCQs – Feb 9, 2023

1. The Reserve Bank of India slowed the pace of interest rate hikes to a quarter-percentage point as inflation moderated, while keeping the option of rate hikes open amid stubborn core inflation. A rapid increase in the rate of inflation is sometimes attributed to the “base effect”. What is “base effect”?
(1) It is the impact of drastic deficiency in supply due to failure of crops
(2) It is the impact of the surge in demand due to rapid economic growth
(3) It is the impact of the price levels of previous year on the calculation of inflation rate
(4) None of the statements (1), (2) and (3) ‘given above is correct in this context
(5) All of the above i.e. (1), (2) and (3)

Ans: 3
Solution:
The base effect relates to inflation in the corresponding period of the previous year, if the inflation rate was too low in the corresponding period of the previous year, even a smaller rise in the Price Index will arithmetically give a high rate of inflation now.

2. Which one of the following statements appropriately describes the “fiscal stimulus”?
(1) It is a massive investment by the Government in manufacturing sector to ensure the supply of goods to meet the demand surge caused by rapid economic growth.
(2) It is an intense affirmative action of the Government to boost economic activity in the country
(3) It is Government’s intensive action on financial institutions to ensure disbursement of loans to agriculture and allied sectors to promote greater food production and contain food inflation
(4) It is an extreme affirmative action by the Government to pursue its policy of financial inclusion
(5) None of these

Ans: 2
Solution:
Fiscal stimulus refers to a set of fiscal policy measures used by the government to stimulate the economy. Fiscal Stimulus involves a conservative approach toward an expansionary fiscal policy that focuses on encouraging private sector spending so as to make up for losses of aggregate demand.

3. In India, which of the following have the highest share in the disbursement of credit to agriculture and allied activities?
(1) Commercial Banks
(2) Cooperative Banks
(3) Regional Rural Banks
(4) Microfinance Institutions
(5) None of these

Ans: 1
Solution:
Commercial banks have the highest share in the disbursement of credit to agriculture and allied activities. The commercial banks disburse around 60% credit followed by cooperative banks around 30% and RRB and others.

4. The lowering of Bank Rate by the Reserve Bank of India leads to:
(1) More liquidity in the market
(2) Less liquidity in the market
(3) No change in the liquidity in the market
(4) Mobilization of more deposits by commercial banks
(5) None of these

Ans: 1
Solution:
The lowering of Bank Rate by the Reserve Bank of India leads to more liquidity in the market.

5. A “closed economy” is an economy in which:
(1) the money supply is fully controlled
(2) deficit financing takes place
(3) only exports take place
(4) neither exports nor imports take place
(5) None of these

Ans: 4
Solution:
A closed economy is an economy that does not participate in international trade, meaning it does not import or export goods and services from another country. A closed economy grows slower than open economies, as all goods must be produced and consumed from within the single economy.

Mint StatiCA MCQs – Feb 8, 2023

1. All individuals who invest up to Rs ________in an issue are classified as retail investors.
(1) 1 Lakh
(2) 2 Lakhs
(3) 3 Lakhs
(4) 5 Lakhs
(5) 10 Lakhs

Ans: 2
Solution:
All individuals who invest up to Rs 2 lakh in an issue are classified as retail investors.
Retail investors investing above Rs 2 lakh are classified as high-net-worth individuals.

2. Among other conditions for an IPO, the company must have net tangible assets of at least Rs (A), and a net worth of Rs _(B) in each of the preceding three full years.
Identify (A) and (B) respectively.
(1) 5 Crores; 2 Crores
(2) 5 Crores; 3 Crores
(3) 3 Crores; 1 Crore
(4) 3 Crores; 2 Crores
(5) None of these

Ans: 3
Solution:
Among other conditions, the company must have net tangible assets of at least Rs 3 crore, and net worth of Rs 1 crore in each of the preceding three full years, and it must have a minimum average pre-tax profit of Rs 15 crore in at least three of the immediately preceding five years.

3. A Disinvestment Commission was established in __ by the government of India, to carefully evaluate the withdrawal of the public sector from non-core, non-strategic areas and assure workers of job security and opportunities for retraining and re-employment. It recommended the sale of equities or the outright sale of several PSEs, including Air India.
(1) 1985
(2) 1996
(3) 1999
(4) 2002
(5) None of these

Ans: 2
Solution:
A Disinvestment Commission was established in 1996 by the government of India, to carefully evaluate the withdrawal of the public sector from non-core, non-strategic areas and assure workers of job security and opportunities for retraining and re-employment. It recommended the sale of equities or outright sale of several PSE’s, including Air India.

4. Hydro power plants of (A) or below are classified as small, and they come under the purview of the ministry of new and renewable energy (MNRE).
Identify (A).
(1) 10 MW
(2) 20 MW
(3) 50 MW
(4) 100 MW
(5) None of these

Ans: 5
Solution:
Hydro power plants of 25 MW or below are classified as small, and they come under the purview of the ministry of new and renewable energy (MNRE).

5. The Central Board of Indirect Taxes and Customs (CBIC) has been planning to clarify the tax implications of certain crypto transactions that do not squarely fit into the concept of sale of goods or service. In India, GST Bill was first introduced in 2014 as The Constitution (__ Amendment) Bill.
(1) 112th
(2) 122nd
(3) 132nd
(4) 138th
(5) None of these

Ans: 2
Solution:
In India, GST Bill was first introduced in 2014 as The Constitution (122nd Amendment) Bill.
This got an approval in 2016 and was renumbered in the statute by Rajya Sabha as The Constitution (101st Amendment) Act, 2016.

Mint StatiCA MCQs – Feb 7, 2023

1. _ provides indication whether a stock at its current market price is expensive or cheap.
(1) Price to Earnings Ratio
(2) Price to Earnings Multiple
(3) Current Ratio
(4) Quick Ratio
(5) Both 1 and 2

Ans: 5
Solution:
Price to Earnings Ratio or Price to Earnings Multiple is the ratio of share price of a stock to its earnings per share (EPS). P/E ratio is one of the most popular valuation metrics of stocks. It provides indication whether a stock at its current market price is expensive or cheap.

2. Which of the following is/are true about treasury bills?
A. Treasury bills or T-bills are short-term debt instruments issued by the Government of India.
B. Treasury Bills are capital market instruments.
C. Presently, the government issues these T-Bills in three tenors, namely, 91-days, 182-days, and 364-days.
Choose the correct option:
(1) Only A and B
(2) Only B and C
(3) Only A and C
(4) Only C
(5) A, B and C

Ans: 3
Solution:
Government of India Treasury Bills (T-Bills): Treasury bills or T-bills are short-term debt instruments issued by the Government of India. So, through these instruments, the government borrows for a short-term period. Presently, the government issues these T-Bills in three tenors, namely, 91-days, 182-days, and 364-days.

3. Indian Government Bond Yields are influenced by which of the following?
A. Actions of the United States Federal Reserve
B. Actions of the Reserve Bank of India
C. Inflation and short-term interest rates
Select the correct answer using the code given below.
(1) A and B only
(2) B only
(3) C only
(4) A, B and C
(5) A only

Ans: 4
Solution:
Fed tapering is the gradual reduction in the bond buying program of the US Federal Reserve. So, any actions of the United States Federal Reserve impact the bond yield in India. Hence, A is correct.
The actions of the RBI plays a crucial role in determining the yield of government bonds. The sovereign yield curve has a special significance for monetary policy in influencing a wide array of interest rates in the economy. Hence, B is correct.
Inflation and short-term interest rates also influence the yield of government bonds. Hence, C is correct.

4. A sovereign green bond is a debt instrument issued by the central or state government to borrow money from investors with the commitment that the mobilised fund will be spent on climate or eco-system related activities. This phenomenon began in _ with the Poland government issuing its first sovereign green bond.
(1) 2010
(2) 2012
(3) 2014
(4) 2016
(5) None of these

Ans: 4
Solution:
A sovereign green bond is a debt instrument issued by the central or state government to borrow money from investors with the commitment that the mobilised fund will be spent on climate or eco-system related activities. This phenomenon began in 2016 with the Poland government issuing its first sovereign green bond.

5. Benchmark SENSEX dropped 334 points due to intense selling pressure in metal and power stocks as FII outflows dampened investor sentiment. SENSEX is the benchmark index of the (A). The SENSEX index is made up of (B) of the (A)‘s largest and most actively traded stocks, and it serves as a gauge for the Indian economy.
Identify (A) and (B) respectively.
(1) NSE; 30
(2) BSE; 30
(3) NSE; 50
(4) BSE; 50
(5) None of these

Ans: 2
Solution:
SENSEX is the benchmark index of the Bombay Stock Exchange (BSE) in India. The SENSEX index is made up of 30 of the BSE’s largest and most actively traded stocks, and it serves as a gauge for the Indian economy.

Mint StatiCA MCQs – Feb 6, 2023

1. A Follow-on Public Offering (FPO) is the issuance of shares to investors by a company listed on a stock exchange. FPOs are also known as (A).
Identify (A).
(1) Primary Offerings
(2) Offer for Sale
(3) Secondary Offerings
(4) Greenshoe Sale
(5) None of these

Ans: 3
Solution:
A Follow-on Public Offering (FPO) is the issuance of shares to investors by a company listed on a stock exchange.
FPOs are also known as secondary offerings.
Companies may use an FPO to reduce debt or raise more capital for expansion.
They typically occur after the company has completed an initial public offering (IPO) to make its shares available to the public.

2. India’s planned strategic gas reserve may come up as a joint venture of public sector companies or as a public-private partnership (PPP). _ is a PPP approach in which the government gives a concession to a private entity to build a facility (and possibly design it as well), own the facility, lease the facility to the public sector and then at the end of the lease period transfer the ownership of the facility to the government.
(1) BOO
(2) BOOT
(3) DBFO
(4) BOLT
(5) None of these

Ans: 4
Solution:
BOLT
In this approach, the government gives a concession to a private entity to build a facility (and possibly design it as well), own the facility, lease the facility to the public sector and then at the end of the lease period transfer the ownership of the facility to the government.

3. (A) is the apex national authority under the (B) for administering all trade remedial measures including anti-dumping, countervailing duties and safeguard measures.
Identify (A) and (B) respectively.
(1) Directorate General of Trade Remedies; Ministry of Finance
(2) India Trade Promotion Organisation; Ministry of Commerce and Industry
(3) Directorate General of Trade Remedies; Ministry of Commerce and Industry
(4) India Trade Promotion Organisation; Ministry of Finance
(5) None of these

Ans: 3
Solution:
Directorate General of Trade Remedies
It is the apex national authority under the Ministry of Commerce and Industry for administering all trade remedial measures including anti-dumping, countervailing duties and safeguard measures.
It provides trade defence support to the domestic industry and exporters in dealing with increasing instances of trade remedy investigations instituted against them by other countries.

4. Bank finance to start-ups up to Rs. __ crores, loans to farmers for installation of solar power plants for solarisation of grid-connected agriculture pumps, and loans for setting up Compressed Biogas plants have been included as fresh categories eligible for finance under the priority sector.
(1) 10
(2) 50
(3) 100
(4) 200
(5) None of these

Ans: 2
Solution:
Bank finance to start-ups up to Rs. 50 crores, loans to farmers for installation of solar power plants for solarisation of grid connected agriculture pumps and loans for setting up Compressed Biogas plants have been included as fresh categories eligible for finance under priority sector.

5. _(A) shows that there is an inverse relationship between the rate of unemployment and the rate of increase in nominal wages.
Identify (A).
(1) Engel Curve
(2) Kuznets Curve
(3) Laffer Curve
(4) Philips Curve
(5) None of these

Ans: 4
Solution:
Phillips Curve shows that there is an inverse relationship between the rate of unemployment and the rate of increase in nominal wages. A.W. Phillips has come up with the concept that there is a trade-off between wage inflation and unemployment. In other words, a lower rate of unemployment is associated with higher wage rate or inflation and vice versa.

Mint StatiCA MCQs – Feb 3, 2023

1. Before SEBI came into existence, the Controller of Capital Issues was the regulatory authority; it derived authority from the Capital Issues (Control) Act, of _.
(1) 1947
(2) 1955
(3) 1962
(4) 1967
(5) None of these

Ans: 1
Solution:
Before SEBI came into existence, Controller of Capital Issues was the regulatory authority; it derived authority from the Capital Issues (Control) Act, 1947.

2. Which of the following statements is/are true about the Securities Appellate Tribunal (SAT)?
A. SAT is a statutory body established under the provisions of the Securities and Exchange Board of India Act, 1992.
B. Consequent to government notification dated 27th May, 2014; SAT hears and disposes of appeals against orders passed by the Pension Fund Regulatory and Development Authority (PFRDA) under the PFRDA Act, 2013.
C. It has the same powers as vested in a civil court. Further, if any person feels aggrieved by SAT’s decision or order can appeal to the Supreme Court.
(1) Only A and B
(2) Only B and C
(3) Only A and C
(4) Only B
(5) A, B and C

Ans: 5
Solution:
SAT is a statutory body established under the provisions of the Securities and Exchange Board of India Act, 1992.
Consequent to government notification dated 27th May, 2014; SAT hears and disposes of appeals against orders passed by the Pension Fund Regulatory and Development Authority (PFRDA) under the PFRDA Act, 2013.
It has the same powers as vested in a civil court. Further, if any person feels aggrieved by SAT’s decision or order can appeal to the Supreme Court.

3. The National Company Law Tribunal (NCLT) barred lenders of bankrupt Reliance Capital (RCap) from conducting a second auction of its assets, terming it an “illegal mechanism” to circumvent rules of India’s bankruptcy code.
Which section of the Companies Act, 2013 deals with the constitution of NCLT?
(1) Section 334
(2) Section 373
(3) Section 408
(4) Section 426
(5) None of these

Ans: 3
Solution:
The National Company Law Tribunal or NCLT is a quasi-judicial body in India adjudicating issues concerning companies in the country. It was formed on June 1, 2016, as per the provisions of the Companies Act 2013 (Section 408) by the Indian government.

4. The CPI uses a base year set at __.
(1) 2001-02
(2) 2004-05
(3) 2008-09
(4) 2011-12
(5) None of these

Ans: 4
Solution:
The CPI uses a base year set at 2011-2012.

5. Decrease in the general price level of goods and services is called _.
(1) Stagflation
(2) Deflation
(3) Disinflation
(4) Hyperinflation
(5) None of these

Ans: 2
Solution:
Deflation
Decrease in the general price level of goods and services.

Mint StatiCA MCQs – Feb 2, 2023

1. Government receipt and expenditure are the two components of a budget. In terms of the magnitudes of receipts and expenditures. When the government expenditure is exactly equal to its receipts, the government has _(A).
Identify (A).
(1) Positive Budget
(2) Surplus Budget
(3) Deficit Budget
(4) Negative Budget
(5) None of these

Ans: 5
Solution:
Government receipt and expenditure are the two components of a budget. In terms of the magnitudes of receipts and expenditures. We may have a balanced budget, deficit budget, and surplus budget. When the government expenditure is exactly equal to its receipts, the government has a balanced budget. When the government expenditure exceeds its receipts, it is a deficit budget.
When the government revenue is greater than its expenditure, the government runs a budget surplus.
Thus:
Balanced budget — Total Budgeted Receipt = Total Budgeted Expenditure
Deficit budget — Total Budgeted Receipts < Total Budgeted Expenditure Surplus budget — Total Budgeted Receipts > Total Budgeted Expenditure

2. The Budget Division of the _ is the nodal body responsible for preparing the Budget.
(1) Department of Revenue, Ministry of Finance
(2) Department of Expenditure, Ministry of Finance
(3) Department of Commerce, Ministry of Commerce
(4) Office of the CEA, Ministry of Commerce
(5) Department of Economic Affairs, Ministry of Finance

Ans: 5
Solution:
The Budget Division of the Department of Economic Affairs in the Finance Ministry is the nodal body responsible for preparing the Budget.

3. __ is incurred with the purpose of increasing assets of a durable nature or of reducing recurring liabilities.
(1) Capital Receipts
(2) Revenue Expenditure
(3) Capital Expenditure
(4) Revenue Receipts
(5) None of these

Ans: 3
Solution:
Capital expenditure is incurred with the purpose of increasing assets of a durable nature or of reducing recurring liabilities.
Consider the expenditure incurred for constructing new schools or new hospitals. All these are classified as capital expenditures as they lead to the creation of new assets.

4. __ is the difference between fiscal deficit and interest payments.
(1) Secondary Deficit
(2) Revenue Deficit
(3) Effective Fiscal Deficit
(4) Primary Deficit
(5) None of these

Ans: 4
Solution:
Primary deficit is the difference between fiscal deficit and interest payments.

5. __ is derived by deducting capital expenditure from fiscal deficits.
(1) Secondary Deficit
(2) Revenue Deficit
(3) Effective Fiscal Deficit
(4) Primary Deficit
(5) None of these

Ans: 2
Solution:
Revenue deficit is derived by deducting capital expenditure from fiscal deficits.

Mint StatiCA MCQs – Feb 1, 2023

1. __ involves a one-shot injection of government expenditure into a depressed economy with the aim of boosting business confidence and encouraging larger private investment.
(1) Pent-up Demand
(2) Compensatory Spending
(3) Pump Priming
(4) Public Spending
(5) None of these

Ans: 3
Solution:
Pump priming involves a one-shot injection of government expenditure into a depressed economy with the aim of boosting business confidence and encouraging larger private investment. It is a temporary fiscal stimulus in order to set off the multiplier process.

2. Fiscal Responsibility and Budget Management (FRBM) Act is the statute to induce discipline and restrictions on expenditure and debt-related things was introduced and was passed by the Parliament in __.
(1) 2000
(2) 2003
(3) 2006
(4) 2008
(5) 2010

Ans: 2
Solution:
Fiscal Responsibility and Budget Management (FRBM) Act is the statute to induce discipline and restrictions on expenditure and debt-related things was introduced and was passed by the Parliament in 2003.

3. Under Article __ of the Constitution, the President of India is required to constitute a Finance Commission at an interval of five years or earlier.
(1) 110
(2) 183
(3) 210
(4) 280
(5) None of these

Ans: 4
Solution:
Under Article 280 of the Constitution, the President of India is required to constitute a Finance Commission at an interval of five years or earlier.

4. _(A) is a tax on gains made on the domestic stock exchange on securities such as equities, options, and futures.
Identify (A).
(1) Commodities Transaction Tax
(2) Securities Transaction Tax
(3) Wealth Tax
(4) Capital Gains Tax
(5) None of these

Ans: 2
Solution:
Securities Transaction Tax
The securities transaction tax is a tax on gains made on the domestic stock exchange on securities such as equities, options, and futures.
It is a direct tax levied and collected by the central government.
P. Chidambaram, the former Finance Minister, proposed the Securities Transaction Tax (STT) in 2004.

5. Central and state governments collected ₹1.56 trillion in goods and services tax (GST) in January, marking the second-highest collection ever and an improvement of 24% from a year ago, the finance ministry said.
Who is the chairman of the GST Council?
(1) Finance Secretary
(2) Union Finance Minister
(3) Revenue Secretary
(4) Prime Minister
(5) None of these

Ans: 2
Solution:
Union Finance Minister is the chairman of the GST Council.

Mint StatiCA MCQs – Jan 31, 2023

1. The term __ refers to an emergency-use regulatory measure that temporarily halts trading on an exchange.
(1) Circuit Trigger
(2) Trading Trigger
(3) Trading Breaker
(4) Circuit Breaker
(5) None of these

Ans: 4
Solution:
Circuit breakers are temporary measures that halt trading to curb panic-selling on stock exchanges.

2. The International Monetary Fund (IMF) projected India to be the fastest-growing major economy in FY24, retaining the forecast at 6.1% in its latest _(A), citing “resilient” domestic demand despite a challenging external environment.
Identify (A).
(1) Global Economic Outlook
(2) World Economic Outlook
(3) Global Economic Prospects
(4) World Economic Prospects
(5) None of these

Ans: 2
Solution:
The International Monetary Fund (IMF) projected India to be the fastest-growing major economy in FY24, retaining the forecast at 6.1% in its latest World Economic Outlook, citing “resilient” domestic demand despite a challenging external environment.

3. The government’s move to amend the Insolvency and Bankruptcy Code (IBC) is set to clear the air on the tax dues of failed businesses and aid their bankruptcy resolution. IBC is a reform enacted in __. It amalgamates various laws relating to the insolvency resolution of business firms.
(1) 2014
(2) 2015
(3) 2016
(4) 2017
(5) None of these

Ans: 3
Solution:
Insolvency and Bankruptcy Code
It is a reform enacted in 2016. It amalgamates various laws relating to the insolvency resolution of business firms.
It lays down clear-cut and faster insolvency proceedings to help creditors, such as banks, recover dues and prevent bad loans, a key drag on the economy.

4. The National Food Security Act, 2013 covers (A) percent of the urban population and (B) percent of the rural population.
Identify (A) and (B) respectively.
(1) 75; 75
(2) 50; 75
(3) 75; 50
(4) 50; 50
(5) None of these

Ans: 2
Solution:
The NFSA covers 50 percent of the urban population and 75 percent of the rural population.

5. The Companies Act, 2013 provides for the constitution of the National Company Law Tribunal (NCLT) & National Company Law Tribunal and Appellate Tribunal (NCLAT). NCLT is set up to bring all lawsuits pertaining to companies under one body. It has replaced the Company Law Board (CLB), BIFR and the Appellate Authority for Industrial and Financial Reconstruction and will have judicial and technical members.
What is ‘I’ in BIFR?
(1) Institutional
(2) Infrastructure
(3) Industrial
(4) Interest
(5) None of these

Ans: 3
Solution:
The Companies Act, 2013 provides for the constitution of the National Company Law Tribunal (NCLT) & National Company Law Tribunal and Appellate Tribunal (NCLAT). NCLT is set up to bring all lawsuits pertaining to companies under one body. It has replaced the Company Law Board (CLB), the Board for Industrial and Financial Reconstruction (BIFR) and the Appellate Authority for Industrial and Financial Reconstruction and will have judicial and technical members.

Mint StatiCA MCQs – Jan 30, 2023

1. The government plans to amend the GST law to clarify the Integrated Goods and Services Tax (IGST) liability for offshore digital businesses offering services such as online advertising, games, and cloud services to Indian users. In India, GST Bill was first introduced in __(A) as The Constitution (122nd Amendment) Bill.
Identify (A).
(1) 2014
(2) 2015
(3) 2016
(4) 2017
(5) None of these

Ans: 1
Solution:
In India, GST Bill was first introduced in 2014 as The Constitution (122nd Amendment) Bill.

2. Article (A) – GST Council to be formed by the President to administer & govern GST. Its chairman is (B) of India with ministers nominated by the state governments as its members.
Identify (A) and (B) respectively.
(1) 246 A; Union Finance Minister
(2) 279 A; Union Finance Minister
(3) 246 A; Prime Minister
(4) 279 A; Prime Minister
(5) None of these

Ans: 2
Solution:
Article 279A – GST Council to be formed by the President to administer & govern GST. Its Chairman is Union Finance Minister of India with ministers nominated by the state governments as its members.

3. Cotton futures trading, suspended on the Multi Commodity Exchange of India Ltd (MCX) to curb excessive speculation, is set to resume after a five-month gap in relief for farmers reliant on the market for price signals.
Identify the Headquarters of MCX.
(1) Hyderabad
(2) Kolkata
(3) New Delhi
(4) Mumbai
(5) None of these

Ans: 4
Solution:
Multi Commodity Exchange of India Ltd is a commodity exchange based in India. It is under the ownership of Ministry of Finance, Government of India. It was established in 2003 by the Government of India and is currently based in Mumbai, Maharashtra. It is India’s largest commodity derivatives exchange.

4. As of April 2022, India signed _____Free Trade Agreements (FTAs) with its trading partners including major trade agreements like the India-UAE Comprehensive Partnership Agreement (CEPA) and the India-Australia Economic Cooperation and Trade Agreement (IndAus ECTA).
Fill in the Blanks.
(1) 9
(2) 11
(3) 13
(4) 15
(5) 17

Ans: 3
Solution:
As of April 2022, India signed 13 Free Trade Agreements (FTAs) with its trading partners including major trade agreements like the India-UAE Comprehensive Partnership Agreement (CEPA) and the India-Australia Economic Cooperation and Trade Agreement (IndAus ECTA).

5. Consider the following statements concerning RoDTEP and choose the correct one(s)!
A. It was launched after the Merchandise Exports from India Scheme was launched.
B. Its Nodal Ministry is the Ministry of Commerce & Industries.
C. The scheme conforms to compliance guidelines laid out by WTO.
(1) Only A
(2) Only C
(3) A and B
(4) B and C
(5) A and C

Ans: 4
Solution:
The US had challenged India’s key export subsidy schemes in the WTO (World Trade Organisation), claiming them to harm the American workers. A dispute panel in the WTO ruled against India, stating that the export subsidy programmes that were provided by the Government of India violated the provisions of the trade body’s norms. The panel further recommended that the export subsidy programmes be withdrawn. This led to the birth of the RoDTEP Scheme, so as to ensure that India stays WTO-compliant.
Launch: 2021 [replacing the existing Merchandise Exports from India Scheme (MEIS) which was not WTO compliant]
Nodal Ministry: Ministry of Commerce and Industry
Scheme’s objective is to refund, currently un-refunded:
Duties/ taxes/ levies, at the Central, State & local level, borne on the exported product, including prior stage cumulative indirect taxes on goods & services used in production of the exported product, and
Such indirect Duties/ taxes/ levies in respect of the distribution of exported products.

Mint StatiCA MCQs – Jan 23, 2023

1. North-eastern states have recorded a compounded annual GST revenue growth rate of 27.5% so far since the implementation of the GST as against 14.8% for all states, making them the biggest gainers of the new indirect tax regime. The top five states that required compensation for the shortfall in GST revenue were _(A), Karnataka, Gujarat, Tamil Nadu, and Punjab.
Identify (A) in the above excerpt.
(1) Uttar Pradesh
(2) Maharashtra
(3) Andhra Pradesh
(4) Telangana
(5) None of these

Ans: 2
Solution:
North-eastern states have recorded a compounded annual GST revenue growth rate of 27.5% so far since the implementation of the GST as against 14.8% for all states, making them the biggest gainers of the new indirect tax regime. The top five states that required compensation for the shortfall in GST revenue were Maharashtra, Karnataka, Gujarat, Tamil Nadu, and Punjab. The five-year compensation period ended on 30 June, 2022.

2. The Reserve Bank of India will likely appeal the Bombay high court ruling that quashed the regulator and Yes Bank administrator’s decision to write off additional tier-I (AT-I) bonds to save the lender from collapsing.
Which of the following statements is/are true about AT-I bonds?
A. AT-1 bonds are a type of secured, perpetual bonds that banks issue to shore up their core capital base to meet the Basel-III norms.
B. AT-1 bonds are like any other bonds issued by banks and companies but pay a slightly higher rate of interest compared to other bonds.
C. AT-1 bonds are regulated by RBI.
Choose the correct option:
(1) Only A and B
(2) Only A and C
(3) Only B and C
(4) Only A
(5) None of these

Ans: 3
Solution:
AT-1 bonds are a type of unsecured, perpetual bonds that banks issue to shore up their core capital base to meet the Basel-III norms.
AT-1 bonds are like any other bonds issued by banks and companies but pay a slightly higher rate of interest compared to other bonds.
AT-1 bonds are regulated by RBI.

3. In a situation where a bank faces severe losses leading to erosion of regulatory capital, the RBI can decide if the bank has reached a situation wherein it is no longer viable. The RBI can then activate a PONV and assume executive powers of the bank.
What is ‘P’ in PONV?
(1) Principal
(2) Payment
(3) Perpetual
(4) Public
(5) None of these

Ans: 5
Solution:
In a situation where a bank faces severe losses leading to erosion of regulatory capital, the RBI can decide if the bank has reached a situation wherein it is no longer viable.
The RBI can then activate a Point of Non-Viability Trigger (PONV) and assume executive powers of the bank.
By doing so, the RBI can do whatever is required to get the bank on track, including superseding the existing management, forcing the bank to raise additional capital and so on.
However, activating PONV is followed by a write down of the AT-1 bonds, as determined by the RBI through the Banking Regulation Act, 1949.

4. Currently, there are a total of __ Nationalised Banks in India.
Fill in the Blanks.
(1) 8
(2) 10
(3) 12
(4) 14
(5) None of these

Ans: 3
Solution:
Currently, there are a total of 12 Nationalised Banks in India.
The list of nationalized banks is as follows:
State Bank of India
Punjab National Bank (With the Merger of Oriental Bank of Commerce and United Bank of India)
Bank of Baroda
Canara Bank (With Merger of Syndicate Bank)
Union Bank of India (With Merger of Andhra Bank and Corporation Bank)
Bank of India
Indian Bank (With Merger of Allahabad Bank)
Central Bank of India
Indian Overseas Bank
UCO Bank
Bank of Maharashtra
Punjab & Sind Bank

5. Markets regulator Sebi has launched an information database on municipal bonds. Municipal bonds were first issued in India in (A), five years after the _(B) Constitutional Amendment decentralized urban local bodies and gave them autonomy; made them accountable to citizens, and reformed their finances enabling them to access capital markets and financial institutions.
Identify (A) and (B) respectively in the above passage.
(1) 1994; 73rd
(2) 1997; 74th
(3) 1994; 74th
(4) 2001; 73rd
(5) 2005; 74th

Ans: 2
Bangalore was the first city in India to issue municipal bonds (Rs 125 Crores) in 1997 followed by Ahmedabad (Rs 100 Crores) in 1998.
Municipal bonds were first issued in India in 1997, five years after the 74th Constitutional Amendment decentralized urban local bodies and gave them autonomy; made them accountable to citizens and reformed their finances enabling them to access capital markets and financial institutions.
Securities and Exchange Board of India (SEBI)’s detailed guidelines for the issue and listing of municipal bonds in March 2015, clarified their regulatory status and rendered them safer for investors.
In 2017, Pune Municipal Corporation had raised Rs. 200 crores through municipal bonds at an interest of 7.59% to finance its 24×7 water supply project.

Mint StatiCA MCQs – Jan 20, 2023

1. The Supreme Court refused to stay a National Company Law Appellate Tribunal (NCLAT) order, directing Google to deposit 10% of a ₹1,337 crore fine imposed by India’s antitrust regulator _(A) for abusing the tech giant’s dominant position in the Android ecosystem.
Identify (A) in the above excerpt.
(1) NCLT
(2) CCI
(3) SEBI
(4) EXIM
(5) None of these

Ans: 2
Solution:
Competition Commission of India (CCI) is a statutory body of the Government of India responsible for enforcing the Competition Act, 2002, it was duly constituted in March 2009.
The Monopolies and Restrictive Trade Practices Act, 1969 (MRTP Act) was repealed and replaced by the Competition Act, 2002, on the recommendations of the Raghavan committee.

2. Retail prices of wheat and wheat flour (atta) have increased, and the government will soon take measures to control the rise, Food Secretary Sanjeev Chopra said. The secretary said that the stocks of wheat and rice in the FCI godowns are comfortable. FCI is a statutory body set up in __ under the Food Corporations Act. It was established against the backdrop of a major shortage of grains, especially wheat.
Fill in the Blanks.
(1) 1948
(2) 1965
(3) 1971
(4) 1975
(5) None of these

Ans: 2
Solution:
Food Corporation of India (FCI) is a Public Sector Undertaking, under the Department of Food & Public Distribution, Ministry of Consumer Affairs, Food and Public Distribution.
FCI is a statutory body set up in 1965 under the Food Corporations Act 1964. It was established against the backdrop of a major shortage of grains, especially wheat.

3. India, the world’s biggest sugar exporter after___(A)_, exported a record 11 million tonnes in the previous season to 30 Sept, 2022.
Which country has been depicted as (A) in the above passage?
(1) Cuba
(2) China
(3) Argentina
(4) Brazil
(5) Indonesia

Ans: 4
Solution:
India, the world’s biggest sugar exporter after Brazil, exported a record 11 million tonnes in the previous season to 30 Sept, 2022.

4. The Prime Minister laid the foundation stone of the headquarters building of the International Financial Services Centers Authority (IFSCA) in GIFT City, __.
Fill in the Blanks.
(1) Gandhinagar
(2) Mumbai
(3) Gurugram
(4) New Delhi
(5) Chennai

Ans: 1
Solution:
GIFT (Gujarat International Finance Tec-City) City is located in Gandhinagar, Gujarat.
It consists of a multi-service Special Economic Zone (SEZ), which houses India’s first International Financial Services Centre (IFSC) and an exclusive Domestic Tariff Area (DTA).
GIFT city (Gujarat International Finance Tec-City) is envisaged as an integrated hub for financial and technology services not just for India but for the world.

5. SAT is a statutory body established under the provisions of the Securities and Exchange Board of India Act, __.
Fill in the Blanks.
(1) 1988
(2) 1992
(3) 2002
(4) 2008
(5) 2012


Ans: 2
Solution:
Securities Appellate Tribunal (SAT)
SAT is a statutory body established under the provisions of the Securities and Exchange Board of India Act, 1992.
It is to hear and dispose of appeals against orders passed by the Securities and Exchange Board of India or by an adjudicating officer under the Act; and to exercise jurisdiction, powers and authority conferred on the Tribunal by or under this Act or any other law for the time being in force.

Mint StatiCA MCQs – Jan 16, 2023

1. FAME India is a part of the National Electric Mobility Mission Plan. The main thrust of FAME is to encourage electric vehicles by providing subsidies. Phase I of the scheme was started in _(A).
Identify (A) in the above excerpt.
(1) 2013
(2) 2015
(3) 2017
(4) 2019
(5) None of these

Ans: 2
Solution:
FAME India is a part of the National Electric Mobility Mission Plan. The main thrust of FAME is to encourage electric vehicles by providing subsidies.
The FAME India Scheme is aimed at incentivising all vehicle segments.
Phase I: started in 2015 and was completed on 31st March, 2019
The scheme covers Hybrid & Electric technologies like Mild Hybrid, Strong Hybrid, Plug in Hybrid & Battery Electric Vehicles.
Monitoring Authority: Department of Heavy Industries, the Ministry of Heavy Industries and Public Enterprises.
Fame India Scheme has four focus Areas:
-Technology development
-Demand Creation
-Pilot Projects
-Charging Infrastructure

2. (A)’s letter to the government citing its failure to meet its inflation mandate for three straight quarters remains under wraps, with Mint Road and North Block citing separate reasons for the secrecy.
Identify the organization (A).
(1) NITI Aayog
(2) SEBI
(3) RBI
(4) NABARD
(5) None of these

Ans: 3
Solution:
Reserve Bank of India’s letter to the government citing the bank’s failure to meet its inflation mandate for three straight quarters remains under wraps, with Mint Road and North Block citing separate reasons for the secrecy.

3. The latest _(A)__Business Confidence Index (BCI) jumped to 67.6 in the fiscal third quarter from 62.2 in the preceding quarter, even as much of the world braces for an impending recession in advanced economies.
Which organization has been depicted as (A) in the above passage?
(1) CII
(2) FICCI
(3) SBI
(4) SME India
(5) SME Network

Ans: 1
Solution:
Business confidence in India rose to a near two-year high in the December quarter, a survey by the Confederation of Indian Industry (CII) showed, reflecting optimism that India will escape the worst of the global economic turmoil.
The latest CII Business Confidence Index (BCI) jumped to 67.6 in the fiscal third quarter from 62.2 in the preceding quarter, even as much of the world braces for an impending recession in advanced economies.

4. Ude Desh Ka Aam Naagrik (UDAN) was launched as a Regional Connectivity Scheme (RCS) under the Ministry of Civil Aviation in __.
Fill in the Blanks.
(1) 2014
(2) 2015
(3) 2016
(4) 2017
(5) 2018

Ans: 3
Solution:
Ude Desh Ka Aam Naagrik (UDAN) was launched as a Regional Connectivity Scheme (RCS) under the Ministry of Civil Aviation in 2016.

5. The National Action Plan on Climate Change (NAPCC) was launched in ____by the Prime Minister’s Council on Climate Change.
Fill in the Blanks.
(1) 1995
(2) 1998
(3) 2002
(4) 2008
(5) 2012

Ans: 4
Solution:
The National Action Plan on Climate Change (NAPCC) was launched in 2008 by the Prime Minister’s Council on Climate Change.
It aims at creating awareness among the representatives of the public, different agencies of government, scientists, industry, and communities on the threat posed by climate change and the steps to counter it.
There are 8 national missions forming the core of the NAPCC which represent multi-pronged, long term and integrated strategies for achieving key goals in climate change. These are-
National Solar Mission
National Mission for Enhanced Energy Efficiency
National Mission on Sustainable Habitat
National Water Mission
National Mission for Sustaining the Himalayan Ecosystem
National Mission for A Green India
National Mission for Sustainable Agriculture
National Mission on Strategic Knowledge for Climate Change

Mint StatiCA MCQs – Jan 13, 2023

1. Retail inflation cooled to a one-year low in December, remaining below the Reserve Bank of India’s upper tolerance limit of _(A)% for the second straight month, thanks primarily to lower vegetable prices.
Identify (A) in the above excerpt.
(1) 4
(2) 6
(3) 8
(4) 10
(5) None of these

Ans: 2
Solution:
Under Section 45ZA, the Central Government, in consultation with the RBI, determines the inflation target in terms of the Consumer Price Index (CPI), once in five years and notifies it in the Official Gazette. Accordingly, on August 5, 2016, the Central Government notified in the Official Gazette 4 per cent Consumer Price Index (CPI) inflation as the target for the period from August 5, 2016 to March 31, 2021 with the upper tolerance limit of 6 per cent and the lower tolerance limit of 2 per cent. On March 31, 2021, the Central Government retained the inflation target and the tolerance band for the next 5-year period – April 1, 2021 to March 31, 2026.

2. IIP is an indicator that measures the changes in the volume of production of industrial products during a given period. It is compiled and published monthly by the National Statistical Office (NSO), Ministry of Statistics and Programme Implementation. Eight Core Sectors comprise __% of the weight of items included in the Index of Industrial Production (IIP).
Fill in the Blanks.
(1) 30.60
(2) 35.55
(3) 41.27
(4) 45.32
(5) None of these

Ans: 5
Solution:
IIP is an indicator that measures the changes in the volume of production of industrial products during a given period.
It is compiled and published monthly by the National Statistical Office (NSO), Ministry of Statistics and Programme Implementation.
Base Year for IIP is 2011-2012.
Eight Core Sectors comprise 40.27% of the weight of items included in the Index of Industrial Production (IIP).
The eight core sector industries in decreasing order of their weightage: Refinery Products> Electricity> Steel> Coal> Crude Oil> Natural Gas> Cement> Fertilizers.

3. The _(A)’s Global Economic Prospects report warned of a global recession this year, adding a sharp and long-lasting slowdown is expected to hit developing countries hard due to elevated inflation, abrupt interest rate spikes, the resurgence of the covid-19 pandemic, reduced investment, and geopolitical tensions caused by Russia’s war in Ukraine.
Which organisation has been depicted as (A) in the above passage?
(1) IMF
(2) OECD
(3) World Bank
(4) ADB
(5) NDB

Ans: 3
Solution:
The World Bank in its latest “Global Economic Prospects” stated that India’s growth is slowing to 6.6% in FY24.
Globally, the bank is forecasting a sharp, long-lasting slowdown, with global growth declining to 1.7% in 2023 from the 3% expected just six months ago.

4. Which of the following statements is/are not true about Fiscal Deficit?
A. The government describes fiscal deficit of India as “the excess of total disbursements from the Consolidated Fund of India, excluding repayment of the debt, over total receipts into the Fund (excluding the debt receipts) during a financial year”.
B. Fiscal Deficit = Total expenditure of the government (capital and revenue expenditure) – Total income of the government (Revenue receipts + recovery of loans + other receipts).
C. The NK Singh committee (set up in 2017) recommended that the government should target a fiscal deficit of 3% of the GDP in years up to 31st March, 2020, cut it to 2.8% in 2020-21 and to 2.5% by 2023.
Choose the correct option:
(1) Only C
(2) Only A and B
(3) Only B and C
(4) Only A and C
(5) A, B and C

Ans: 1
Solution:
The government describes fiscal deficit of India as “the excess of total disbursements from the Consolidated Fund of India, excluding repayment of the debt, over total receipts into the Fund (excluding the debt receipts) during a financial year”.
In simple words, it is a shortfall in a government’s income compared with its spending.
A government that has a fiscal deficit is spending beyond its means.
Fiscal Deficit = Total expenditure of the government (capital and revenue expenditure) – Total income of the government (Revenue receipts + recovery of loans + other receipts)
The NK Singh committee (set up in 2016) recommended that the government should target a fiscal deficit of 3% of the GDP in years up to 31st March, 2020, cut it to 2.8% in 2020-21 and to 2.5% by 2023.

5. _ became the 36th State/UT to implement One Nation One Ration Card (ONORC) in 2022.
Fill in the Blanks.
(1) Odisha
(2) West Bengal
(3) Assam
(4) Tripura
(5) Nagaland

Ans: 3
Solution:
The ONORC scheme is being implemented under National Food Security Act (NFSA).
This system allows all NFSA beneficiaries, particularly migrant beneficiaries, to claim either full or part foodgrains from any Fair Price Shop (FPS) in the country through existing ration card with biometric/Aadhaar authentication in a seamless manner.
The system also allows their family members back home, if any, to claim the balance foodgrains on same the ration card.
The implementation of ONORC was initiated in August 2019.
Assam has become the 36th State/UT to implement One Nation One Ration Card (ONORC).

Mint StatiCA MCQs – Jan 12, 2023

1. Accounting rule maker Institute of Chartered Accountants of India (ICAI) will soon issue fresh instructions to professionals to make tax audits more foolproof. Affairs of the ICAI are managed with the provisions of the Chartered Accountants Act, _(A) and the Chartered Accountants Regulations, 1988.
Identify (A) in the above excerpt.
(1) 1949
(2) 1955
(3) 1961
(4) 1965
(5) None of these

Ans: 1
Solution:
The government of India passed the Chartered Accountants Act in 1949.
Under Section 3 of the said Act, ICAI is established as a statutory body.
It functions under the administrative control of the Ministry of Corporate Affairs, Government of India.
Affairs of the ICAI are managed with the provisions of the Chartered Accountants Act, 1949 and the Chartered Accountants Regulations, 1988.

2. Which of the following statements is/are correct about the Pradhan Mantri Bhartiya Janaushadhi Pariyojana (PMBJP)?
A. PMBJP is a campaign launched by the Department of Pharmaceuticals in 2006 under the name Jan Aushadhi Campaign.
B. The campaign was revamped as PMBJP in 2015-16.
C. Bureau of Pharma PSUs of India (BPPI) is the implementation agency for PMBJP. The Bureau of Pharma Public Sector Undertakings (PSUs) of India works under the Ministry of Health and Family Welfare.
Choose the correct option:
(1) Only A and B
(2) Only B and C
(3) Only A
(4) Only B
(5) A, B and C

Ans: 4
Solution:
PMBJP is a campaign launched by the Department of Pharmaceuticals in 2008 under the name Jan Aushadhi Campaign.
The campaign was revamped as PMBJP in 2015-16.
The Bureau of Pharma PSUs of India (BPPI) is the implementation agency for PMBJP.
The Bureau of Pharma Public Sector Undertakings (PSUs) of India works under the Ministry of Chemicals & Fertilisers.

3. The Centre’s direct tax collections have surpassed 86% of the budget target for 2022-23 so far, providing a cushion to the government amid higher-than-estimated spending due to global economic disruptions, data released by the central board of direct taxes (CBDT) showed.
Who is the present Chairman of CBDT?
(1) Sushil Chandra
(2) Nitin Gupta
(3) Pramod Chandra Mody
(4) Jagannath Bidyadhar Mahapatra
(5) None of these

Ans: 2
Solution:
Nitin Gupta, a 1986 batch Indian Revenue Services officer, has been appointed as the new chairman of the Central Board of Direct Taxes (CBDT), the administrative body of the Income Tax department.

4. The Union cabinet on Wednesday approved a ₹2,600 crore scheme to incentivize the use of unified payments interface (UPI) and RuPay debit cards for low-value transactions. Which of the following statements is/are not true about UPI?
A. It is an advanced version of Immediate Payment Service (IMPS)- round–the-clock funds transfer service to make cashless payments faster, easier, and smoother.
B. NPCI launched UPI with 21 member banks in 2015.
C. The value of transactions made using the UPI crossed USD100 billion in a month for the first time in October, 2022 according to data from the NPCI, further cementing its position as India’s most popular digital payments system.
Choose the correct option:
(1) Only A
(2) Only A and B
(3) Only B and C
(4) Only A and C
(5) A, B and C

Ans: 3
Solution:
Unified Payments Interface (UPI)
It is an advanced version of Immediate Payment Service (IMPS)- round–the-clock funds transfer service to make cashless payments faster, easier, and smoother.
UPI is a system that powers multiple bank accounts into a single mobile application (of any participating bank), merging several banking features, seamless fund routing & merchant payments into one hood.
UPI is currently the biggest among the National Payments Corporation of India (NPCI) operated systems including National Automated Clearing House (NACH), Immediate Payment Service (IMPS), Aadhaar enabled Payment System (AePS), Bharat Bill Payment System (BBPS), RuPay etc.
The top UPI apps today include PhonePe, Paytm, Google Pay, Amazon Pay and BHIM, the latter being the Government offering.
As part of an agreement, India’s UPI will be linked to Singapore’s PayNow.
NPCI launched UPI with 21 member banks in 2016.
The value of transactions made using the UPI crossed USD100 billion in a month for the first time in October, 2021 according to data from the NPCI, further cementing its position as India’s most popular digital payments system.

5. The Reserve Bank of India (RBI) governor inaugurated the Reserve Bank Innovation Hub (RBIH) in Bengaluru. It has been set up as a _ company under the Companies Act, 2013 to create an ecosystem that focuses on promoting access to financial services and products for the low-income population in the country. It is a wholly owned subsidiary of the RBI.
Fill in the Blanks.
(1) Section 8
(2) Section 14
(3) Section 18
(4) Section 25
(5) None of these

Ans: 1
Solution:
The Reserve Bank of India (RBI) governor inaugurated the Reserve Bank Innovation Hub (RBIH) in Bengaluru.
It has been set up as a Section 8 company under the Companies Act, 2013 to create an ecosystem that focuses on promoting access to financial services and products for the low-income population in the country.
It is a wholly owned subsidiary of the RBI.

Mint StatiCA MCQs – Jan 11, 2023

1. Petroleum minister Hardeep Singh Puri said that India had increased the number of its crude oil suppliers from 27 countries in 2006-07 to _(A) in 2021-22, with Columbia, Russia, Libya, Gabon, Equatorial Guinea, among others, now part of the list.
Identify (A) in the above excerpt.
(1) 32
(2) 39
(3) 42
(4) 45
(5) 49

Ans: 2
Solution:
Petroleum minister Hardeep Singh Puri said that India had increased the number of its crude oil suppliers from 27 countries in 2006-07 to 39 in 2021-22, with Columbia, Russia, Libya, Gabon, Equatorial Guinea, among others, now part of the list.

2. Which of the following statements is/are correct about Inverted Duty Structure?
A. Inverted Duty Structure denotes the prevalence of higher taxes on inputs than on finished products.
B. It hurts the government’s exchequer by resulting in revenue loss.
C. It increases the input tax credit which further creates administrative problems under the GST regime.
Choose the correct option:
(1) Only A and B
(2) Only B and C
(3) Only A and C
(4) Only A
(5) A, B and C

Ans: 5
Solution:
An inverted duty structure arises when the taxes on output or final product is lower than the taxes on inputs, creating an inverse accumulation of input tax credit which in most cases has to be refunded.
Problems with the current inverted duty structure under GST
The inverted duty structure is causing several administrative problems in our GST system:
Taxpayers will have accumulated credits in the form of refund claims with the tax department.
The inverted duty structure is a revenue loss for the government as it has to refund the tax already paid (in inputs).
Under GST, the inverted duty structure is identified for goods and not for services. Or in other words, there is recognition for ‘input good’ and not for ‘input services.’

3. Sustainable Development refers to ‘Development which meets the needs of the present without compromising the ability of future generations to meet their own needs’. This most widely accepted definition of Sustainable Development was given by the Brundtland Commission in its report Our Common Future in __.
Fill in the Blanks.
(1) 1972
(2) 1975
(3) 1979
(4) 1984
(5) 1987

Ans: 5
Solution:
Sustainable Development
‘Development which meets the needs of the present without compromising the ability of future generations to meet their own needs’.
This most widely accepted definition of Sustainable Development was given by the Brundtland Commission in its report ‘Our Common Future’ (1987).
Sustainable development (SD) calls for concerted efforts towards building an inclusive, sustainable, and resilient future for people and the planet.

4. Which of the following statements is/are true about the wheat crop in India?
A. Wheat is a kharif crop that requires a cool growing season and bright sunshine at the time of ripening.
B. Well-drained fertile loamy and clayey loamy is best suited for wheat crop in India.
C. It is the second most important cereal crop in India after rice.
Choose the correct option:
(1) Only A
(2) Only A and B
(3) Only B and C
(4) Only A and C
(5) A, B and C

Ans: 3
Solution:
Wheat is a rabi crop that requires a cool growing season and bright sunshine at the time of ripening.
Temperature: Between 10-15°C (Sowing time) and 21-26°C (Ripening & Harvesting) with bright sunlight.
Rainfall: Around 75-100 cm.
Soil Type: Well-drained fertile loamy and clayey loamy (Ganga-Satluj plains and black soil region of the Deccan).
It is the second most important cereal crop in India after rice.

5. Concerning ‘Water Credit’, consider the following statements:
A. It puts microfinance tools to work in the water and sanitation sector.
B. It is a global initiative launched under the aegis of the World Health Organization and the World Bank.
C. It aims to enable poor people to meet their water needs without depending on subsidies.
Which of the statements given above are correct?
(1) A and B only
(2) B and C only
(3) A and C only
(4) A, B and C
(5) None of these

Ans: 5
Solution:
Water Credit is a powerful solution and the first to put microfinance tools to work in the water and sanitation sector. Water Credit helps bring small loans to those who need access to affordable financing and expert resources to make household water and toilet solutions a reality.
Water Credit is an initiative of Water.org. Water.org is a global non-profit organization working to bring water and sanitation to the world. 
It helps people get access to safe water and sanitation through affordable financing, such as small loans.

Mint StatiCA MCQs – Jan 10, 2023

1. Which of the following statements best describes the term ‘Scheme for Sustainable Structuring of Stressed Assets (S4A)’?
(1) It is a procedure for considering the ecological costs of developmental schemes formulated by the Government.
(2) It is a scheme of RBI for reworking the financial structure of big corporate entities facing genuine difficulties.
(3) It is a disinvestment plan of the Government regarding Central Public Sector Undertakings.
(4) It is an important provision in The Insolvency and Bankruptcy Code’ recently implemented by the Government.
(5) None of these

Ans: 2
Solution:
The scheme was launched by the Reserve Bank of India (RBI) on 13 June 2016, named as Scheme for Sustainable Structuring of Stressed Assets (S4A Scheme) for addressing the large, stressed assets of the corporate sector with banks.
The S4A Scheme aims at a deep financial restructuring of big debted projects by allowing lenders (banks) to acquire equity in the stressed project. In this context, the scheme makes the financial restructuring of large projects at the same time, helping the lender’s ability to deal with such stressed assets. It is intended to restore the flow of credit to critical sectors including infrastructure.

2. IDBI (Industrial Development Bank of India) Bank was established in _ by an Act of Parliament as a wholly owned subsidiary of the Reserve Bank of India (RBI).
Fill in the Blanks.
(1) 1964
(2) 1975
(3) 1979
(4) 1985
(5) 1989

Ans: 1
Solution:
IDBI (Industrial Development Bank of India) Bank was established in 1964 by an Act of Parliament as a wholly owned subsidiary of the Reserve Bank of India (RBI).
It was established as a specialised development financial institution (DFI) – on the lines of NABARD, NHB and SIDBI – to provide credit and other financial facilities for the development of Indian industry.
In 1976, the ownership of IDBI was transferred to the Government of India.
In 2004 it was converted into a commercial bank when RBI incorporated IDBI as a ‘scheduled bank’ under the RBI Act, of 1934.
August 2018: The cabinet approved the acquisition of a controlling stake by LIC as a promoter in the bank through a combination of preferential allotment and an open offer of equity.
September 2018: Following the transfer of controlling 51 % to LIC from the government of India, the government holding came down to 46.46 % from nearly 86 %, while LIC stake increased from about 8 % to 51 %.
January 2019: LIC completed the acquisition of a 51 % controlling stake in IDBI Bank, marking the entry of the LIC into the banking space.
March 2019: RBI changed the categorisation of IDBI Bank to a private sector lender from a public sector lender following the acquisition of a majority stake by LIC.

3. According to the new definition of MSMEs, businesses in the manufacturing industry, the wholesale industry, the retail industry, and the service industry are classified as “micro” enterprises, when their investments in plants, machineries, or equipment do not exceed Rs. (A), while their annual turnovers remain below _(B).
Identify (A) and (B) respectively in the above excerpt.
(1) 50 Lakhs; 1 Crore
(2) 1 Crore; 5 Crores
(3) 5 Crores; 50 Crores
(4) 1 Crore; 10 Crores
(5) None of these

Ans: 2
Solution:
The Indian Government has modified the definition of MSMEs under the Atmanirbhar Package announced in the year 2020 to increase the number of enterprises eligible for being classified as micro, small and medium enterprises. For this purpose, the threshold limits of investment and turnovers of MSMEs have been raised higher and have been extended to both goods-based and service-based businesses. Besides, the method of calculation of investments and turnovers have also been altered to introduce two new concepts of net investments and net turnovers.

4. MCA21 is the online portal of the Ministry of Corporate Affairs (MCA) that has made all company-related information accessible to various stakeholders and the general public. It was launched in __.
Fill in the Blanks.
(1) 1995
(2) 2006
(3) 2010
(4) 2014
(5) 2016

Ans: 2
Solution:
MCA21 is the online portal of the Ministry of Corporate Affairs (MCA) that has made all company-related information accessible to various stakeholders and the general public. It was launched in 2006.

5. (A) is the minimum percentage of deposits that a commercial bank has to maintain in the form of liquid cash, gold or other securities. It is fixed by the RBI and is a form of control over credit growth in India. The government uses it to regulate inflation and fuel growth. It was prescribed by Section 24 (2A) of the Banking Regulation Act, of 1949.
Identify (A) in the above excerpt.
(1) Repo Rate
(2) SDF
(3) MSF
(4) CRR
(5) SLR

Ans: 5
Solution:
The statutory Liquidity Ratio or SLR is the minimum percentage of deposits that a commercial bank has to maintain in the form of liquid cash, gold or other securities. It is basically the reserve requirement that banks are expected to keep before offering credit to customers. The SLR is fixed by the RBI and is a form of control over credit growth in India.
The government uses the SLR to regulate inflation and fuel growth. Increasing the SLR will control inflation in the economy while decreasing the statutory liquidity rate will cause growth in the economy. The SLR was prescribed by Section 24 (2A) of the Banking Regulation Act, of 1949.

Mint StatiCA MCQs – Jan 9, 2023

1. The bidding for Reliance Capital took an unexpected twist, with the committee of creditors (CoC) considering a second round of e-auction as the bidders tried to top each other’s bids after the completion of the auction.
Which of the following statements is/are not true about the Insolvency and Bankruptcy Code in India?
A. It was enacted in 2016, against the backdrop of mounting non-performing loans, with a view to establishing a consolidated framework for insolvency resolution of corporations, partnership firms and individuals in a time-bound manner
B. The IBC sets out three classes of persons who can trigger the corporate insolvency resolution process (CIRP) – financial creditors, operational creditors, and corporate debtors.
C. When a corporate debtor or a company which has taken loans, defaults on its loan repayment, either the creditor or the debtor can apply for the initiation of a Corporate Insolvency Resolution Process (CIRP) under Section 6 of the IBC.
Choose the correct option:
(1) Only A and B
(2) Only B and C
(3) Only A and C
(4) Only B
(5) None of these

Ans: 5
Solution:
Insolvency is a situation where individuals or companies are unable to repay their outstanding debt.
The Insolvency and Bankruptcy Code, 2016 (IBC) was enacted in 2016 against mounting non-performing loans.
The IBC sets out three classes of persons who can trigger the corporate insolvency resolution process (CIRP) – financial creditors, operational creditors, and corporate debtors.
When a corporate debtor, or a company which has taken loans, defaults on its loan repayment, either the creditor or the debtor can apply for the initiation of a Corporate Insolvency Resolution Process (CIRP) under Section 6 of the IBC.
The minimum amount of default after which the creditor or debtor could apply for insolvency is Rs. 1 crore.
The IBC was amended to complete the resolution process by 330 days from the earlier 180+90 days deadline.

2. A flood of imports swelled the Centre’s customs duty receipts in the three months to November, reversing the declining trend of most of the first half of the current fiscal.
Which of the following statements is/are true about Customs Duty?
A. Customs Duty is a tax collected on imports and exports by the customs authorities of a country. It is usually based on the value of goods that are imported. Depending on the context, import duty may also be referred to as tariff, import tax, customs duty, and import tariff.
B. The Customs Act of 1965 defines custom duty in India.
C. GST, which went into effect on July 1, 2017, absorbed most indirect taxes, including Additional Duties of Customs (commonly known as CVD) and Special Additional Duty of Customs (SAD).
Choose the Correct option:
(1) Only A and B
(2) Only B and C
(3) Only A and C
(4) Only A
(5) All of the Above

Ans: 3
Solution:
Customs duty is a charge levied by a country’s customs authority on imports and exports. It is usually calculated depending on the value of imported items.
It is sometimes used as a tool to penalize a particular nation by charging high import duties on its products.
The customs duty is a kind of fee that is collected by the customs authorities for the movement of goods and services to and from that country.
The tax that is levied for the import of products is referred to as import duty, while the tax levied on the goods that are exported to some other country is known as export duty.
Import tariffs are intended to generate revenue for local governments while also providing a competitive advantage to locally grown or manufactured commodities that are not subject to import duties.
Customs duty rates are either specific or ad valorem, meaning they are calculated depending on the worth of the goods.
Almost all items brought into the country are subject to customs taxes.
Life-saving medications, fertilizers, and food grains are exempt from customs duties.
The Customs Act of 1962 defines custom duty in India.
GST, which went into effect on July 1, 2017, absorbed most indirect taxes, including Additional Duties of Customs (commonly known as CVD) and Special Additional Duty of Customs (SAD).
The IGST is levied on imported goods to level the playing field for domestic manufacturers.
The value on which IGST is calculated is equal to the Value of Imported Goods + Basic Customs Duty + Social Welfare Surcharge

3. Under the SAMARTH Scheme of the Textile Ministry, more than 13,235 artisans have been trained in the last three years. Samarth is a flagship skill development scheme approved in continuation to the _(A) for the 12th Five Year Plan (FYP), Cabinet Committee of Economic Affairs.
Identify (A) in the above excerpt.
(1) Scheme for Integrated Textile Park (SITP)
(2) Integrated Skill Development Scheme
(3) Power-Tex India
(4) Amended Technology Upgradation Fund Scheme
(5) National Technical Textile Mission

Ans: 2
Solution:
Samarth (Scheme for Capacity Building In Textile Sector) is a flagship skill development scheme approved in continuation to the Integrated Skill Development Scheme for the 12th Five Year Plan (FYP), Cabinet Committee of Economic Affairs.
The office of the Development Commissioner (Handicrafts) is implementing the SAMARTH to provide skill training to handicraft artisans under the component ‘Skill Development in Handicrafts Sector’ of National Handicrafts Development Programme (NHDP).

4. The Department of Industrial Policy & Promotion (DIPP) was renamed as the Department for Promotion of Industry and Internal Trade (DPIIT) in _. The idea behind the rechristening of the DIPP is to include internal trade in its ambit.
Fill in the Blanks.
(1) 2015
(2) 2016
(3) 2017
(4) 2018
(5) 2019

Ans: 5
Solution:
The Department of Industrial Policy & Promotion (DIPP) was renamed as the Department for Promotion of Industry and Internal Trade (DPIIT) in 2019. The idea behind the rechristening of the DIPP is to include internal trade in its ambit.
After Internal Trade was added to the mandate of DIPP, the department was renamed as the Department for Promotion of Industry and Internal Trade (DPIIT). Administered by the Ministry of Commerce and Industry, it is a nodal Government agency with a responsibility to formulate and implement growth strategies for the Industrial Sector along with other Socio-Economic objectives and national priorities.

5. Cash Reserve Ratio (CRR) is the share of a bank’s total deposit that is mandated by the Reserve Bank of India (RBI) to be maintained with the latter as reserves in the form of liquid cash. Under section _ of the Reserve Bank of India Act, 1934, all Scheduled Banks are required to maintain with the Reserve Bank of India Cash Reserve Ratio (CRR).
Fill in the Blanks.
(1) Section 42
(2) Section 24
(3) Section 18
(4) Section 56
(5) Section 22

Ans: 1
Solution:
Cash Reserve Ratio (CRR) is the share of a bank’s total deposit that is mandated by the Reserve Bank of India (RBI) to be maintained with the latter as reserves in the form of liquid cash.
Under section 42(1) of the Reserve Bank of India Act, 1934, all Scheduled Banks are required to maintain with the Reserve Bank of India a Cash Reserve Ratio (CRR) of 4.5% of Net Demand and Time Liabilities (NDTL).

Mint StatiCA MCQs – Jan 6, 2023

1. While India’s headline inflation is finally showing signs of easing, the country’s services sector is seeing a demand rebound- and higher prices to match—after two years of pandemic-induced slump.
Which of the following statements is/are not true about the services sector in India?
A. India’s services sector remains the engine of growth for India’s economy and contributed 53% to India’s Gross Value Added at current prices in FY21-22.
B. According to the WTO, India’s services export market share improved from 3% in 2010 to 3.5% in 2019 and 4% in 2020 and 2021.
C. The Services Sector in India was the largest recipient of FDI inflows worth US$ 94.19 billion between April 2000-March 2022. The services category ranked 1st in FDI inflow as per data released by the Department for Promotion of Industry and Internal Trade (DPIIT).
Choose the correct option:
(1) Only A and B
(2) Only B and C
(3) Only A and C
(4) Only B
(5) None of these

Ans: 5
Solution:
India’s services sector remains the engine of growth for India’s economy and contributed 53% to India’s Gross Value Added at current prices in FY21-22.
According to the WTO, India’s services export market share improved from 3% in 2010 to 3.5% in 2019 and 4% in 2020 and 2021.
The Services Sector in India was the largest recipient of FDI inflows worth US$ 94.19 billion between April 2000-March 2022. The services category ranked 1st in FDI inflow as per data released by the Department for Promotion of Industry and Internal Trade (DPIIT).

2. The fundamental cause of cost-push inflation is rising production costs. Which of the following reasons can cause production costs to rise?
(1) Employees’ salaries being raised
(2) Raw material prices increasing
(3) Increase in Import prices
(4) Increase in indirect taxes
(5) All of the Above
Ans: 5
Solution:
The fundamental cause of cost-push inflation is rising production costs. The following reasons can cause production costs to rise:
Employees’ salaries being raised
Raw material prices increasing
Increase in Import prices
Increase in indirect taxes


3. Which of the following statements is/are true about the National Monetisation Pipeline?
A. The NMP estimates an aggregate monetisation potential of Rs 12 lakh crores through core assets of the Central Government, over a four-year period, from FY 2022 to FY 2025.
B. It aims to unlock value in greenfield projects by engaging the private sector, transferring to them revenue rights and not ownership in the projects, and using the funds generated for infrastructure creation across the country.
C. The NMP will run co-terminus with the Rs 100 lakh crore National Infrastructure Pipeline (NIP) announced in December 2019.
(1) Only C
(2) Only A and B
(3) Only A and C
(4) Only B
(5) Only B and C

Ans: 1
Solution:
The NMP estimates an aggregate monetisation potential of Rs 6 lakh crores through core assets of the Central Government, over a four-year period, from FY 2022 to FY 2025.
It aims to unlock value in brownfield projects by engaging the private sector, transferring to them revenue rights and not ownership in the projects, and using the funds generated for infrastructure creation across the country.
The NMP will run co-terminus with the Rs 100 lakh crore National Infrastructure Pipeline (NIP) announced in December 2019.

4. India assumed the presidency of the G20 on 1 December last year. G20 is one of the most important global forums for economic governance, with its membership comprising (A)% of global GDP, (B)% of international trade and two-thirds of the world’s population. The theme unveiled by Prime Minister Narendra Modi for G20 is ‘One Earth, One Family, One Future.’
Identify (A) and (B) respectively in the above excerpt.
(1) 90; 80
(2) 85; 75
(3) 75; 65
(4) 60; 75
(5) 65; 80

Ans: 2
Solution:
India assumed the presidency of the G20 on 1 December last year. G20 which is one of the most important global forums for economic governance, with its membership comprising 85% of the global GDP, 75% of international trade and two-thirds of the world’s population. The theme unveiled by Prime Minister Narendra Modi for G20 is ‘One Earth, One Family, One Future.’

5. Which of the following statements is/are Incorrect about the recommendations made by the first Narasimhan Committee?
A. A 4-tier hierarchy for the Indian banking system with 3 or 4 major public sector banks at the top and rural development banks for agricultural activities at the bottom
B. Reduction in statutory liquidity ratio
C. Reaching of 9% capital adequacy ratio
Choose the correct option:
(1) Only A and B
(2) Only B and C
(3) Only C
(4) Only A and C
(5) None of these

Ans: 3
Solution:
The Committee was set up under the chairmanship of Maidavolu Narasimham. He was the 13th governor of the Reserve Bank of India (RBI) from 2 May 1977 to 30 November 1977. There was another Committee, this time under P Chidambaram as the finance minister, headed by Narasimham, which was formed in 1998. The first Committee was set up in 1991 and is referred to as the Narasimham Committee- I and the 1998 Committee is known as the Narasimham Committee – II.
The first Narasimhan Committee made the following recommendations for the growth of the banking sector:
A 4-tier hierarchy for the Indian banking system with 3 or 4 major public sector banks at the top and rural development banks for agricultural activities at the bottom
A quasi-autonomous body under RBI for supervising banks and financial institutions
Reduction in statutory liquidity ratio
Reaching of 8% capital adequacy ratio
Deregulation of Interest rates
Full discloser banks’ accounts and proper classification of assets
Setting up Asset Reconstruction fund

Mint StatiCA MCQs – Jan 5, 2023

1. The Union cabinet approved the national green hydrogen mission, allocating ₹19,744 crore to produce 5 million tonnes of green hydrogen annually by (A).
Identify (A) in the above excerpt.
(1) 2025
(2) 2030
(3) 2050
(4) 2070
(5) None of these

Ans: 2
Solution:
The Union cabinet on Wednesday approved the national green hydrogen mission, allocating ₹19,744 crores to produce 5 million tonnes of green hydrogen annually by 2030.

2. The National Asset Reconstruction Co. Ltd (NARCL) emerged as the top bidder for the assets of the Kolkata-based Srei Group after a lengthy auction process. The NARCL will first purchase bad loans from banks. It will pay (A) of the agreed price in cash and the remaining _(B) will be in the form of “Security Receipts”.
Identify (A) and (B) respectively in the above excerpt.
(1) 10 %; 90 %
(2) 15 %; 85 %
(3) 30 %; 70 %
(4) 40 %; 60 %
(5) None of the above

Ans: 2
Solution:
NARCL has been incorporated under the Companies Act and has applied to the Reserve Bank of India for a license as an Asset Reconstruction Company (ARC).
NARCL will acquire stressed assets worth about Rs 2 lakh crore from various commercial banks in different phases.
Public Sector Banks (PSBs) will maintain 51% ownership in NARCL.
The NARCL will first purchase bad loans from banks.
It will pay 15% of the agreed price in cash and the remaining 85% will be in the form of “Security Receipts”.
When the assets are sold, with the help of IDRCL, the commercial banks will be paid back the rest.
If the bad bank is unable to sell the bad loan or has to sell it at a loss, then the government guarantee will be invoked.

3. Volatility in natural gas prices due to the Russia-Ukraine war is expected to add a dash of uncertainty to India’s budget estimations for the next year, potentially upending calculations for fertilizer subsidies. The Department of Fertilizers (DoF) has made it mandatory for all domestic producers to produce __ urea as Neem Coated Urea (NCU).
Fill in the Blanks.
(1) 50 %
(2) 60 %
(3) 80 %
(4) 90 %
(5) 100 %

Ans: 5
Solution:
The Department of Fertilizers (DoF) has made it mandatory for all domestic producers to produce 100% urea as Neem Coated Urea (NCU).
New Urea Policy (NUP) 2015
Objectives of the policy are-
To maximize indigenous urea production.
To promote energy efficiency in the urea units.
To rationalize the subsidy burden on the Government of India.

4. Services sector activity in India rose to a six-month high in December, driven by an increase in new orders and sustained demand. The services sector of India remains the engine of growth for India’s economy and contributed nearly (A)% to India’s Gross Value Added at current prices in FY21-22.
Identify (A) in the excerpt.
(1) 43
(2) 48
(3) 53
(4) 58
(5) 65

Ans: 3
Solution:
The services sector is the most dominant sector of the Indian economy in terms of GDP. It has also attracted significant foreign investment and is a major contributor to India’s exports. It also provides employment to ~30% of India’s workforce. The services sector in India covers a wide variety of activities such as trade, hotel and restaurants, transport, storage and communication, financing, insurance, real estate, business services, community, social and personal services, and services associated with construction. Services Sector is poised to play a vital role in achieving the goals of a US$ 5 trillion economy in the medium term and a developed economy status in the long term.
The services sector of India remains the engine of growth for India’s economy and contributed 53% to India’s Gross Value Added at current prices in FY21-22.

5. Consider the following actions that the Government can take:
A. Devaluing the domestic currency.
B. Reduction in the export subsidy.
C. Adopting suitable policies which attract greater FDI and more funds from FIIs.
Which of the above action/actions can help in reducing the current account deficit?
(1) A and B
(2) B and C
(3) C only
(4) A and C
(5) None of these

Ans: 4
Solution:
A current account deficit occurs when the total value of goods and services a country imports exceeds the total value of goods and services it exports.
The balance of exports and imports of goods is referred to as the trade balance. Trade Balance is a part of the ‘Current Account Balance’.
Devaluing the domestic currency supports the exports, thereby bringing down the CAD.
Similarly, adopting suitable policies which attract greater FDI and more funds from FIIs also helps in reducing the CAD.
However, a decrease in export subsidies hampers the exports, thereby increasing CAD.

Mint StatiCA MCQs – Jan 4, 2023

1. The National Company Law Tribunal, Mumbai, granted interim relief to Torrent Investments, by directing the administrator of bankrupt Reliance Capital Ltd not to present the revised offer Hinduja group made past the auction deadline to the lenders until the next hearing date.
Which of the following statements is/are true about NCLT?
A. NCLT is a quasi-judicial authority incorporated for dealing with corporate disputes that are of civil nature arising under the Companies Act.
B. The Central Government has constituted National Company Law Tribunal (NCLT) under section 408 of the Companies Act, 2013 (18 of 2013) w.e.f. 01st June 2017.
C. In the first phase the Ministry of Corporate Affairs has set up eleven Benches, one Principal Bench at New Delhi and ten other Benches at New Delhi, Ahmedabad, Allahabad, Bengaluru, Chandigarh, Chennai, Guwahati, Hyderabad, Kolkata, and Mumbai.
Choose the correct one:
(1) Only A and C
(2) Only A and B
(3) Only B
(4) Only B and C
(5) A, B and C

Ans: 1
Solution:
National Company Law Tribunal
The Central Government has constituted National Company Law Tribunal (NCLT) under section 408 of the Companies Act, 2013 (18 of 2013) w.e.f. 01st June 2016. In the first phase, the Ministry of Corporate Affairs has set up eleven Benches, one Principal Bench at New Delhi and ten other Benches at New Delhi, Ahmedabad, Allahabad, Bengaluru, Chandigarh, Chennai, Guwahati, Hyderabad, Kolkata, and Mumbai. These Benches are headed by the President Chief Justice (Retd.) Ramalingam Sudhakar and comprises sixteen Judicial Members and nine Technical Members at different locations. Subsequently, more Benches at Cuttack, Jaipur, Kochi, Amravati, and Indore have been set up and new members have joined.

2. Nationalisation essentially meant that the government took over the ownership of certain private banks. The Congress government had nationalised 14 banks in (A) and then followed it up with nationalising another (B) in 1980.
Identify (A) and (B) respectively in the above excerpt.
(1) 1959; 6
(2) 1969; 6
(3) 1959; 8
(4) 1969; 8
(5) None of the above

Ans: 2
Solution:
The Congress government had nationalised 14 banks in 1969 and then followed it up by nationalising another 6 in 1980.
Nationalisation essentially meant that the government took over the ownership of certain private banks.

3. The Reserve Bank of India (RBI) directed the State Bank of India (SBI) to delete or amend more than 10 clauses in its original memorandum of association for setting up a subsidiary to provide operational support services to the country’s largest lender before approving the proposal.
Which of the following statements is/are not true about the SBI?
A. The government accepted the recommendation and nationalised the Imperial Bank of India on July 1, 1955, to establish the State Bank of India.
B. The State Bank of India (Subsidiary Banks) Act was passed in September 1959, and over time, 7 major State associated banks were merged with the main bank as subsidiary banks.
C. According to the State Bank of India Act, the bank had a statutory obligation to open 200 new branches in the rural, semi-urban and unbanked areas during the first five years of its existence or such extended period as might be permitted by the bank.
Choose the correct one:
(1) Only A and B
(2) Only B and C
(3) Only C
(4) Only A and C
(5) None of these

Ans: 2
Solution:
The government accepted the recommendation and nationalised the Imperial Bank of India on July 1, 1955, to establish the State Bank of India.
The State Bank of India (Subsidiary Banks) Act was passed in September 1959, and over time, eight major State associated banks were merged with the main bank as subsidiary banks. These were: Bank of Saurashtra, Bank of Patiala, Bank of Bikaner, Bank of Jaipur, Bank of Rajasthan, Bank of Indore, Bank of Baroda, Bank of Mysore, Hyderabad State Bank, and Travancore Bank.
According to the State Bank of India Act, the bank had a statutory obligation to open 400 new branches in the rural, semi-urban and unbanked areas during the first five years of its existence or such extended period as might be permitted by the bank.

4. Choose the Incorrect statements about India’s Export Promotion Schemes from the following options.
A. MEIS was introduced in the Foreign Trade Policy (FTP) 2015-20, under MEIS, the government provides duty benefits depending on the product and country.
B. Rewards under the MEIS scheme are payable as a percentage of realised free-on-board value (of 2%, 3% and 5%) and MEIS duty credit scrip can be transferred or used for payment of a number of duties including the basic customs duty.
C. SEIS was introduced in April 2015 for 5 Years under the Foreign Trade Policy of India 2015-2020.
D. RoDTEP was started in January 2021 as a replacement for the MEIS, which was not compliant with the rules of the World Trade Organisation.
(1) Only A, B and C
(2) Only B, C and D
(3) Only C and D
(4) Only A, C and D
(5) None of these

Ans: 5
Solution:
India’s Export Promotion Schemes
Merchandise Exports from India Scheme
MEIS was introduced in the Foreign Trade Policy (FTP) 2015-20, under MEIS, the government provides duty benefits depending on the product and country.
Rewards under the scheme are payable as a percentage of realized free-on-board value (of 2%, 3% and 5%) and MEIS duty credit scrip can be transferred or used for payment of a number of duties including the basic customs duty.
Service Exports from India Scheme
It was introduced in April 2015 for 5 Years under the Foreign Trade Policy of India 2015-2020.
Earlier, this Scheme was named as Served from India Scheme (SFIS Scheme) for Financial Year 2009-2014.
Under it, incentives are given by the Ministry of Commerce and Industry to Service Exporters based in India to promote the export of services from India.
Remission of Duties or Taxes on Export Product (RoDTEP)
It is a fully automated route for Input Tax Credit (ITC) in the GST (Goods and Service Tax) to help increase exports in India.
ITC is provided to set off tax paid on the purchase of raw materials, consumables, goods, or services that were used in the manufacturing of goods or services. This helps in avoiding double taxation and the cascading effect of taxes.
It was started in January 2021 as a replacement for the MEIS, which was not compliant with the rules of the World Trade Organisation.
Rebate of State and Central Taxes and Levies (RoSCTL)
Announced in March, 2019, RoSCTL was offered for embedded state and central duties and taxes that are not refunded through GST.
It was available only for garments and made-ups. It was introduced by the Ministry of Textiles.
Previously, it was Rebate for State Levies (ROSL).

5. Which one of the following is not the most likely measure the Government/RBI takes to stop the slide of the Indian rupee?
(1) Curbing imports of non-essential goods and promoting exports
(2) Encouraging Indian borrowers to issue rupee-denominated Masala Bonds
(3) Easing conditions relating to external commercial borrowing
(4) Following an expansionary monetary policy
(5) None of the above

Ans: 4
Solution:
Following an Expansionary Monetary Policy will infuse liquidity into the system. This results in the Strengthening of the dollar causing the rupee to slide. Thus, following a Contractionary Monetary Policy helps in controlling the slide of the Indian Rupee.

Mint StatiCA MCQs – Jan 3, 2023

1. The NCLAT was constituted under (A) of the Companies Act, 2013 to hear appeals against the orders of the National Company Law Tribunal (NCLT).
Identify (A) in the above excerpt.
(1) Section 157
(2) Section 235
(3) Section 360
(4) Section 435
(5) None of these

Ans: 5
Solution:
National Company Law Appellate Tribunal
The NCLAT was constituted under Section 410 of the Companies Act, 2013 to hear appeals against the orders of the National Company Law Tribunal (NCLT).
NCLT is a quasi-judicial body that adjudicates issues relating to companies.
It is also the appellate tribunal for orders passed by the NCLT(s) under Section 61 of the Insolvency and Bankruptcy Code (IBC), 2016, and for orders passed by the Insolvency and Bankruptcy Board of India (IBBI) under Sections 202 and 211 of the IBC.
Any person aggrieved by any order of the NCLAT may file an appeal to the Supreme Court.

2. The Supreme Court closed at least the legal chapter of the debate on the (A) demonetization policy by ruling _(B)___that the government was empowered to take the decision and that due process was followed.
Identify (A) and (B) respectively in the above excerpt.
(1) 2016; 3-2
(2) 2017; 4-1
(3) 2016; 4-1
(4) 2017; 3-2
(5) None of the above

Ans: 3
Solution:
The Supreme Court on Monday closed at least the legal chapter of the debate on the 2016 demonetization policy by ruling 4-1 that the government was empowered to take the decision and that due process was followed.
8th November 2022 marked the 6-year anniversary of demonetisation when Rs. 500 and Rs. 1,000 notes were withdrawn from the system in 2016.
Demonetisation is the act of stripping a currency unit of its status as legal tender. It occurs whenever there is a change of national currency and the current form or forms of money is pulled from circulation and retired, often to be replaced with new notes or coins.

3. Convertibility of rupee implies:
(1) being able to convert rupee notes into gold
(2) allowing the value of rupee to be fixed by market forces
(3) freely permitting the conversion of rupee to other currencies and vice versa
(4) developing an international market for currencies in India
(5) None of these

Ans: 3
Solution:
Convertibility of money implies a system where a country’s currency becomes convertible in foreign exchange and vice versa.
As far as the rupee is concerned, it is fully convertible in the current account, but partially in capital account.

4. The government has proposed to bring online real money gaming platforms under the IT (Intermediary Guidelines and Digital Media Ethics Code) Rules, 2021, which also regulates social media platforms.
Which of the following is the Nodal Ministry to govern online gaming in India?
(1) Ministry of Electronics and Information Technology
(2) Ministry of Information and Broadcasting
(3) Ministry of Education
(4) Ministry of Commerce and Industry
(5) None of these

Ans: 1
Solution:
The government has proposed to bring online real money gaming platforms under the IT (Intermediary Guidelines and Digital Media Ethics Code) Rules, 2021, which also regulates social media platforms. The new rules could hurt foreign betting firms while boosting the business of Indian fantasy sports and other gaming companies, experts said.
The ministry of electronics and IT (MeitY), which became the nodal ministry to govern online gaming on 23 December, released a draft for public consultation. The final amendment to the IT rules after industry consultation will be notified by April, the ministry said.

5. Which of the following statements is not true about the Uday Kotak Committee on Corporate Governance?
A. It was constituted in 2018 for improving standards concerning corporate governance of listed companies in India.
B. It recommended a reduction in the maximum number of listed entity directorships from 10 to 8 by April 01, 2019 and to 7 by April 1, 2020.
C. It recommended mandatory disclosure of consolidated quarterly results with effect from FY21
(1) Only C
(2) Only A and C
(3) Only B and C
(4) Only A and B
(5) All of the above

Ans: 2
Solution:
Uday Kotak Committee
It was constituted in 2017 under the chairmanship of Uday Kotak.
Primary objective: Improving standards concerning corporate governance of listed companies in India.
Key recommendations of the committee
Proposals accepted without modifications
Following are proposals accepted by SEBI without any modifications:
Reduction in the maximum number of listed entity directorships from 10 to 8 by April 01, 2019 and to 7 by April 1, 2020
Expanding the eligibility criteria for independent directors
Enhanced role of the audit committee, nomination and remuneration committee and risk management committee
Disclosure of utilization of funds from QIP/preferential issue
Disclosures of auditor credentials, audit fee, reasons for the resignation of auditors
Disclosure of expertise/skills of directors
Enhanced disclosure of related party transactions (RPTs)
Mandatory disclosure of consolidated quarterly results with effect from FY20
Enhanced obligations on the listed entities with respect to subsidiaries
Secretarial Audit to be mandatory for listed entities and their material unlisted subsidiaries
Proposals accepted with modifications
Minimum six directors in the top 1,000 listed entities by market capitalization by April1, 2019 and in the top 2000 listed entities, by April 1, 2020
At least one woman independent director in the top 500 listed entities by market capitalization by April 1, 2019 and in the top 1000 listed entities, by April 1, 2020
Separation of CEO/MD and Chairperson (to be initially made applicable to the top 500 listed entities by market capitalization w.e.f. April 1, 2020)
Separation of CEO/MD and Chairperson (to be initially made applicable to the top 500 listed entities by market capitalization w.e.f. April 1, 2020)
Top 100 entities to hold AGMs within 5 months after the end of FY 2018-19 i.e. by August 31, 2019
Webcast of AGMs will be compulsory for top 100 entities by market capitalization a w.e.f. FY19
Shareholder approval for Royalty/brand payments to related party exceeding 2% of consolidated turnover (instead of the proposed 5%)

Mint StatiCA MCQs – Jan 2, 2023

1. After recommending sweeping changes to India’s bankruptcy framework and competition regulation, the Parliament’s influential standing committee on finance may examine the issue of state finances, the working of market and banking regulators, and the menace of cybercrimes, the panel’s chairperson, _(A), said.
Who is the Chairperson of the Standing Committee on Finance depicted as (A) in the above excerpt?
(1) Adhir Ranjan Chowdhury
(2) Girish Bapat
(3) Santosh Gangwar
(4) Jayant Sinha
(5) Rakesh Singh1.

Ans: 4
Solution:
After recommending sweeping changes to India’s bankruptcy framework and competition regulation, the Parliament’s influential standing committee on finance may examine the issue of state finances, the working of market and banking regulators, and the menace of cybercrimes, the panel’s chairperson, Jayant Sinha, said.

2. Central and state governments collected ₹1.5 trillion in goods and services tax (GST) in December, up 15% from a year earlier, on higher sales of automobiles and a pick-up in services activity. (A) says that GST Council is to be formed by The President to administer & govern GST. Its Chairman is the (B)_ of India with ministers nominated by the state governments as its members.
Identify (A) and (B) respectively in the above passage.
(1) Art. 279 A; Union Finance Minister
(2) Art. 275 A; Prime Minister
(3) Art. 279 A; Prime Minister
(4) Art. 275 A; Union Finance Minister
(5) None of the above

Ans: 1
Solution:
Article 279A – GST Council to be formed by The President to administer & govern GST. Its Chairman is the Union Finance Minister of India with ministers nominated by the state governments as its members.
The council is devised in such a way that the centre will have 1/3rd of voting power and the states will have 2/3rd.
The decisions will be taken by a 3/4th majority.

3. The government is considering raising interest equalization or subsidy benefits extended to small and medium exporters in the annual budget for 2023-24 to relieve some of the interest rate burden on them due to a tightening monetary policy. The Micro, Small and Medium Enterprises (MSMEs) sector is an important pillar of the Indian economy as it contributes greatly to the growth of the Indian economy with a vast network contributing about _ to manufacturing output.
Fill in the Blanks.
(1) 20
(2) 30
(3) 45
(4) 60
(5) 75

Ans: 3
Solution:
The Micro, Small and Medium Enterprises (MSMEs) sector is an important pillar of the Indian economy as it contributes greatly to the growth of the Indian economy with a vast network contributing about 45% to manufacturing output.
MSMEs provide about 110 million jobs which is 22-23% of the total employment in India. It is next highest to Agriculture.

4. Corporate tax collections exceeded 3% of the GDP after a gap of two years in 2021-22, reflecting overall improvement in the profitability of India Inc, propelled by an increase in demand for goods and services.
Which of the following statements is/are not true about Corporate Tax?
A. Also known as Corporation Tax, corporate tax is the tax imposed by the Government of India on the Gross income that corporate enterprises make from their businesses
B. Corporate Income Tax is a Direct Tax
C. The tax is imposed at a specific rate as per the provisions of the Income Tax Act, of 1961
Choose the correct option:
(1) Only A and B
(2) Only A
(3) Only B and C
(4) Only A and C
(5) All of the above

Ans: 2
Solution:
Also known as Corporation Tax, corporate tax is the tax imposed by the Government of India on the net income or profit that corporate enterprises make from their businesses.
It is a tax imposed on the net income of the company.
Corporate Income Tax is a Direct Tax.
The tax is imposed at a specific rate as per the provisions of the Income Tax Act, of 1961.

5. The Reserve Bank of India (RBI) penalized more than 180 erring cooperative banks in 2022, the highest since the government vested more powers with the regulator two years ago.
Which of the following statements is not true about the Reserve Bank of India?
(1) An amendment to RBI Act, 1934, was made in May 2016, providing the statutory basis for the implementation of the flexible inflation targeting framework.
(2) The Reserve Bank of India (RBI) governor inaugurated the Reserve Bank Innovation Hub (RBIH) in Bengaluru.
(3) Under section 11 of the RBI Act, the central government may from time to time give such directions to the RBI as it may, after consultation with the Governor of the Bank, consider necessary in the public interest.
(4) The Reserve Bank of India was established on April 1, 1935, in accordance with the provisions of the Reserve Bank of India Act, of 1934.
(5) None of the above

Ans: 3
Solution:
The Reserve Bank of India was established on April 1, 1935, in accordance with the provisions of the Reserve Bank of India Act, of 1934.
An amendment to RBI Act, 1934, was made in May 2016, providing the statutory basis for the implementation of the flexible inflation targeting framework.
The Reserve Bank of India (RBI) governor inaugurated the Reserve Bank Innovation Hub (RBIH) in Bengaluru.
Under section 7 of the RBI Act, the central government may from time to time give such directions to the RBI as it may, after consultation with the Governor of the Bank, consider necessary in the public interest.

Mint StatiCA MCQs – Dec 30, 2022

1. Financial Stability Report (FSR) is a biannual report released by the Reserve Bank of India (RBI). The FSR reflects the collective assessment of the Sub-Committee of the Financial Stability and Development Council (FSDC) on risks to financial stability and the financial system’s resilience. As per the report, the country’s financial system remains stable despite weakening domestic growth despite the risks from global economic uncertainties and geopolitical developments.
Who is the Chairperson of the Sub-Committee of the Financial Stability and Development Council (FSDC)?
(1) Union Finance Minister
(2) Union Finance Secretary
(3) RBI Governor
(4) Vice Chairperson, NITI Aayog
(5) Secretary, Department of Financial Services

Ans: 3
Solution:
The Reserve Bank releases the Financial Stability Report (FSR) biannually, which reflects the collective assessment of the Sub-Committee of the Financial Stability and Development Council (FSDC) on risks to financial stability and the resilience of the financial system.
FSDC sub-committee is headed by the Governor of RBI.

2. India’s current account deficit (CAD) widened to a nine-year high of 4.4% of gross domestic product (GDP) in the three months to September, up from 2.2% in the June quarter, primarily due to a widening goods trade deficit and an increase in net outgo under investment income.
With reference to the Balance of Payments, which of the following constitutes/constitute the Current Account?
A. Balance of trade
B. Foreign assets
C. Balance of invisibles
D. Special Drawing Rights
Choose the correct option:
(1) Only A
(2) Only B and C
(3) Only A and C
(4) Only A and B
(5) None of the above

Ans: 3
Solution:
Balance of Payments (BoP)
BoP of a country can be defined as a systematic statement of all economic transactions of a country with the rest of the world during a specific period, usually one year.
Purposes of Calculation of BoP
Reveals the financial and economic status of a country.
Can be used as an indicator to determine whether the country’s currency value is appreciating or depreciating.
Helps the Government to decide on fiscal and trade policies.
Provides important information to analyse and understand the economic dealings of a country with other countries.
Components of BoP
For preparing BoP accounts, economic transactions between a country and the rest of the world are grouped under – Current account, Capital account and Errors and Omissions. It also shows changes in Foreign Exchange Reserves.
Current Account
It shows the export and import of visibles (also called merchandise or goods – represent trade balance) and invisibles (also called non-merchandise).
Invisibles include services, transfers, and income.
Capital Account
It shows the capital expenditure and income of a country.
It gives a summary of the net flow of both private and public investment into an economy.
External Commercial Borrowing (ECB), Foreign Direct Investment, Foreign Portfolio Investment, etc form a part of the capital account.
Errors and Omissions
Sometimes the balance of payments does not balance. This imbalance is shown in the BoP as errors and omissions. It reflects the country’s inability to record all international transactions accurately.
Changes in Foreign Exchange Reserves
Movements in the reserves comprise changes in the foreign currency assets held by the Reserve Bank of India (RBI) and also in Special Drawing Rights (SDR)balances.
Overall, the BoP account can be a surplus or a deficit. If there is a deficit, then it can be bridged by taking money from the Foreign Exchange (Forex) Account.
If the reserves in the forex account are falling short, then this scenario is referred to as a BoP crisis.

3. The National Action Plan on Climate Change (NAPCC) was launched in (A) by the Prime Minister’s Council on Climate Change. It aims to create awareness among the representatives of the public, different agencies of government, scientists, industry, and communities on the threat posed by climate change and the steps to counter it.
Identify the year (A) in the above excerpt.
(1) 2000
(2) 2004
(3) 2006
(4) 2008
(5) 2012

Ans: 4
Solution:
The National Action Plan on Climate Change (NAPCC) was launched in 2008 by the Prime Minister’s Council on Climate Change.
It aims at creating awareness among the representatives of the public, different agencies of government, scientists, industry, and communities on the threat posed by climate change and the steps to counter it.
There are 8 national missions forming the core of the NAPCC which represent multi-pronged, long term and integrated strategies for achieving key goals in climate change. These are-
1.National Solar Mission
2.National Mission for Enhanced Energy Efficiency
3.National Mission on Sustainable Habitat
4.National Water Mission
5.National Mission for Sustaining the Himalayan Ecosystem
6.National Mission for A Green India
7.National Mission for Sustainable Agriculture
8.National Mission on Strategic Knowledge for Climate Change

4. Macro-stress tests help central banks assess the resilience of bank balance sheets to unexpected shocks from the macroeconomic environment. In its financial stability report, RBI said that falling slippages, an increase in write-offs and the continuous improvement in loan growth have led to banks’ gross non-performing asset (NPA) ratio falling to a seven-year low of 5% in September.
Which of the following statements is/are true about the Slippage Ratio?
(1) Slippage ratio is defined as the ratio of increase in NPAs during the year with respect to standard advances at the beginning of the year.
(2) Slippage Ratio depicts the percentage of funds that a bank sets aside for losses due to bad debts.
(3) Slippage Ratio is the ratio of a bank’s capital in relation to its risk-weighted assets and current liabilities.
(4) Slippage Ratio is the proportion of current account and savings account deposits in the total deposits of the bank.
(5) All of the above

Ans: 1
Solution:
Slippage ratio is defined as the ratio of increase in NPAs during the year with respect to standard advances at the beginning of the year.

5. The number of Americans filing new claims for unemployment benefits edged higher last week but remain in a range indicating the US job market remains tight, even as the Federal Reserve works to cool demand for labour as part of its bid to lower inflation.
Which of the following options depicts the causes of Cost-Push Inflation?
(1) Increase in price of inputs
(2) Hoarding and Speculation of commodities
(3) Defective Supply chain
(4) Increase in indirect taxes
(5) All of the above

Ans: 5
Solution:
In economics, inflation (or less frequently, price inflation) is a general rise in the price level of an economy over a period of time. When the general price level rises, each unit of currency buys fewer goods and services; consequently, inflation reflects a reduction in the purchasing power per unit of money – a loss of real value in the medium of exchange and a unit of account within the economy.
As per RBI, an inflation target of 4 per cent with a +/-2 per cent tolerance band, is appropriate for the next five years (2021-2025).
Cost-Push Inflation
This type of inflation is caused due to various reasons such as:
Increase in price of inputs
Hoarding and Speculation of commodities
Defective Supply chain
Increase in indirect taxes
Depreciation of Currency
Crude oil price fluctuation
Defective food supply chain
Low growth of the Agricultural sector
Food Inflation
Interest rates increased by RBI

Mint StatiCA MCQs – Dec 29, 2022

1. Which of the following statements is/are true about the Union Budget in India?
A. According to Article 110 of the Indian Constitution, the Union Budget of a year is referred to as the Annual Financial Statement (AFS).
B. It is a statement of the estimated receipts and expenditure of the Government in a Financial Year (which begins on 1st April of the current year and ends on 31st March of the following year).
C. The Budget Division of the Department of Economic Affairs in the Ministry of Finance is the nodal body responsible for preparing the Budget.
D. The first Budget of Independent India was presented in 1948.
Choose the correct option:
(1) Only A and D
(2) Only B and C
(3) Only A, C and D
(4) Only B
(5) All of the above

Ans: 2
Solution:
According to Article 112 of the Indian Constitution, the Union Budget of a year is referred to as the Annual Financial Statement (AFS).
It is a statement of the estimated receipts and expenditures of the Government in a Financial Year (which begins on 1st April of the current year and ends on 31st March of the following year).
Overall, the Budget contains:
Estimates of revenue and capital receipts,
Ways and means to raise the revenue,
Estimates of expenditure,
Details of the actual receipts and expenditures of the closing financial year and the reasons for any deficit or surplus in that year, and
The economic and financial policy of the coming year, i.e., taxation proposals, prospects of revenue, spending programme and introduction of new schemes/projects.
In Parliament, the Budget goes through six stages:
Presentation of Budget.
General discussion.
Scrutiny by Departmental Committees.
Voting on Demands for Grants.
Passing an Appropriation Bill.
Passing of Finance Bill.
The Budget Division of the Department of Economic Affairs in the Ministry of Finance is the nodal body responsible for preparing the Budget.
The first Budget of Independent India was presented in 1947.

2. The Central Board of Indirect Taxes and Customs (CBIC) has allowed tax officers to recover reduced dues from bankrupt businesses as decided by the National Company Law Tribunal (NCLT) while deciding on the future of a distressed company.
Which of the following statements is/are not true about the NCLT?
A. The National Company Law Tribunal (NCLT) is a quasi-judicial body in India that adjudicates issues relating to companies in India.
B. The tribunal was established based on the recommendation of the V. Balakrishna Eradi committee on the law relating to insolvency and the winding up of companies.
C. The NCLT has eleven benches, two at New Delhi (one being the principal bench) and one each at Ahmedabad, Allahabad, Bengaluru, Chandigarh, Chennai, Bhubaneswar, Hyderabad, Kolkata, and Mumbai.
Choose the correct option:
(1) Only C
(2) Only B and C
(3) Only A and C
(4) Only A and B
(5) None of the above

Ans: 1
Solution:
The National Company Law Tribunal (NCLT) is a quasi-judicial body in India that adjudicates issues relating to companies in India.
Established: 1st June, 2016 (Companies Act, 2013)
The tribunal was established based on the recommendation of the V. Balakrishna Eradi committee on the law relating to insolvency and the winding up of companies.
The NCLT has eleven benches, two at New Delhi (one being the principal bench) and one each at Ahmedabad, Allahabad, Bengaluru, Chandigarh, Chennai, Guwahati, Hyderabad, Kolkata, and Mumbai.

3. India and Australia have signed a historic interim Economic Cooperation And Trade Agreement (IND-AUS ECTA), which will give a fillip to India’s exports in the textiles, leather, gems, and jewellery sector Down Under. For India, the ECTA with Australia is the first agreement with a large developed economy of the world after more than a decade. Australia is also the third OECD country after (A) and (B) with which India has signed a Free Trade Agreement (FTA).
Identify (A) and (B) respectively in the above excerpt.
(1) USA; Japan
(2) USA; South Korea
(3) Japan; South Korea
(4) Germany; Japan
(5) USA; Germany

Ans: 3
Solution:
India and Australia have signed a historic interim Economic Cooperation And Trade Agreement (IND-AUS ECTA), which will give a fillip to India’s exports in the textiles, leather, gems, and jewellery sector Down Under.
For India, the ECTA with Australia is the first agreement with a large developed economy of the world after more than a decade. Australia is also the third OECD country after Japan and South Korea with which India has signed a Free Trade Agreement (FTA).

4. Despite the Narendra Modi government’s high-octane push to boost manufacturing through the ‘Make in India’ initiative, foreign investors continue to chase bets in the services sector. Launched in _, Make in India aims to transform the country into a leading global manufacturing and investment destination.
Fill in the Blanks.
(1) 2014
(2) 2015
(3) 2016
(4) 2017
(5) None of the above

Ans: 1
Solution:
Launched in 2014, Make in India aims to transform the country into a leading global manufacturing and investment destination.
Objectives
To attract foreign investment for new industrialisation and develop the already existing industry base in India to surpass that of China.
The target of an increase in manufacturing sector growth to 12-14% per annum over the medium term.
To increase the share of the manufacturing sector in the country’s Gross Domestic Product from 16% to 25% by 2022.
To create 100 million additional jobs by 2022.
To promote export-led growth.

5. Office of the Economic Adviser (OEA) is an attached office of the Department for Promotion of Industry and Internal Trade, (A)_.
Identify the Ministry (A) in the above excerpt.
(1) Ministry of Commerce and Industry
(2) Ministry of Finance
(3) Ministry of External Affairs
(4) Ministry of Heavy Industries
(5) None of these

Ans: 1
Solution:
Office of the Economic Adviser (OEA) is an attached office of the Department for Promotion of Industry and Internal Trade, Ministry of Commerce & Industry.
The main functions of the Office of Economic Adviser include, inter alia the following:
Policy Functions
Economic policy inputs on industrial development.
Rendering advice relating to formulation of Industrial Policy, Foreign Trade Policy with respect to the industrial sector in general with a thrust on manufacturing, issues relating to bilateral and multilateral trade, as well as taxes and duties related to the industry, including but not restricted to safeguard and anti-dumping duties.
Analysis of trends of industrial production and growth.
Examination of multilateral and bilateral issues and processing Policy Notes with economic implications referred to the Office.
Statistical Functions
Compiling and releasing monthly Wholesale Price Indices
Compiling and releasing monthly Index of Core Industries Production
Developing other Indices on an experimental basis, e.g., select business service price indices
Supervising as a ‘source agency’, a compilation of monthly production statistics for identified industrial items, their validation, and onward transmission for computation of the monthly Index of Industrial Production (IIP) by the Central Statistics Office.
Monthly Statistical compilation of macro indicators (secondary information).

Mint StatiCA MCQs – Dec 28, 2022

1. Which of the following statements is/are true about the Insolvency and Bankruptcy Code?
A. It was enacted in 2015, against the backdrop of mounting non-performing loans, with a view to establishing a consolidated framework for insolvency resolution of corporations, partnership firms and individuals in a time-bound manner.
B. The IBC sets out three classes of persons who can trigger the corporate insolvency resolution process (CIRP) – financial creditors, operational creditors, and corporate debtors.
C. The Supreme Court in Kridhan Infrastructure Vs Venketesan Sankaranarayan, observed that the insolvency resolution should not suffer from an indefinite delay in complete abeyance of the timelines fixed under the IBC.
Choose the correct option:
(1) Only A
(2) Only B and C
(3) Only A and C
(4) Only B
(5) All of the above

Ans: 2
Solution:
It was enacted in 2016, against the backdrop of mounting non-performing loans, with a view to establishing a consolidated framework for insolvency resolution of corporations, partnership firms and individuals in a time-bound manner
Companies have to complete the entire insolvency exercise within 180 days under IBC and the deadline may be extended if the creditors do not raise objections to the extension. Presently, the maximum time limit is 330 days.
It seeks to tackle the non-performing asset (NPA) problem in two ways:
Behavioural change on part of the debtors to ensure sound business decision-making and prevent business failures is encouraged.
It envisages a process through which financially ailing corporate entities are put through a rehabilitation process and brought back up on their feet.
The IBC sets out three classes of persons who can trigger the corporate insolvency resolution process (CIRP) – financial creditors, operational creditors, and corporate debtors.
The most important aspect of the IBC is the timeliness of insolvency resolution.
The Supreme Court in Kridhan Infrastructure Vs Venketesan Sankaranarayan, observed that the insolvency resolution should not suffer from an indefinite delay in complete abeyance of the timelines fixed under the IBC.

2. While economic reforms, including the implementation of goods and services tax and insolvency code, have positively impacted India’s economy, they have been overshadowed by the pandemic, balance sheet repair for banks and companies, and external shocks such as commodity price increases and interest rate hikes, chief economic adviser in the finance ministry V. Anantha Nageswaran said. However, these reforms have set the stage for sustained 6.5-7% growth in the coming years, Nageswaran said in an interview.
Which of the following statements is/are not true about Economic Growth?
A. Economic Growth relates to the growth of human capital, decrease in inequality figures, and structural changes that improve the quality of life of the population.
B. It indicates both quantitative and qualitative changes in an economy.
C. It reflects the growth of national or per capita income
Choose the correct option:
(1) Only A and B
(2) Only B and C
(3) Only A and C
(4) Only C
(5) All of the above

Ans: 1
Solution:

3. The push provided by Jan Dhan, Aadhaar and mobile—often called the JAM trinity—has resulted in increased access to banking services to the unserved and the underserved sections. The overdraft (OD) limit under the Pradhan Mantri Jan Dhan Yojana doubled from Rs (A)_ to (B)__.
Identify (A) and (B) respectively in the above excerpt.
(1) 1000; 2000
(2) 2000; 4000
(3) 3000; 6000
(4) 4000; 8000
(5) 5000; 10000

Ans: 5
Solution:
JAM (short for Jan Dhan-Aadhaar-Mobile) trinity refers to the government of India’s initiative to link Jan Dhan accounts, mobile numbers and Aadhaar cards of Indians to plug the leakages of government subsidies.
Overdraft (OD) limit under the Pradhan Mantri Jan Dhan Yojana doubled from Rs 5,000/- to Rs 10,000/-; OD up to Rs 2,000/- (without conditions).

4. India is planning to push for a waiver of intellectual property rights (IPR) for technologies related to green energy and energy transition in a bid to bridge the technology gap across G20 countries. India has assumed the charge of the G20 presidency and the ____summit will be held in India in 2023.
Fill in the Blanks.
(1) 13th
(2) 14th
(3) 17th
(4) 18th
(5) None of the above

Ans: 4
Solution:
The 17th annual summit of G-20 was hosted by the Indonesian G20 presidency in Bali under the theme ‘Recover Together, Recover Stronger’.
Now, India has assumed the charge of the G20 presidency, and the 18th summit will be held in India in 2023.

5. A review report of the Centre’s rural schemes has recommended more permissible works and higher average person-days in the flagship job scheme MGNREGA, while offering a slew of steps in other welfare programmes. The MGNREGA act mandates Gram Sabhas to recommend the works that are to be undertaken and at least _(A)% of the works must be executed by them.
Identify (A) in the above excerpt.
(1) 25
(2) 33
(3) 50
(4) 60
(5) None of these

Ans: 3
Solution:
MGNREGA is one of the largest work guarantee programmes in the world launched in 2005 by the Ministry of Rural development.
The primary objective of the scheme is to guarantee 100 days of employment in every financial year to adult members of any rural household willing to do public work-related unskilled manual work.
The act mandates Gram Sabhas to recommend the works that are to be undertaken and at least 50% of the works must be executed by them.

Mint StatiCA MCQs – Dec 27, 2022

1. Benchmark indices rebounded, driven by a short squeeze and value buying, reclaiming key psychological levels amid thin cash trading. Monday’s rebound led to investor wealth rising by ₹5.58 trillion, offering some relief after nearly ₹16 trillion of wealth was lost last week due to the resurgence of covid-19 in China. The Nifty and Sensex rose 1.2% each to close at 18,014.6 and 60,566.42, reclaiming the psychologically important levels of 18,000 and 60,000. The NIFTY is a benchmark-based index and the NSE’s flagship, showcasing the top____(A)__ equity stocks listed on the stock exchange.
Identify (A) in the above passage.
(1) 20
(2) 30
(3) 50
(4) 100
(5) 200

Ans: 3
Solution:
The National Stock Exchange introduced the NIFTY market index. It is a combination of the words National Stock Exchange and Fifty, which was coined by the NSE on April 21, 1996. The NIFTY 50 is a benchmark-based index and the NSE’s flagship, showcasing the top 50 equity stocks listed on the stock exchange.

2. The RoDTEP scheme would refund to exporters the embedded central, state and local duties or taxes that were not being rebated or refunded and, therefore, placed India’s exports at a disadvantage. It was started in _(A) as a replacement for the (B), which was not compliant with the rules of the World Trade Organisation.
Identify (A) and (B) respectively in the above excerpt.
(1) 2015; Service Export from India Scheme
(2) 2017; Duty-free Import Authorisation Scheme
(3) 2019; Merchandise Exports from India Scheme
(4) 2021; Service Export from India Scheme
(5) None of these

Ans: 5
Solution:
The RoDTEP scheme would refund to exporters the embedded central, state, and local duties or taxes that were not being rebated or refunded and, therefore, placed India’s exports at a disadvantage.
The rebate under the scheme would not be available in respect of duties and taxes already exempted or remitted or credited.
It was started in January 2021 as a replacement for the Merchandise Export from India Scheme (MEIS), which was not compliant with the rules of the World Trade Organisation.
The tax refund rates range from 0.5% to 4.3% for various sectors.
The rebate will have to be claimed as a percentage of the Freight On Board value of exports.
Rebates will be issued in the form of a transferable duty credit/electronic scrip (e-scrip) which will be maintained in an electronic ledger by the Central Board of Indirect Taxes and Customs (CBIC).

3. The National Green Tribunal has extended the stay on a Union Ministry of Environment, Forests and Climate Change notification modifying the Environmental Impact Assessment (EIA) conditions for certain construction projects.
Which of the following statements is/are not true about the National Green Tribunal?
A. It is a specialised body set up under the National Green Tribunal Act (2008) for effective and expeditious disposal of cases relating to environmental protection and conservation of forests and other natural resources.
B. With the establishment of the NGT, India became the third country in the world to set up a specialised environmental tribunal, only after Australia and USA and the first developing country to do so.
C. NGT is mandated to make disposal of applications or appeals finally within 6 months of the filing of the same.
D. The NGT has five places of sittings, New Delhi is the Principal place of sitting and Ahmedabad, Bhopal, Kolkata, and Chennai are the other four.
Choose the correct option:
(1) Only A, B and D
(2) Only C and D
(3) Only A and B
(4) Only B, C and D
(5) All of the above

Ans: 1
Solution:
National Green Tribunal (NGT)
It is a specialised body set up under the National Green Tribunal Act (2010) for effective and expeditious disposal of cases relating to environmental protection and conservation of forests and other natural resources.
With the establishment of the NGT, India became the third country in the world to set up a specialised environmental tribunal, only after Australia and New Zealand, and the first developing country to do so.
NGT is mandated to make disposal of applications or appeals finally within 6 months of the filing of the same.
The NGT has five places of sittings, New Delhi is the Principal place of sitting and Bhopal, Pune, Kolkata, and Chennai are the other four.

4. The 15th Conference of Parties (COP) to the Convention on Biological Diversity (CBD) ended on a high note with nearly 200 countries, including India, adopting a historic deal to preserve nature and reverse damage to ecosystems after four years of fraught negotiations. At the (A) Earth Summit in Rio de Janeiro, world leaders agreed on a comprehensive strategy for “sustainable development” — meeting our needs while ensuring that we leave a healthy and viable world for future generations. One of the key agreements adopted at Rio was the Convention on Biological Diversity. The Convention on Biological Diversity is the international legal instrument for “the conservation of biological diversity, the sustainable use of its components and the fair and equitable sharing of the benefits arising out of the utilization of genetic resources” that has been ratified by (B) nations.
Identify (A) and (B) respectively in the above passage.
(1) 1988; 188
(2) 1992; 192
(3) 1988; 196
(4) 1992; 196
(5) None of the above

Ans: 4
Solution:
At the 1992 Earth Summit in Rio de Janeiro, world leaders agreed on a comprehensive strategy for “sustainable development” — meeting our needs while ensuring that we leave a healthy and viable world for future generations. One of the key agreements adopted at Rio was the Convention on Biological Diversity.
The Convention on Biological Diversity is the international legal instrument for “the conservation of biological diversity, the sustainable use of its components and the fair and equitable sharing of the benefits arising out of the utilization of genetic resources” that has been ratified by 196 nations.

5. Stock Exchange is a physically existing institutionalised set-up where instruments of the security stock market (shares, bonds, debentures, securities, etc.) are traded.
Which of the following is/are major functions of a stock exchange?
(1) Makes a floor available to the buyers and sellers of stocks. It is the single most important institution in the secondary market for securities
(2) Makes available the prices of trading as an important piece of information to the investors
(3) By following institutionalised rules and procedures, ensures that the participants in the stock market live up to their commitments
(4) Passes updated information to the enlisted companies about their present stockholders (so that they can pass on dividends etc., to them)
(5) All of the above

Ans: 5
Solution:
Stock Exchange
It is a physically existing institutionalised set-up where instruments of the security stock market (shares, bonds, debentures, securities, etc.) are traded
It serves the following major functions:
Makes a floor available to the buyers and sellers of stocks. It is the single most important institution in the secondary market for securities
Makes available the prices of trading as an important piece of information to the investors
By following institutionalised rules and procedures, ensures that the participants in the stock market live up to their commitments
Passes updated information to the enlisted companies about their present stockholders (so that they can pass on dividends etc., to them)
By publishing its ‘Index’, it fulfils the purpose of projecting the moods of the stock market

Mint StatiCA MCQs – Dec 26, 2022

1. Under Section 4(2) of the FRBM Act, the Centre can exceed the annual fiscal deficit target by citing certain grounds. Which of the following depicts the grounds as mentioned in the act?
A. National security, war
B. National calamity
C. Collapse of agriculture
D. Structural reforms
E. Decline in real output growth of a quarter by at least 2 percentage points below the average of the previous four quarters.
(1) Only A, B and C
(2) Only B, C and E
(3) Only A, B, D and E
(4) Only A, B, C and D
(5) All of the above

Ans: 4
Solution:
Escape Clause
Under Section 4(2) of the FRBM Act, the Centre can exceed the annual fiscal deficit target by citing certain grounds.
The grounds include
National security, war
National calamity
Collapse of agriculture
Structural reforms
Decline in real output growth of a quarter by at least three percentage points below the average of the previous four quarters.

2. SENSEX is the benchmark index of the Bombay Stock Exchange (BSE) in India. The SENSEX index is made up of (A)_of the BSE’s largest and most actively traded stocks, and it serves as a gauge for the Indian economy. It is a market capitalization-weighted and float-adjusted. The Sensex is India’s oldest stock index, having been established in (B)__ by Standard & Poor’s (S&P).
Identify (A) and (B) respectively in the above passage.
(1) 30; 1986
(2) 50; 1992
(3) 30; 1992
(4) 50; 1986
(5) None of these

Ans: 1
Solution:
SENSEX is the benchmark index of the Bombay Stock Exchange (BSE) in India. The SENSEX index is made up of 30 of the BSE’s largest and most actively traded stocks, and it serves as a gauge for the Indian economy. It is a market capitalization-weighted and float-adjusted. The Sensex is India’s oldest stock index, having been established in 1986 by Standard & Poor’s (S&P).

3. When a company has some proven numbers, venture capitalists often invest in the _.
Fill in the Blanks.
(1) Seed Stage
(2) Growth Stage
(3) Expansion Stage
(4) Early Stage
(5) None of these

Ans: 2
Solution:
Venture Capital refers to the investments or capital that entrepreneurs obtain from affluent investors, and Venture Capitalists refer to the investors.
They are always motivated by return on investment (ROI).
When a company has some proven numbers, venture capitalists often invest in the growth stage.

4. The union budget for FY24 is likely to extend the subsidy of ₹200 per cooking gas cylinder for 12 cylinders a year for another fiscal for Ujjwala beneficiaries. The scheme provides financial support of Rs _(A) for each LPG connection to the BPL households.
Identify (A) in the above excerpt.
(1) 400
(2) 800
(3) 1200
(4) 1600
(5) 2000

Ans: 4
Solution:
PMUY-I
Launched in May 2016 to provide LPG (liquefied petroleum gas) connections to poor households.
PMUY-II
It is aimed to provide maximum benefit to the migrants who live in other states and find it difficult to submit address proof.
Now they will only have to give “Self-Declaration” to avail the benefit.
The scheme provides financial support of Rs 1600 for each LPG connection to the BPL households.
Nodal Ministry: Ministry of Petroleum and Natural Gas (MoPNG)
Along with a deposit-free LPG connection, Ujjwala 2.0 will provide the first refill and a hotplate free of cost to the beneficiaries.

5. Riding on the back of a 26% surge in tax collections, the government is set to unveil the next set of reforms in tax administration by pruning the number of forms available for filing income tax returns (ITR) to improve taxpayers’ experience and reduce the time taken to file returns.
Which of the following is/are not a Direct Tax in India?
(1) Income Tax
(2) Goods and Services Tax
(3) Customs Duty
(4) Corporation Tax
(5) Both 2 and 3

Ans: 5
Solution:
Income Tax and Corporation Tax are direct taxes.
Goods and Services Tax and Customs Duty are indirect taxes.

Mint StatiCA MCQs – Dec 23, 2022

1. A Disinvestment Commission was established in _______ by the government of India, to carefully evaluate the withdrawal of the public sector from non-core, non-strategic areas and assure workers of job security and opportunities for retraining and re-employment. It recommended the sale of equities or the outright sale of several PSEs including Air India.

Fill in the Blanks.

(1) 1948

(2) 1975

(3) 1996

(4) 2002

(5) 2008

Ans: 3

Solution:

A Disinvestment Commission was established in 1996 by the government of India, to carefully evaluate the withdrawal of the public sector from non-core, non-strategic areas and assure workers of job security and opportunities for retraining and re-employment. It recommended the sale of equities or the outright sale of several PSEs including Air India.

2. The UPA manifesto in 2004 said it would take up privatization selectively. Unlike what the NDA had done, there would be no disinvestment just to raise funds to meet short-term targets. Proceeds of disinvestment would be used for designated social welfare programs. In pursuance of this, the government formed a National Investment Fund (NIF) in ____(A)____, to which the funds raised from disinvestment were channelled. The purpose of the Fund, managed by professional investment managers, was to utilize _____(B)____ of the proceeds to fund social welfare schemes in education, health, and employment.

Identify (A) and (B) respectively in the above passage.

(1) 2005; 50%

(2) 2005; 75%

(3) 2007; 50%

(4) 2007; 75%

(5) None of these

Ans: 2

Solution:

The government was not keen on treading the strategic sales route. The UPA manifesto in 2004 said it would take up privatization selectively. Unlike what the NDA had done, there would be no disinvestment just to raise funds to meet short-term targets. Proceeds of disinvestment would be used for designated social welfare programs.

In pursuance of this, the government formed a National Investment Fund (NIF) in 2005, to which the funds raised from disinvestment were channelled. The purpose of the Fund, managed by professional investment managers, was to utilize 75% of the proceeds to fund social welfare schemes in education, health, and employment. But due to the financial crisis of 2008-09, and later a drought, this was put on hold for 3 years, and later in 2013, it was restructured to provide flexibility in using the Fund.

3. The Monopolies and Restrictive Trade Practices Act, 1969 (MRTP Act) was repealed and replaced by the Competition Act, 2002, on the recommendations of the ______(A)_____.

Identify the Committee depicted as (A) in the above excerpt.

(1) Bimal Jalan Committee

(2) Tarapore Committee

(3) Raghavan Committee

(4) Malhotra Committee

(5) None of these

Ans: 3

Solution:

Competition Commission of India (CCI) is a statutory body of the Government of India responsible for enforcing the Competition Act, 2002, it was duly constituted in March 2009.

The Monopolies and Restrictive Trade Practices Act, 1969 (MRTP Act) was repealed and replaced by the Competition Act, 2002, on the recommendations of the Raghavan committee.

The Commission consists of one Chairperson and six Members who shall be appointed by the Central Government.

The commission is a quasi-judicial body which gives opinions to statutory authorities and also deals with other cases. The Chairperson and other Members shall be whole-time Members.

4. According to ___(A)___ of the Reserve Bank of India Act, 1934, the Reserve Bank shall publish, on the fourteenth day after every meeting of the Monetary Policy Committee, the minutes of the proceedings of the meeting.

Identify (A) in the above excerpt.

(1) Section 45ZA

(2) Section 45ZB

(3) Section 45ZL

(4) Section 48

(5) Section 56

Ans: 3

Solution:

According to Section 45ZL of the Reserve Bank of India Act, 1934, the Reserve Bank shall publish, on the fourteenth day after every meeting of the Monetary Policy Committee, the minutes of the proceedings of the meeting which shall include the following, namely:

  • the resolution adopted at the meeting of the Monetary Policy Committee;
  • the vote of each member of the Monetary Policy Committee, ascribed to such member, on the resolution adopted in the said meeting; and
  • the statement of each member of the Monetary Policy Committee under sub-section (11) of section 45ZI on the resolution adopted in the said meeting.

5. The Y.M. Deosthalee committee, ___(A)___ set up by the Reserve Bank of India (RBI) recommended the setting up of a Public Credit Registry.

Which year has been depicted as (A) in the above excerpt?

(1) 2008

(2) 2012

(3) 2014

(4) 2016

(5) 2018

Ans: 5

Solution:

Public Credit Registry (PCR)

PCR refers to an extensive database of credit information of borrowers that is accessible to all lending and credit decision-making institutions.

The Y.M. Deosthalee committee, 2018 set up by the Reserve Bank of India (RBI) recommended the setting up of a PCR.

Mint StatiCA MCQs – 22nd Dec, 2022

1. Which of the following statements is/are not true about the Reserve Bank of India?
A. The Reserve Bank of India was established on April 1, 1934 in accordance with the provisions of the Reserve Bank of India Act, 1934.
B. An amendment to RBI Act, 1934, was made in May 2015, providing the statutory basis for the implementation of the flexible inflation targeting framework.
C. Section 45ZB of the amended RBI Act, 1934, also provides for an empowered six-member Monetary Policy Committee (MPC) to be constituted by the Central Government by notification in the Official Gazette
Choose the correct option:
(1) Only A
(2) Only B
(3) Only A and B
(4) Only B and C
(5) All of the above

Ans: 3
Solution:
The Reserve Bank of India was established on April 1, 1935 in accordance with the provisions of the Reserve Bank of India Act, 1934.
An amendment to RBI Act, 1934, was made in May 2016, providing the statutory basis for the implementation of the flexible inflation targeting framework.
Section 45ZB of the amended RBI Act, 1934, also provides for an empowered six-member Monetary Policy Committee (MPC) to be constituted by the Central Government by notification in the Official Gazette

2. Which of the following statements is/are true about Inflation?
A. When an increase in the demand for goods and services causes a substantial increase in the price, it is known as Demand-pull inflation.
B. An increase in the cost of production, such as wages, raw materials, etc., causes an increase in prices, creating cost-push inflation.
C. The impact of the increased cost of living on the salary of employees is an example of built-in inflation.
Choose the correct option:
(1) Only A
(2) Only A and B
(3) Only B and C
(4) Only A and C
(5) All of the above

Ans: 5
Solution:
When an increase in the demand for goods and services causes a substantial increase in the price, it is known as Demand-pull inflation.
An increase in the cost of production, such as wages, raw materials, etc., causes an increase in prices, creating cost-push inflation.
The impact of the increased cost of living on the salary of employees is an example of built-in inflation.

3. The Centre has imposed anti-dumping duty on stainless steel seamless tubes and pipes imported from China to remove “injury” to the domestic industry. An anti-dumping duty is valid for a period of ______years from the date of imposition unless revoked earlier.
Fill in the Blanks.
(1) 2
(2) 5
(3) 7
(4) 10
(5) 12

Ans: 2
Solution:
The imposition of Anti-dumping duty is a measure to rectify the situation arising out of the dumping of goods and its trade distortive effect.
An anti-dumping duty is valid for a period of five years from the date of imposition unless revoked earlier.
Directorate General of Trade Remedies is the apex national authority under the Ministry of Commerce and Industry for administering all trade remedial measures including anti-dumping, countervailing duties and safeguard measures.

4. CBDCs are a digital form of a paper currency and unlike cryptocurrencies that operate in a regulatory vacuum, these are legal tenders issued and backed by a central bank.
Which country became the first economy to launch its nationwide CBDC — Sand Dollar?
(1) China
(2) Bahamas
(3) Nigeria
(4) Jamaica
(5) Sweden

Ans: 2
Solution:
CBDCs are a digital form of a paper currency and unlike cryptocurrencies that operate in a regulatory vacuum, these are legal tenders issued and backed by a central bank.
It is the same as a fiat currency and is exchangeable one-to-one with the fiat currency.
The digital fiat currency or CBDC can be transacted using wallets backed by blockchain.
Global Trends
Bahamas has been the first economy to launch its nationwide CBDC — Sand Dollar.
Nigeria is another country to have rolled out the eNaira in 2020.
China became the world’s first major economy to pilot a digital currency e-CNY in April 2020.
Korea, Sweden, Jamaica, and Ukraine are some of the countries to have begun testing its digital currency and many more may soon follow.

5. NCLT was formed based on the recommendations of the (A) Committee that was related to insolvency and winding up of companies in India.
Which committee has been depicted as (A) in the above excerpt?
(1) N.K. Singh Committee
(2) Bimal Jalan Committee
(3) Justice Eradi Committee
(4) Justice B.N. Srikrishna
(5) None of these

Ans: 3
Solution:
NCLT
It is a quasi-judicial body in India that adjudicates issues relating to companies in India.
Established on 1st June, 2016 (Companies Act, 2013).
Formed based on the recommendations of the Justice Eradi Committee.
It deals with matters mainly related to Companies law and Insolvency law.

Mint StatiCA MCQs – 21st Dec, 2022

1. The country’s markets regulator will gradually eliminate share buybacks through stock exchanges by April 2025 as it works to create a more equitable process for shareholders.
Who is the chairperson of the regulator being talked about in the above excerpt?
(1) Shaktikanta Das
(2) Madhabi Puri Buch
(3) Suraj Bhan
(4) Debasish Panda
(5) None of these

Ans: 2
Solution:
The regulator being talked about is SEBI.
Madhabi Puri Buch is the current chairman of SEBI.
SEBI is a statutory body established on April 12, 1992, in accordance with the provisions of the Securities and Exchange Board of India Act, 1992.
The basic functions of the Securities and Exchange Board of India are to protect the interests of investors in securities and to promote and regulate the securities market.

2. Finance minister Nirmala Sitharaman may unveil a range of initiatives to support India’s energy transition in the Union budget, including a ₹21,650 crore scheme to encourage the setting up of grid-scale BESS, ₹3,765 crores in viability grants for such projects, and reduced import duties on parts to build BESS.
What is ‘E’ in the acronym BESS?
(1) Energy
(2) Environment
(3) Export
(4) Ecosystem
(5) None of these

Ans: 1
Solution:
Finance minister Nirmala Sitharaman may unveil a range of initiatives to support India’s energy transition in the Union budget, including a ₹21,650 crore scheme to encourage the setting up of grid-scale battery energy storage systems (BESS), ₹3,765 crores in viability grants for such projects, and reduced import duties on parts to build BESS.

3. The government has established 12 committees comprising top officials to devise ways to stoke long-term economic growth amid global uncertainties. The 12 working groups are based on six sub-themes of two broad pillars—growth and job creation and inclusive human development.
Which of the following is the leader under the sub-theme ‘Minimizing Regulatory Compliances’?
(1) Madhya Pradesh
(2) Haryana
(3) Punjab
(4) Uttar Pradesh
(5) Tamil Nadu

Ans: 3
Solution:
The government has established 12 committees comprising top officials to devise ways to stoke long-term economic growth amid global uncertainties.
The 12 working groups are based on six sub-themes of two broad pillars—growth and job creation and inclusive human development. The six themes include minimizing regulatory compliances, infrastructure, and investments, thrust on small businesses, empowering women, health and nutrition, and skill development.
Punjab leads the group in minimizing regulatory compliance.

4. A Non-Banking Financial Company (NBFC) is a company registered under the Companies Act, 1956 engaged in the business of loans and advances, acquisition of shares/stocks/bonds/debentures/securities issued by Government or local authority or other marketable securities of a like nature, leasing, hire-purchase, insurance business, chit business but does not include any institution whose principal business is that of agriculture activity, industrial activity, purchase or sale of any goods (other than securities) or providing any services and sale/purchase/construction of immovable property.
Which of the following statements is/are not true about NBFCs in India?
A. NBFC can accept demand deposits.
B. NBFCs do not form part of the payment and settlement system and cannot issue cheques drawn on itself.
C. Deposit insurance facility of Deposit Insurance and Credit Guarantee Corporation is available to depositors of NBFCs.
Choose the correct option:
(1) Only B
(2) Only A and C
(3) Only B and C
(4) Only A
(5) All of the above

Ans: 2
Solution:
A Non-Banking Financial Company (NBFC) is a company registered under the Companies Act, 1956 engaged in the business of loans and advances, acquisition of shares/stocks/bonds/debentures/securities issued by Government or local authority or other marketable securities of a like nature, leasing, hire-purchase, insurance business, chit business but does not include any institution whose principal business is that of agriculture activity, industrial activity, purchase or sale of any goods (other than securities) or providing any services and sale/purchase/construction of immovable property.
A non-banking institution which is a company and has the principal business of receiving deposits under any scheme or arrangement in one lump sum or in instalments by way of contributions or in any other manner is also a non-banking financial company (Residuary non-banking company).
Features of NBFCs
NBFC cannot accept demand deposits.
NBFCs do not form part of the payment and settlement system and cannot issue cheques drawn on itself.
Deposit insurance facility of Deposit Insurance and Credit Guarantee Corporation is not available to depositors of NBFCs.

5. Under the _(A), securities are not issued directly to the public but are offered for sale through intermediaries like issuing houses or stockbrokers. In this case, a company sells securities enbloc at an agreed price to brokers who, in turn, resell them to the investing public.
Identify (A) in the above excerpt.
(1) IPO
(2) FPO
(3) OFS
(4) Private Placement
(5) None of these

Ans: 3
Solution:
Offer for Sale
Under the offer for sale method, securities are not issued directly to the public but are offered for sale through intermediaries like issuing houses or stockbrokers.
In this case, a company sells securities enbloc at an agreed price to brokers who, in turn, resell them to the investing public.

Mint StatiCA MCQs – 20th Dec, 2022

1. The Competition Act, of 2002 was enacted by the Parliament of India and governs Indian competition law. It replaced the archaic Monopolies and Restrictive Trade Practices Act, of _(A). Under this legislation, the Competition Commission of India was established to prevent activities that have an adverse effect on competition in India. This act extends to the whole of India.
Identify the year (A) in the above excerpt.
(1) 1948
(2) 1969
(3) 1975
(4) 1982
(5) 1995

Ans: 2
Solution:
The Competition Act, of 2002 was enacted by the Parliament of India and governs Indian competition law. It replaced the archaic Monopolies and Restrictive Trade Practices Act, of 1969. Under this legislation, the Competition Commission of India was established to prevent activities that have an adverse effect on competition in India. This act extends to the whole of India.

2. The government has developed new valuation metrics for imports after unearthing large-scale under-invoicing and misdeclaration of steel imports from China aimed at evading basic customs duty and goods and services tax. The Directorate General of Valuations under the_____(A)_ found that in April-July this year, stainless steel flat products of J3 grade were imported at an average value of ₹87 per kg as against the similar products of 201 grade imported at an average value of ₹163 per kg. In addition, similar products of 201 and J3 Dual Grade were imported at an average value of ₹149 per kg.
Identify the entity (A) in the above passage.
(1) Central Board of Indirect Taxes and Customs
(2) Central Board of Direct Taxes
(3) Export Inspection Council
(4) Export Promotion Council
(5) None of these

Ans: 1
Solution:
The Directorate of Valuation (DOV) was established in the year 1997 as an attached office of the Central Board of Excise and Customs under the Department of Revenue, Ministry of Finance, Government of India. The Directorate was upgraded to the Directorate General of Valuation (DGOV) in December 2002. The headquarters of the Directorate is situated at New Customs House, Mumbai. The Directorate is headed by the Director General assisted by the Commissioner and three Additional/Joint Commissioners at the Headquarters office. There are two zonal units functioning at Delhi (Northern Zone) and Chennai (Southern zone) each headed by a Joint Commissioner. The Directorate is neither a public dealing office nor a field formation.

3. The SURE project is a commitment by India’s apparel industry to set a sustainable pathway for the Indian fashion industry.
What does ‘SURE’ stand for?
(1) Sustainable Resolution
(2) Sustainable Revolution
(3) Sustainable Reforms
(4) Sustainable Rise
(5) Sustainable Renewables

Ans: 1
Solution:
The SURE project is a commitment by India’s apparel industry to set a sustainable pathway for the Indian fashion industry.
SURE, stands for ‘Sustainable Resolution’ – a firm commitment from the industry to move towards fashion that contributes to a clean environment.
The project has been launched by the union Textiles Ministry, along with the Clothing Manufacturers Association of India (CMAI); United Nations in India; and IMG Reliance.
Significance: It will be the first holistic effort by the apparel industry towards gradually introducing a broader framework for establishing critical sustainability goals for the industry.
This framework would help the industry reduce its carbon emissions, increase resource efficiency, tackle waste and water management, and create a positive social impact to achieve long-term sustainability targets.

4. The National Monetisation Pipeline (NMP) launched in August 2021, is designed to unlock the value of investments in such brownfield public sector assets by tapping institutional and long-term capital. The National Monetisation Pipeline (NMP) envisages an aggregate monetisation potential of (A) through the leasing of core assets of the Central government in sectors such as roads, railways, power, oil and gas pipelines, telecom, civil aviation etc, over (B)_.
Identify (A) and (B) respectively in the above excerpt.
(1) 2 Lakh Crores; 2 Years
(2) 4 Lakh Crores; 4 Years
(3) 6 Lakh Crores; 4 Years
(4) 10 Lakh Crores; 6 Years
(5) 10 Lakh Crores; 10 Years

Ans: 3
Solution:
The National Monetisation Pipeline (NMP) envisages an aggregate monetisation potential of ₹6-lakh crore through the leasing of core assets of the Central government in sectors such as roads, railways, power, oil and gas pipelines, telecom, civil aviation etc, over a four-year period (FY 2022-25).
The National Monetisation Pipeline (NMP) launched in August 2021, is designed to unlock the value of investments in such brownfield public sector assets by tapping institutional and long-term capital.

5. The most important part of MGNREGA’s design is its legally backed guarantee for any rural adult to get work within __ days of demanding it, failing which an ‘unemployment allowance’ must be given.
Fill in the Blanks.
(1) 5
(2) 10
(3) 15
(4) 20
(5) 305.

Ans: 3
Solution:
MGNREGA is one of the largest work guarantee programmes in the world launched in 2005 by the Ministry of Rural development.
The primary objective of the scheme is to guarantee 100 days of employment in every financial year to adult members of any rural household willing to do public work-related unskilled manual work.
As of 2022-23, there are 15.4 crores of active workers, under the MGNREGA.
Unlike earlier employment guarantee schemes, the act aims at addressing the causes of chronic poverty through a rights-based framework.
At least one-third of beneficiaries have to be women.
Wages must be paid according to the statutory minimum wages specified for agricultural labourers in the state under the Minimum Wages Act, of 1948.
The most important part of MGNREGA’s design is its legally backed guarantee for any rural adult to get work within 15 days of demanding it, failing which an ‘unemployment allowance’ must be given.

Mint StatiCA MCQs – 19th Dec, 2022

1. Generally, the Organization of the Petroleum Exporting Countries (OPEC) used to work as a cartel and fix prices in a favourable band. OPEC is led by Saudi Arabia, which is the largest exporter of crude oil in the world. OPEC has a total of 13 Member Countries.
Which of the following countries is/are not members of OPEC?
A. Algeria
B. Gabon
C. Angola
D. Russia
E. India
Choose the correct option:
(1) Only D
(2) Only A, B and C
(3) Only D and E
(4) Only B and C
(5) All of the above

Ans: 3
Solution:
Generally, the Organization of the Petroleum Exporting Countries (OPEC) used to work as a cartel and fix prices in a favourable band.
OPEC is led by Saudi Arabia, which is the largest exporter of crude oil in the world (single-handedly exporting 10% of the global demand).
OPEC has a total of 13 Member Countries viz. Iran, Iraq, Kuwait, United Arab Emirates (UAE), Saudi Arabia, Algeria, Libya, Nigeria, Gabon, Equatorial Guinea, Republic of Congo, Angola, and Venezuela.

2. Net direct tax collection of the central government including the December instalment of advance tax receipts has grown 20% so far, this fiscal from the year-ago period to ₹11.36 trillion, covering about 80% of the budgeted estimate for the whole fiscal, the Central Board of Direct Taxes (CBDT) said.
Which of the following are Direct Taxes?
(1) Minimum Alternate Tax
(2) GST
(3) Excise Duty
(4) Income Tax
(5) Both 1 and 4

Ans: 5
Solution:
Minimum Alternate Tax and Income Tax are Direct Taxes.
GST and Excise Duty are Indirect Taxes.

3. The Union Minister for Environment, Forest and Climate Change addressed the fifteenth session of the Conference of the Parties (COP15) of the United Nations Convention to Combat Desertification (UNCCD) in which African Country?
(1) Nigeria
(2) Ethiopia
(3) Cote d’Ivoire
(4) Botswana
(5) South Africa

Ans: 3
Solution:
The Union Minister for Environment, Forest and Climate Change addressed the fifteenth session of the Conference of the Parties (COP15) of the United Nations Convention to Combat Desertification (UNCCD) in Cote d’Ivoire (Western Africa).
Theme: ‘Land. Life. Legacy: From scarcity to prosperity’

4. Insolvency and Bankruptcy Code, _(A) provides a time-bound process for resolving insolvency in companies and among individuals.
Identify the year (A) in the above excerpt.
(1) 2016
(2) 2012
(3) 2018
(4) 2008
(5) None of these4.

Ans: 1
Solution:
Insolvency and Bankruptcy Code, 2016 provides a time-bound process for resolving insolvency in companies and among individuals.
Objectives of IBC
To consolidate and amend all existing insolvency laws in India.
To simplify and expedite the Insolvency and Bankruptcy Proceedings in India.
To protect the interest of creditors including stakeholders in a company.
To revive the company in a time-bound manner.
To promote entrepreneurship.
To get the necessary relief to the creditors and consequently increase the credit supply in the economy.
To work out a new and timely recovery procedure to be adopted by the banks, financial institutions, or individuals.
To set up an Insolvency and Bankruptcy Board of India.
Maximization of the value of assets of corporate persons.
Salient features of the Insolvency and Bankruptcy Code, 2016
Covers all individuals, companies, Limited Liability Partnerships (LLPs) and partnership firms.
Adjudicating authority:
National Company Law Tribunal (NCLT) for companies and LLPs
Debt Recovery Tribunal (DRT) for individuals and partnership firms
Establishment of an Insolvency and Bankruptcy Board of India to exercise regulatory oversight over insolvency professionals, insolvency professional agencies and information utilities.
Insolvency professionals handle the commercial aspects of the insolvency resolution process.
Insolvency professional agencies develop professional standards, code of ethics and be first-level regulator for insolvency professionals’ members leading to the development of a competitive industry for such professionals.
Information utilities collect, collate, authenticate and disseminate financial information to be used in insolvency, liquidation and bankruptcy proceedings.
Enabling provisions to deal with cross-border insolvency.

5. SEBI is a statutory body established on April 12, 1992, in accordance with the provisions of the Securities and Exchange Board of India Act, 1992.
Which of the following statements is/are true about SEBI?
A. Before SEBI came into existence, the Controller of Capital Issues was the regulatory authority; it derived authority from the Capital Issues (Control) Act, of 1950.
B. The headquarters of SEBI is situated in Mumbai. The regional offices of SEBI are located in Ahmedabad, Kolkata, Chennai and Delhi.
C. SEBI is a quasi-legislative and quasi-judicial body which can draft regulations, conduct inquiries, pass rulings, and impose penalties.
Choose the correct option:
(1) Only A
(2) Only B and C
(3) Only A and B
(4) Only A and C
(5) All of the above

Ans: 2
Solution:
Before SEBI came into existence, the Controller of Capital Issues was the regulatory authority; it derived authority from the Capital Issues (Control) Act, of 1947.
SEBI is a quasi-legislative and quasi-judicial body which can draft regulations, conduct inquiries, pass rulings, and impose penalties.
The headquarters of SEBI is situated in Mumbai. The regional offices of SEBI are located in Ahmedabad, Kolkata, Chennai and Delhi.

Mint StatiCA MCQs – 16th Dec, 2022

1. The Budget may announce steps to reduce India’s import dependence on China through various tariff and non-tariff measures, pursuing its strategy of Atmanirbhar Bharat (self-reliant India) in a “calibrated” manner.
Which of the following is not a pillar of Atmanirbhar Bharat?
(1) Economy
(2) Demography
(3) Development
(4) System
(5) Demand


Ans: 3
Solution:
Atmanirbhar Bharat or self-reliant India was launched by the PM in the wake of the Covid pandemic to manage the economic disruption in the country.
The objective of the Atmanirbhar Bharat program is to make the country and its people self-reliant and independent with the help of a full-fledged economic stimulus package.
Atmanirbhar Bharat Abhiyan refers to the measures taken by the central government to make India self-sufficient in areas like infrastructure, services, etc. The value of the economic package is INR 20 lakh crore or 10% of India’s Gross Domestic Product (GDP).
Atmanirbhar Bharat is based on 5 basic components: economy, demography, system, demand, and infrastructure.

2. According to commerce ministry data, India’s merchandise imports from China in October 2022 contracted 9.73% to $7.85 billion compared with $8.7 billion a year ago. In 2021-2022, China accounted for (A) per cent of India’s total trade ($1035 billion) and it was New Delhi’s second biggest trading partner after the (B)_ ($119.48 billion).
Identify (A) and (B) respectively in the above excerpt.
(1) 9.09; Saudi Arabia
(2) 11.19; UAE
(3) 9.09; UAE
(4) 11.19; EU
(5) 11.19; USA

Ans: 5
Solution:
In 2021-2022, China accounted for 11.19 per cent of India’s total trade ($1035 billion) and it was New Delhi’s second biggest trading partner after the United States ($119.48 billion).

3. Retail inflation, which surprisingly dropped to 5.88% in November, would have been as high as 7%, an increase from October, if not for cooling vegetable prices.
Which of the following components of the Consumer Price Index (Combined) has the highest weightage?
Choose the correct option:
(1) Housing
(2) Food and Beverage
(3) Fuel and Light
(4) Clothing and Footwear
(5) Pan, tobacco, and intoxicants

Ans: 2
Solution:
Components of CPI
The following are the primary components of CPI (C): (along with their weights)
Food and Beverage – 45.86
Housing – 10.07
Fuel and Light – 6.84
Clothing and Footwear – 6.53
Pan, tobacco, and intoxicants – 2.38
Miscellaneous – 28.32

4. The National Company Law Appellate Tribunal (NCLAT) was constituted under Section ____of the Companies Act, 2013 to hear appeals against the orders of the National Company Law Tribunal (NCLT).
Fill in the Blanks.
(1) Section 54
(2) Section 91
(3) Section 162
(4) Section 312
(5) Section 410

Ans: 5
Solution:
The NCLAT was constituted under Section 410 of the Companies Act, 2013 to hear appeals against the orders of the National Company Law Tribunal (NCLT).
NCLT is a quasi-judicial body that adjudicates issues relating to companies.
It is also the appellate tribunal for orders passed by the NCLT(s) under Section 61 of the Insolvency and Bankruptcy Code (IBC), 2016, and for orders passed by the Insolvency and Bankruptcy Board of India (IBBI) under Sections 202 and 211 of the IBC.
Any person aggrieved by any order of the NCLAT may file an appeal to the Supreme Court.

5. Some investors believe an expected recession will force the Federal Reserve to loosen monetary policy next year, even as the central bank projects it will raise rates higher than it previously anticipated and keep them there longer as it fights to crush inflation.
Which of the following statements is not true about the Monetary Policy?
(1) Monetary policy refers to the policy of the central bank – i.e., Reserve Bank of India in India – in matters of interest rates, money supply and availability of credit.
(2) An expansionary monetary policy is implemented by increasing key interest rates thus increasing market liquidity.
(3 A contractionary monetary policy is implemented by decreasing key interest rates thus reducing market liquidity.
(4) The Reserve Bank of India (RBI) is vested with the responsibility of conducting monetary policy. This responsibility is explicitly mandated under the Reserve Bank of India Act, 1934.
(5) Both 2 and 3

Ans: 5
Solution:
Monetary policy refers to the policy of the central bank – i.e., Reserve Bank of India in India – in matters of interest rates, money supply and availability of credit.
An expansionary monetary policy is focused on expanding (increasing) the money supply in an economy. An expansionary monetary policy is implemented by lowering key interest rates thus increasing market liquidity.
A contractionary monetary policy is focused on contracting (decreasing) the money supply in an economy. A contractionary monetary policy is implemented by increasing key interest rates thus reducing market liquidity.
The Reserve Bank of India (RBI) is vested with the responsibility of conducting monetary policy. This responsibility is explicitly mandated under the Reserve Bank of India Act, 1934.

Mint StatiCA MCQs – 15th Dec, 2022

1. India’s annual wholesale price index (WPI) inflation fell to its lowest in 21 months in November, driven by easing prices of food, fuel and manufactured goods, official data showed.
What is the base year of the wholesale price index (WPI)?
(1) 2004-05
(2) 2007-08
(3) 2011-12
(4) 2014-15
(5) 2017-18


Ans: 3
Solution:
Wholesale Price Index
It measures the changes in the prices of goods sold and traded in bulk by wholesale businesses to other businesses.
Published by the Office of Chief Economic Adviser, Ministry of Commerce and Industry.
The base year of All-India WPI has been revised from 2004-05 to 2011-12 in 2017.

2. The Ministry of Labour and Employment released the new series of___(A)__ with the base year of 2016.
Identify (A) in the above excerpt.
Choose the correct option:
(1) CPI-IW
(2) CPI-AL
(3) CPI-RL
(4) CPI-Rural
(5) CPI-Urban

Ans: 1
Solution:
Recently, the Ministry of Labour and Employment released the new series of the Consumer Price Index for Industrial Worker (CPI-IW) with the base year 2016.
Consumer Price Index
It measures price changes from the perspective of a retail buyer. It is released by the National Statistical Office (NSO).
The CPI calculates the difference in the price of commodities and services such as food, medical care, education, electronics etc, which Indian consumers buy for use.
Four types of CPI are as follows:
CPI for Industrial Workers (IW).
CPI for Agricultural Labourer (AL).
CPI for Rural Labourer (RL).
CPI (Rural/Urban/Combined).

3. While the International Monetary Fund (IMF) has forecast slower growth for the global economy in 2023, it is positive for India that countries that are looking to diversify their supply chains are considering India as an option, said the IMF’s first deputy managing director, Gita Gopinath.
Which of the following reports is/are not published by the IMF?
A. World Economic Outlook
B. Global Financial Stability Report
C. Fiscal Monitor
D. World Development Report
E. Global Hunger Index
Choose the correct option:
(1) Only A, B and D
(2) Only A, B and C
(3) Only B, C and D
(4) Only D and E
(5) Only B and C

Ans: 4
Solution:
Publications of IMF
World Economic Outlook
Global Financial Stability Report
Fiscal Monitor
Regional Economic Outlook
Annual Report of the Executive Board

4. NaBFID was set up as a corporate body with an authorised share capital of one lakh crore rupees.
Which of the following options depicts the functions of NaBFID?
(1) Extending loans and advances for infrastructure projects.
(2) Taking over or refinancing such existing loans.
(3) Attracting investment from private sector investors and institutional investors for infrastructure projects.
(4) Organising and facilitating foreign participation in infrastructure projects.
(5) All of the above

Ans: 5
Solution:
NaBFID was set up as a corporate body with an authorised share capital of one lakh crore rupees.
Functions of NaBFID
Extending loans and advances for infrastructure projects.
Taking over or refinancing such existing loans.
Attracting investment from private sector investors and institutional investors for infrastructure projects.
Organising and facilitating foreign participation in infrastructure projects.
Facilitating negotiations with various government authorities for dispute resolution in the field of infrastructure financing.
Providing consultancy services in infrastructure financing.

5. The Production Linked Incentive Scheme (PLI) for Large Scale Electronics Manufacturing shall extend an incentive of (A) on incremental sales (over the base year) of goods manufactured in India and covered under target segments, to eligible companies, for a period of (B) subsequent to the base year as defined.
(1) 1-2 %; 3 Years
(2) 1-2 %; 5 Years
(3) 2-4 %; 3 Years
(4) 4-6 %; 5 Years
(5) 4-6 %; 3 Years

Ans: 4
Solution:
Production Linked Incentive Scheme (PLI) for Large Scale Electronics Manufacturing notified vide Gazette Notification dated April 01, 2020 offers a production linked incentive to boost domestic manufacturing and attract large investments in mobile phone manufacturing and specified electronic components, including Assembly, Testing, Marking and Packaging (ATMP) units. The Scheme would tremendously boost the electronics manufacturing landscape and establish India at the global level in electronics sector.
The scheme shall extend an incentive of 4% to 6% on incremental sales (over the base year) of goods manufactured in India and covered under target segments, to eligible companies, for a period of five (5) years subsequent to the base year as defined.
The Scheme will be implemented through a Nodal Agency which shall act as a Project Management Agency (PMA) and be responsible for providing secretarial, managerial and implementation support and carrying out other responsibilities as assigned by MeitY from time to time.

Mint StatiCA MCQs – 14th Dec, 2022

1. Industrial Policy Resolution of 1956 (IPR 1956) is a resolution adopted by the Indian parliament in April 1956. It was the second comprehensive statement on the industrial development of India after the Industrial Policy of 1948. The 1956 policy continued to constitute the basic economic policy for a long time. This fact has been confirmed in all the Five-Year Plans of India.
Which of the following industries were included under Schedule A of the IPR 1956?
A. Arms and ammunition and allied items of defence equipment
B. Atomic energy
C. Iron and Steel
D. Heavy castings and forgings of iron and steel
Choose the correct option:
(1) Only B
(2) Only A and B
(3) Only A, B and C
(4) Only A, B and D
(5) All of the Above

Ans: 5
Solution:
Industrial Policy Resolution of 1956 (IPR 1956) is a resolution adopted by the Indian parliament in April 1956. It was the second comprehensive statement on the industrial development of India after the Industrial Policy of 1948. The 1956 policy continued to constitute the basic economic policy for a long time. This fact has been confirmed in all the Five-Year Plans of India. According to this resolution, the objective of the social and economic policy in India was the establishment of a socialistic pattern of society. It provided more powers to the governmental machinery. It laid down three categories of industries which were more sharply defined. These categories were:
Schedule A: those industries which were to be an exclusive responsibility of the state.
Schedule B: those which were to be progressively state-owned and in which the state would generally set up new enterprises, but in which private enterprise would be expected only to supplement the effort of the state; and
Schedule C: all the remaining industries and their future development would, in general, be left to the initiative and enterprise of the private sector.
Schedule A Industries
1.Arms and ammunition and allied items of defence equipment
2.Atomic energy
3.Iron and Steel
4.Heavy castings and forgings of iron and steel
5.Heavy plant and machinery required for iron and steel production, for mining, for machine tool manufacture and for such other basic industries as may be specified by the Central Government
6.Heavy electrical plant including large hydraulic and steam turbines
7.Coal and lignite
8.Mineral oils
9.Mining of iron ore, manganese ore, chrome-ore, gypsum, sulphur, gold and diamond
10.Mining and processing of copper, lead, zinc, tin, molybdenum and wolfram
11.Minerals specified in the Schedule to the Atomic Energy (Control of Production and Use) Order, 1953
12.Aircraft
13.Air transport
14.Railway Transport
15.Ship Building
16.Telephones and telephone cables, telegraph and wireless apparatus (excluding radio receiving sets)
17.Generation and distribution of electricity

2. Green bonds are issued by companies, countries, and multilateral organisations to exclusively fund projects that have positive environmental or climate benefits and provide investors with fixed-income payments.
Which of the following options are excluded from the Sovereign Green Bonds Framework of the Government of India?
A. Projects involving new or existing extraction, production, and distribution of fossil fuels, including improvements and upgrades; or where the core energy source is fossil-fuel based
B. Nuclear power generation
C. Direct waste incineration
D. Alcohol, weapons, tobacco, gaming, or palm oil industries
Choose the correct option:
(1) Only D
(2) Only B, C and D
(3) Only A, B and C
(4) Only A, C and D
(5) All of the Above

Ans: 5
Solution:
Union Minister for Finance & Corporate Affairs Smt. Nirmala Sitharaman approved the final Sovereign Green Bonds framework of India.
Projects excluded from the Sovereign Green Bond Framework
Direct waste incineration
Alcohol, weapons, tobacco, gaming, or palm oil industries
Nuclear power generation
Landfill projects
Hydropower plants larger than 25 MW
Renewable energy projects generating energy from biomass using feedstock originating from protected areas
Projects involving new or existing extraction, production, and distribution of fossil fuels, including improvements and upgrades; or where the core energy source is fossil-fuel based

3. Non-Performing Assets (NPA) are loans and arrears lent by banks or financial institutions whose principal and interests are delayed beyond ____days. In simpler terms, any asset that ceases to provide returns to its investors for an extended period of time is referred to as a non-performing asset (NPA).
Fill in the Blanks.
(1) 30
(2) 60
(3) 90
(4) 180
(5) 270

Ans: 3
Solution:
Non-Performing Assets (NPA) are loans and arrears lent by banks or financial institutions whose principal and interests are delayed beyond 90 days. In simpler terms, any asset that ceases to provide returns to its investors for an extended period of time is referred to as a non-performing asset (NPA).
Classification of Non-Performing Assets
Sub-standard: When the NPAs have aged <= 12 months. Doubtful: When the NPAs have aged > 12 months.
Loss assets: When the bank or its auditors have identified the loss, but it has not been written off.

4. Hybrid Annuity Model is one of the most important variants of the PPP model infrastructure investment that is adopted in our country. It can be said that it evolved as a solution to the limitations faced by the existing EPC model. EPC accounts for (A)% of this model, whereas BOT-Annuity accounts for (B)%.
Identify (A) and (B) respectively in the above excerpt.
(1) 40; 60
(2) 60; 40
(3) 50; 50
(4) 30; 70
(5) 70; 30

Ans: 1
Solution:
Hybrid Annuity Model is one of the most important variants of the PPP model infrastructure investment that is adopted in our country. It can be said that it evolved as a solution to the limitations faced by the existing EPC model. EPC accounts for 40% of this model, whereas BOT-Annuity accounts for 60%. As per the circular dated 9th February 2016, released by the Ministry of Road Transport and Highways, National Highway Projects are also to be implemented on the Hybrid Annuity Model as one of the modes of delivery.

5. Competition Commission of India (CCI) is a statutory body of the Government of India responsible for enforcing the Competition Act, __(A), it was duly constituted in March 2009.
Identify (A) in the above excerpt.
(1) 1988
(2) 1992
(3) 1998
(4) 2002
(5) 2006

Ans: 4
Solution:
Competition Commission of India (CCI) is a statutory body of the Government of India responsible for enforcing the Competition Act, 2002, it was duly constituted in March 2009.
The Monopolies and Restrictive Trade Practices Act, 1969 (MRTP Act) was repealed and replaced by the Competition Act, 2002, on the recommendations of the Raghavan committee.

Mint StatiCA MCQs – 13th Dec, 2022

1. As defined in Securities and Exchange Board of India (Alternative Investment Funds) Regulations, __(A), AIFs refer to any privately pooled investment fund, (whether from Indian or foreign sources), in the form of a trust or a company or a body corporate or a Limited Liability Partnership (LLP).
Identify the year (A) as mentioned in the above excerpt.
(1) 2004
(2) 2009
(3) 2012
(4) 2015
(5) 2017

Ans: 3
Solution:
As defined in the Securities and Exchange Board of India (Alternative Investment Funds) Regulations, 2012, AIFs refer to any privately pooled investment fund, (whether from Indian or foreign sources), in the form of a trust or a company or a body corporate or a Limited Liability Partnership (LLP).
As per SEBI (AIF) Regulations, 2012, AIFs shall seek registration in one of the three categories:
Category I: Mainly invests in start-ups, small and medium enterprises, or any other sector that Govt. considers economically and socially viable.
Category II: These include Alternative Investment Funds such as private equity funds or debt funds for which no specific incentives or concessions are given by the government or any other Regulator
Category III: Alternative Investment Funds such as hedge funds or funds which trade with a view to make short-term returns or such other funds which are open ended and for which no specific incentives or concessions are given by the government or any other Regulator.

2. The Central Government has notified the following as the factors that constitute failure to achieve the inflation target: (a) the average inflation is more than the upper tolerance level of the inflation target for any (A) consecutive quarters, or (b) the average inflation is less than the lower tolerance level for any (A) consecutive quarters.
Which of the following can replace (A) in the above passage?
(1) 1
(2) 2
(3) 3
(4) 4
(5) 5

Ans: 3
Solution:
In May 2016, the RBI Act, 1934 was amended to provide a statutory basis for the implementation of the flexible inflation targeting framework.
Inflation Target: Under Section 45ZA, the Central Government, in consultation with the RBI, determines the inflation target in terms of the Consumer Price Index (CPI), once in five years and notifies it in the Official Gazette.
Accordingly, on August 5, 2016, the Central Government notified in the Official Gazette 4 per cent Consumer Price Index (CPI) inflation as the target for the period from August 5, 2016 to March 31, 2021 with the upper tolerance limit of 6 per cent and the lower tolerance limit of 2 per cent. On March 31, 2021, the Central Government retained the inflation target and the tolerance band for the next 5-year period – April 1, 2021 to March 31, 2026.
Section 45ZB of the RBI Act provides for the constitution of a six-member Monetary Policy Committee (MPC) to determine the policy rate required to achieve the inflation target.
Failure to Maintain Inflation Target: The Central Government has notified the following as the factors that constitute a failure to achieve the inflation target: (a) the average inflation is more than the upper tolerance level of the inflation target for any three consecutive quarters; or (b) the average inflation is less than the lower tolerance level for any three consecutive quarters.
Where the Bank fails to meet the inflation target, it shall set out in a report to the Central Government:
a. the reasons for failure to achieve the inflation target;
b. remedial actions proposed to be taken by the Bank; and
c. an estimate of the time period within which the inflation target shall be achieved pursuant to timely implementation of proposed remedial actions.
The operating framework of monetary policy aims at aligning the operating target – the weighted average call rate (WACR) – with the policy repo rate through proactive liquidity management to facilitate the transmission of repo rate changes through the entire financial system, which, in turn, influences aggregate demand – a key determinant of inflation and growth.

3. The Monetary Policy Committee (MPC) uses CPI data to control inflation. In April 2014, the Reserve Bank of India (RBI) had adopted the CPI as its key measure of inflation.
What is the base year of CPI?
(1) 2002
(2) 2006
(3) 2010
(4) 2012
(5) 2016

Ans: 4
Solution:
Consumer Price Index
It measures price changes from the perspective of a retail buyer.
It is released by the National Statistical Office (NSO).
The CPI calculates the difference in the price of commodities and services such as food, medical care, education, electronics etc, which Indian consumers buy for use.
Four types of CPI are as follows:
CPI for Industrial Workers (IW).
CPI for Agricultural Labourer (AL).
CPI for Rural Labourer (RL).
CPI (Rural/Urban/Combined).
Of these, the first three are compiled by the Labour Bureau in the Ministry of Labour and Employment. Fourth is compiled by the NSO in the Ministry of Statistics and Programme Implementation.
The base year for CPI is 2012.

4. The empowered group of secretaries which was instituted in June 2020 was tasked with identifying the bottlenecks in PLI schemes, coordinating between states and companies for faster approvals, evaluating and ensuring quick investments in PLI schemes, and ensuring overall turnaround of projects.
Who is the chairman of the empowered group of secretaries?
(1) Prime Minister
(2) Union Finance Secretary
(3) Cabinet Secretary
(4) Secretary, DPIIT
(5) CEO, NITI Aayog

Ans: 3
Solution:
Recently, the NITI Aayog has commenced work on evolving a set of objective criteria to track value addition by companies which avail financial rewards under Production-Linked Incentive (PLI) schemes.
The empowered group of secretaries which was instituted in June 2020 was tasked with identifying the bottlenecks in PLI schemes, coordinating between states and companies for faster approvals, evaluating and ensuring quick investments in PLI schemes, and ensuring overall turnaround of projects.
The group is chaired by the Cabinet Secretary and has the Chief Executive Officer of NITI Aayog, the secretaries of the Department for Promotion of Industry and Internal Trade, the Department of Commerce, the Department of Revenue, the Department of Economic Affairs, and the Secretary of the concerned ministry as its members.

5. The GST council is devised in such a way that the centre will have (A)_ voting power and the states have (B). The decisions are taken by (C)__ majority.
Identify (A), (B) and (C) respectively in the above excerpt.
(1) 50%; 50%; 50%
(2) 33%; 66%; 75%
(3) 66%; 33%; 50%
(4) 25%; 75%; 75%
(5) 75%; 25%; 75%

Ans: 2
Solution:
Article 279A
GST Council to be formed by the President to administer & govern GST. Its chairman is the Union Finance Minister of India with ministers nominated by the state governments as its members.
The council is devised in such a way that the centre will have 1/3rd voting power and the states have 2/3rd.
The decisions are taken by 3/4th majority.

Mint StatiCA MCQs – 12th Dec, 2022

1. The 101st Amendment Act of 2016 paved the way for the introduction of a new tax regime (i.e., goods and services tax – GST) in the country. The amendment inserted a new Article 279-A in the Constitution of India. This article empowered the President to constitute a GST Council by an order.
Who is the chairman of the GST Council?
(1) Prime Minister
(2) Union Finance Minister
(3) Finance Minister of a State in Rotation
(4) Union Finance Secretary
(5) Vice Chairman, NITI Aayog

Ans: 2
Solution:
The 101st Amendment Act of 2016 paved the way for the introduction of a new tax regime (i.e., goods and services tax – GST) in the country. The smooth and efficient administration of this tax requires cooperation and coordination between the centre and the states.
To facilitate this consultation process, the amendment provided for the establishment of a GST Council.
The amendment inserted a new Article 279-A in the Constitution of India. This article empowered the President to constitute a GST Council by an order.
Accordingly, the President issued the order in 2016 and constituted the Council. The Secretariat of the Council is in New Delhi. The Union Revenue Secretary acts as the ex-officio Secretary to the Council.

2. Free Trade Agreement (FTA) is a pact between two or more nations to reduce barriers to imports and exports among them. Under a free trade policy, goods and services can be bought and sold across international borders with little or no government tariffs, quotas, subsidies, or prohibitions to inhibit their exchange. So far, India has signed _______Free Trade Agreements (FTAs) with its trading partners, including the 3 agreements, namely India-Mauritius Comprehensive Economic Cooperation and Partnership Agreement (CECPA), India-UAE Comprehensive Partnership Agreement (CEPA) and India-Australia Economic Cooperation and Trade Agreement (Ind-Aus ECTA) signed during the last five years.
Fill in the Blanks.
(1) 5
(2) 7
(3) 10
(4) 13
(5) 17

Ans: 4
Solution:
So far, India has signed 13 Free Trade Agreements (FTAs) with its trading partners, including the 3 agreements, namely India-Mauritius Comprehensive Economic Cooperation and Partnership Agreement (CECPA), India-UAE Comprehensive Partnership Agreement (CEPA) and India-Australia Economic Cooperation and Trade Agreement (Ind-Aus ECTA) signed during the last five years.

3. The National Health Authority (NHA) is setting up a health exchange platform that will digitize and simplify the process of filing health insurance claims, NHA CEO _(A) said in an interview.
Identify (A) in the above excerpt.
(1) Amitabh Kant
(2) R.S. Sharma
(3) Parameswaran Iyer
(4) Samant Goel
(5) None of these

Ans: 2
Solution:
The National Health Authority (NHA) is setting up a health exchange platform that will digitize and simplify the process of filing health insurance claims, NHA CEO R.S. Sharma said in an interview.
Ayushman Bharat Pradhan Mantri Jan Arogya Yojana (AB PM-JAY) will be the first insurance scheme in the world to implement this digital solution, with the platform to be rolled out in Haryana, followed by a pan-India launch.
PM-JAY is the world’s largest health insurance/ assurance scheme fully financed by the government.
Launched in February 2018, it offers a sum insured of Rs.5 lakh per family for secondary care (which doesn’t involve a super-specialist) as well as tertiary care (which involves a super-specialist).
Under PMJAY, cashless and paperless access to services is provided to the beneficiaries at the point of service, that is, the hospital.
Health Benefit Packages cover surgery, medical and daycare treatments, cost of medicines and diagnostics.
Packaged rates (Rates that include everything so that each product or service is not charged for separately).
These are flexible but the hospitals can’t charge the beneficiary more once fixed.

4. An AA is a framework that simply facilitates sharing of financial information in a real-time and data-blind manner (Data flow through AA are encrypted) between regulated entities (Banks and NBFCs). The RBI (Reserve Bank of India) in _ approved AA as a new class of NBFC (Non-Banking Financial Companies), whose primary responsibility is to facilitate the transfer of user’s financial data with their explicit consent.
Fill in the Blanks.
(1) 2010
(2) 2013
(3) 2016
(4) 2018
(5) 2020

Ans: 3
Solution:
An AA is a framework that simply facilitates sharing of financial information in a real-time and data-blind manner (Data flow through AA are encrypted) between regulated entities (Banks and NBFCs).
The RBI (Reserve Bank of India) in 2016 approved AA as a new class of NBFC (Non Banking Financial Companies), whose primary responsibility is to facilitate the transfer of user’s financial data with their explicit consent.
AAs enable flow of data between Financial Information Providers (FIPs) and Financial Information Users (FIUs).
The architecture of AA is based on the Data Empowerment and Protection Architecture (DEPA) framework.

5. Which state is the largest producer of coffee in India accounting for about 70% of total coffee production and 60% of the area under coffee in India?
(1) Karnataka
(2) Assam
(3) Kerela
(4) Tamil Nadu
(5) Andhra Pradesh

Ans: 1
Solution:
Karnataka is the largest producer (about 70% of total coffee production and 60% of the area under coffee in India).
Conditions of Growth
-The hot and humid climate
-Temperature between 15°C and 28 °C.
-Rainfall from 150 to 250 cm.
-Well-drained, rich friable loams containing a good deal of humus and minerals like iron and calcium are ideal for coffee cultivation.
-Dry weather is necessary at the time of ripening of the berries.
-The crop is not tolerant to
Frost and snowfall.
High temperature above 30°C and strong sunshine (Hence, generally grown under shady trees on Northern and Eastern aspects of Hill).
Prolonged drought.
Stagnant water (Hence, grown on hill slopes at elevations from 600 to 1,600 metres above sea level).

Mint StatiCA MCQs – 9th Dec, 2022

1. In May 2016, the RBI Act, 1934 was amended to provide a statutory basis for the implementation of the flexible inflation targeting framework. Under Section (A), the Central Government, in consultation with the RBI, determines the inflation target in terms of the Consumer Price Index (CPI), once in five years and notifies it in the Official Gazette.
Which Section of the RBI Act, 1934 has been depicted as (A) in the above excerpt?
(1) Section 7
(2) Section 18C
(3) Section 24
(4) Section 45ZA
(5) Section 58

Ans: 4
Solution:
In May 2016, the RBI Act, 1934 was amended to provide a statutory basis for the implementation of the flexible inflation targeting framework.
Inflation Target
Under Section 45ZA, the Central Government, in consultation with the RBI, determines the inflation target in terms of the Consumer Price Index (CPI), once in five years and notifies it in the Official Gazette. Accordingly, on August 5, 2016, the Central Government notified in the Official Gazette 4 per cent Consumer Price Index (CPI) inflation as the target for the period from August 5, 2016 to March 31, 2021 with the upper tolerance limit of 6 per cent and the lower tolerance limit of 2 per cent. On March 31, 2021, the Central Government retained the inflation target and the tolerance band for the next 5-year period – April 1, 2021 to March 31, 2026.

2. Which of the following indices is not released by the Labour Bureau?
(1) CPI for Industrial Workers (IW)
(2) CPI for Agricultural Labourer (AL)
(3) CPI for Rural Labourer (RL)
(4) CPI (Rural/Urban/Combined)
(5) None of the above

Ans: 4
Solution:
Four types of CPI are as follows:
CPI for Industrial Workers (IW)
CPI for Agricultural Labourer (AL)
CPI for Rural Labourer (RL)
CPI (Rural/Urban/Combined)
Of these, the first three are compiled by the Labour Bureau in the Ministry of Labour and Employment. Fourth is compiled by the NSO in the Ministry of Statistics and Programme Implementation.

3. The Union government has infused about ₹5,000 crore in SWAMIH Investment Fund I, helping the fund mark its final close of ₹15,530 crore.
What is ‘A’ in the acronym “SWAMIH”?
(1) Agreement
(2) Affordable
(3) Accommodation
(4) Accumulation
(5) Adjusted

Ans: 2
Solution:
SWAMIH (Special Window for Affordable & Mid-Income Housing) Fund
This is a government-backed fund that was set up as a Category-II AIF (Alternate Investment Fund) debt fund registered with SEBI and launched in 2019.
SWAMIH Investment Fund was formed to complete the construction of stalled, RERA-registered affordable and mid-income category housing projects which are stuck due to paucity of funds.
The Investment Manager of the Fund is SBICAP Ventures, a wholly owned subsidiary of SBI Capital Markets, which in turn is a wholly owned subsidiary of the State Bank of India.
The Sponsor of the Fund is the Secretary, Department of Economic Affairs, Ministry of Finance, on behalf of the Government of India.

4. The government is planning to set up a “National Cooperative Export Society” to help boost India’s exports, with a proposal on it having been circulated by the ministry of cooperation among central ministries and departments for consultation. The Constitution ((A) Amendment) Act of 2011 on cooperatives aims to promote cooperative economic activity, which in turn aids rural India’s advancement.
Which of the following options can replace (A) in the above passage?
(1) 88th
(2) 93rd
(3) 97th
(4) 99th
(5) 101st

Ans: 3
Solution:
An independent group of people who have come together voluntarily to address their common economic, social, and cultural needs and goals through a jointly owned and democratically controlled enterprise is known as a cooperative society.
The Constitution (Ninety Seventh Amendment) Act of 2011 on cooperatives aims to promote cooperative economic activity, which in turn aids rural India’s advancement.

5. The PHD Chamber of Commerce and Industry warned that exports could suffer more because of the unavailability of export credit insurance provided by the state-run Export Credit Guarantee Corp. of India (ECGC). The Government of India had initially set up the Export Risks Insurance Corporation in __. After the introduction of insurance covers to banks during the period 1962-64, the name was changed to Export Credit & Guarantee Corporation Ltd in 1964. It was changed to ECGC Ltd in August 2014.
Fill in the Blanks.
(1) 1947
(2) 1950
(3) 1953
(4) 1957
(5) 1960

Ans: 4
Solution:
ECGC Ltd is wholly owned by the Ministry of Commerce and Industry.
The Government of India had initially set up the Export Risks Insurance Corporation in 1957.
After the introduction of insurance covers to banks during the period 1962-64, the name was changed to Export Credit & Guarantee Corporation Ltd in 1964.
It was changed to ECGC Ltd in August 2014.
Objectives: ECGC was established to promote exports by providing credit insurance services to exporters against non-payment risks by overseas buyers due to commercial and political reasons.

Mint StatiCA MCQs – 8th Dec, 2022

1. The Reserve Bank of India Act, 1934 (RBI Act) was amended by the Finance Act, _(A), to provide for a statutory and institutionalized framework for a Monetary Policy Committee, for maintaining price stability, while keeping in mind the objective of growth.
Identify the year (A) in the above excerpt.
(1) 2012
(2) 2014
(3) 2016
(4) 2018
(5) 2020

Ans: 3
Solution:
The Reserve Bank of India Act, 1934 (RBI Act) was amended by the Finance Act, 2016, to provide for a statutory and institutionalized framework for a Monetary Policy Committee, for maintaining price stability, while keeping in mind the objective of growth. The Monetary Policy Committee is entrusted with the task of fixing the benchmark policy rate (repo rate) required to contain inflation within the specified target level.
The Government of India, in consultation with RBI, notified the ‘Inflation Target’ in the Gazette of India dated 5 August 2016 for the period beginning from the date of publication of the notification and ending on March 31, 2021, as 4%. At the same time, lower and upper tolerance levels were notified to be 2% and 6% respectively.

2. Which of the following statements is incorrect concerning the instruments of Monetary Policy?
(1) An open market operation is an instrument which involves buying/selling securities like government bonds from or to the public and banks.
(2) Cash Reserve Ratio is a specified amount of bank deposits that banks are required to keep with the RBI in the form of reserves or balances.
(3) All financial institutions have to maintain a certain quantity of liquid assets with themselves at any point in time of their total time and demand liabilities. This is known as the Statutory Liquidity Ratio.
(4) Also known as the discount rate, bank rates are interest charged by the RBI for providing funds and loans to the banking system.
(5) None of the above

Ans: 5
Solution:
Instruments of monetary policy
Some of the following instruments are used by RBI as a part of their monetary policies.
Open Market Operations: An open market operation is an instrument which involves buying/selling securities like government bonds from or to the public and banks. The RBI sells government securities to control the flow of credit and buys government securities to increase credit flow.
Cash Reserve Ratio (CRR): Cash Reserve Ratio is a specified amount of bank deposits that banks are required to keep with the RBI in the form of reserves or balances. The higher the CRR with the RBI, the lower will be the liquidity in the system and vice versa.
Statutory Liquidity Ratio (SLR): All financial institutions have to maintain a certain quantity of liquid assets with themselves at any point in time of their total time and demand liabilities. This is known as the Statutory Liquidity Ratio.
Bank Rate Policy: Also known as the discount rate, bank rates are interest charged by the RBI for providing funds and loans to the banking system. An increase in bank rate increases the cost of borrowing by commercial banks which results in the reduction in credit volume to the banks and hence the supply of money declines. An increase in the bank rate is the symbol of the tightening of the RBI monetary policy.
Credit Ceiling: With this instrument, RBI issues prior information or direction that loans to the commercial bank will be given up to a certain limit. In this case, a commercial bank will be tight in advancing loans to the public. They will allocate loans to limited sectors. A few examples of credit ceiling are agriculture sector advances and priority sector lending.

3. Demonetisation is the act of stripping a currency unit of its status as legal tender. It occurs whenever there is a change of national currency and the current form or forms of money is pulled from circulation and retired, often to be replaced with new notes or coins.
Demonetisation in India was implemented in which of the following years?
(A) 1946
(B) 1978
(C) 2016
(D) 2018
Choose the correct option:
(1) Only C
(2) Only A, B and C
(3) Only B and C
(4) Only A, B and D
(5) All of the above

Ans: 2
Solution:
Demonetisation: It is the act of stripping a currency unit of its status as legal tender. It occurs whenever there is a change of national currency and the current form or forms of money is pulled from circulation and retired, often to be replaced with new notes or coins.
On 8 November 2016, the Government of India announced the demonetisation of all ₹500 and ₹1,000 banknotes of the Mahatma Gandhi Series.
The Indian government had demonetised banknotes on two prior occasions—once in 1946 and again in 1978—and in both cases, the goal was to combat tax evasion via “black money” held outside the formal economic system. In 1978, the Janata Party coalition government demonetised banknotes of ₹1,000, ₹5,000 and ₹10,000, again in the hopes of curbing counterfeit money and black money.

4. NIIF is a government-backed entity established to provide long-term capital to the country’s infrastructure sector.
What is the stake of the Government of India in NIIF?
(1) 49 %
(2) 33%
(3) 51 %
(4) 66 %
(5) 85 %

Ans: 1
Solution:
NIIF is a government-backed entity established to provide long-term capital to the country’s infrastructure sector.
The Indian government has a 49% stake in NIIF with the rest held by foreign and domestic investors.
With the Centre’s significant stake, NIIF is considered India’s quasi-sovereign wealth fund.
It was set up in December 2015 as a Category-II Alternate Investment Fund.
Across its three funds viz. Master Fund, Fund of Funds, and Strategic Opportunities Fund, it manages over USD 4.3 billion of capital.
Its registered office is in New Delhi.

5. One of the major highlights of this year’s budget is the announcement of the Government’s decision to launch the Digital Rupee – India’s version of a Central Bank Digital Currency (CBDC). The announcement is a reiteration of the _ that had asked the RBI to introduce its own digital currency and ban private cryptocurrencies completely.
Fill in the Blanks.
(1) N.K Singh Committee
(2) S.C. Garg Committee
(3) Raghuram Rajan Committee
(4) Urjit Patel Committee
(5) Bimal Jalan Committee

Ans: 2
Solution:
One of the major highlights of this year’s budget is the announcement of the Government’s decision to launch the Digital Rupee – India’s version of a Central Bank Digital Currency (CBDC).
The announcement is a reiteration of the SC Garg Committee that had asked the RBI to introduce its own digital currency and ban private cryptocurrencies completely.
A Central Bank Digital Currency (CBDC), or national digital currency, is the digital form of a country’s fiat currency. Instead of printing paper currency or minting coins, the central bank issues electronic tokens. This token value is backed by the full faith and credit of the Government.

Mint StatiCA MCQs – 7th Dec, 2022

1. The price cap on Russian seaborne crude oil and oil products has been imposed by G7, in which the European Union is a non-enumerated member, to curb Russia’s earnings from its oil exports that fuel its war in Ukraine. The latest move may not have any more severe impact as the world’s (A) largest oil producer has been selling its oil at discounted prices to other buyers including India, China and Southeast Asian countries, routing a large quantum of its supplies to these countries of late.
Identify (A) in the above passage.
(1) 2nd
(2) 3rd
(3) 5th
(4) 6th
(5) 8th

Ans: 2
Solution:
Russia is the 3rd largest oil producer in the world.
The USA is the largest oil producer in the world and Saudi Arabia holds the 2nd Rank.
Russian oil exports are already shunned by several European Union countries and the US. The latest move may not have any more severe impact as the world’s third-largest oil producer has been selling its oil at discounted prices to other buyers including India, China and Southeast Asian countries, routing a large quantum of its supplies to these countries of late. China imported 904,000 barrels per day and India imported 883,000 barrels per day in October from Russia. And Suppliers to India and China together constituted 58% of total seaborne oil exports by Russia.

2. The Prime Minister Gati Shakti Master Plan, which aims to reduce logistics costs in India to levels comparable to developed nations, has garnered international interest and could be shared with neighbouring countries to aid infrastructure planning.
Which of the following statements is incorrect concerning Prime Minister Gati Shakti Master Plan?
(1) The Gati Shakti scheme subsumed the Rs 110 lakh crore National Infrastructure Pipeline that was launched in 2018.
(2) It aims to ensure integrated planning and implementation of infrastructure projects in the next four years, with a focus on expediting works on the ground, saving costs and creating jobs.
(3) It also aims to have 11 industrial corridors and two new defence corridors – one in Tamil Nadu and the other in Uttar Pradesh.
(4) It intends to bring together 16 infrastructure-related Ministries.
(5) None of the above

Ans: 1
Solution:
PM Gati Shakti National Master Plan
It aims to ensure integrated planning and implementation of infrastructure projects in the next four years, with a focus on expediting works on the ground, saving costs and creating jobs.
The Gati Shakti scheme will subsume the Rs 110 lakh crore National Infrastructure Pipeline that was launched in 2019.
Besides cutting logistics costs, the scheme is also aimed at increasing cargo handling capacity and reducing the turnaround time at ports to boost trade.
It also aims to have 11 industrial corridors and two new defence corridors – one in Tamil Nadu and the other in Uttar Pradesh. Extending 4G connectivity to all villages is another aim. Adding 17,000 km to the gas pipeline network is being planned.
It will help in fulfilling the ambitious targets set by the government for 2024-25, including expanding the length of the national highway network to 2 lakh kms, creation of more than 200 new airports, heliports and water aerodromes.
It intends to bring together 16 infrastructure-related Ministries.

3. More women than men in India are being diagnosed with cancer, a disease that is likely to strike 1.57 million in (A) from 1.46 million this year, the Indian Council of Medical Research (ICMR) has revealed in its bi-annual report, alarming the Indian health establishment.
Identify the year (A) in the above passage.
(1) 2025
(2) 2027
(3) 2030
(4) 2035
(5) 2040

Ans: 1
More women than men in India are being diagnosed with cancer, a disease that is likely to strike 1.57 million in 2025 from 1.46 million this year, the Indian Council of Medical Research (ICMR) has revealed in its bi-annual report, alarming the Indian health establishment.
These disturbing figures do not include data from two of India’s most populous states—Uttar Pradesh and Bihar. The national average of cancer cases for 2022 is 100.4 per 100,000, with a large number of women (105.4 per 100,000) being diagnosed with breast cancer, a preventable disease. By comparison, 95.6 men per 100,000 have been diagnosed with lung cancer.
The study was carried out at the ICMR’s National Centre for Disease Informatics and Research, Bengaluru. The scientist analysed National Cancer Registry Programme (NCRP) data for 2020-2022 to evaluate the burden of cancer in India in 2025. The report said that one in nine Indians is likely to develop cancer in their lifetime, by the time they hit their 74th year.
The top cancers in women were breast cancer, which had the highest incidence, followed by cervix, ovary and corpus uteri. Among men, cancers of the lung, mouth and tongue were common—all associated with smoking and other tobacco use.

4. Federal policy Think Tank NITI Aayog has received requests from over a dozen states to help set up similar institutions that will help them chart out their growth trajectory.
Which of the following initiatives is not an initiative of NITI Aayog?
(1) SDG India Index
(2) Composite Water Management Index
(3) Atal Innovation Mission
(4) SATH Project
(5) State of Environment Report

Ans: 5
Solution:
-Major Initiatives have been taken by NITI Aayog
-SDG India Index
-Composite Water Management Index
-Atal Innovation Mission
-SATH Project
-Aspirational District Programme
-School Education Quality Index
-District Hospital Index
-Health Index
-Agriculture Marketing And Farmer Friendly reform Index
-India Innovation Index
-Women Transforming India Awards
-Good Governance Index
-Women Entrepreneurship Platform (WEP)
-Strategy for New India at 75
-‘Methanol Economy’ programme
-e-AMRIT Portal

5. The World Development Report is an annual report published since 1978 by the International Bank for Reconstruction and Development or World Bank. What was the theme of the World Development Report 2022?
(1) Data for Better Lives
(2) Trading for Development in the Age of Global Value Chains
(3) LEARNING to Realize Education’s Promise
(4) Finance for an Equitable Recovery
(5) Risk and Opportunity

Ans: 4
Solution:
The World Bank publishes the World Development Report (WDR) annually. It has been published since 1978. The latest world bank report is the World Development Report 2022 which was released by the World Bank on February 15th. The report notifies the risks posed by the COVID-19 pandemic’s increased burden on the public and private debt as well as the urgent need for nations, especially developing countries, to strengthen their financial sectors.
The key role played by finance in the post-pandemic economic recovery is examined in the World Development Report 2022: Finance for an Equitable Recovery. It underlines the effects of the crisis that are most likely to have an impact on emerging countries and advocates a set of policies to reduce the interrelated financial risks brought on by the pandemic and guide economies toward a sustainable and fair recovery.

Mint StatiCA MCQs – 6th Dec, 2022

1. Consider the following statements and choose the incorrect one(s)!
A. The State of Extreme Weather Report – 2022 is published by NITI Aayog
B. CoP-21 was held in Copenhagen.
C. CoP-27 was held in 2022.
(1) A and B
(2) A and C
(3) B and C
(4) A, B and C
(5) None of the above

Ans: 1
Solution:
Extreme Weather: It is a weather event such as snow, rain, drought, flood, or storm that is rare for the place where it occurs.
The State of Extreme Weather Report – 2022: India saw extreme weather disasters on 241 of 273 days of 2022, claiming 2577 lives. This report by India’s environment think-tank Centre for Science and Environment, India.
In 2009, at the UNFCCC COP15 (held in Copenhagen), The developed country parties, to achieve meaningful mitigation actions and transparency on implementation, jointly set a target of USD 100 billion a year by 2020 to address the needs of developing countries. This was further endorsed in CoP-21 (Paris Agreement, 2015).
The 27th Conference of Parties (CoP-27) was held in Egypt under the aegis of the UNFCCC.

2. With the covid pandemic settling down, the Ayushman Bharat Pradhan Mantri Jan Arogya Yojana (AB – PM – JAY) may see a nominal increase in fund allocation in the upcoming union budget for 2023.
Which of the following statements is incorrect concerning AB-PMJAY?
(1) NHA is the apex implementing agency.
(2) In 2021-22, the scheme got ₹6,400 crore in the Union Budget.
(3) It was launched in 2018.
(4) It will provide a one-time cover of up to 5 lakhs per family.
(5) None of the above

Ans: 4
Solution:
With the covid pandemic settling down, the National Health Authority (NHA) may see a nominal increase in fund allocation in the upcoming union budget for 2023. NHA is the apex implementing agency of Ayushman Bharat Pradhan Mantri Jan Arogya Yojana (PM-JAY). In 2021-22, it got ₹6,400 crore.
India took a giant leap towards providing accessible and affordable healthcare to the common man with the launch of Ayushman Bharat – Pradhan Mantri Jan Aarogya Yojana (AB-PMJAY) by the Prime Minister, Shri Narendra Modi on 23rd September, 2018 at Ranchi, Jharkhand.
Ayushman Bharat- Pradhan Mantri Jan ArogyaYojana (PMJAY) will provide a cover of up to Rs. 5 lakhs per family per year, for secondary and tertiary care hospitalization.

3. The Centre has released a list of 117 United Nations bodies and other international organisations whose contribution to Indian entities will not be covered under the stringent Foreign Contribution (Regulation) Act (FCRA), 2010.
When was FCRA first enacted?
(1) 1971
(2) 1976
(3) 1980
(4) 2003
(5) 2010

Ans: 2
Solution:
The FCRA was enacted during the Emergency in 1976 in an atmosphere of apprehension that foreign powers were interfering in India’s affairs by pumping in funds through independent organisations.
It was amended in 2010 to “consolidate the law” on utilisation of foreign funds, and “to prohibit” their use for “any activities detrimental to national interest”
The law was amended again by the current government in 2020, giving the government tighter control and scrutiny over the receipt and utilisation of foreign funds by NGOs.

4. Business momentum in India’s services sector rose to a three-month high in November aided by accommodative demand conditions. The seasonally adjusted India Services PMI Business Activity Index increased to 56.4 in November from 55.1 in October. 
PMI Index is prepared by _ in India.
(1) S&P Global
(2) Bloomberg
(3) Fitch
(4) CARE
(5) KPMG

Ans: 1
Solution:
PMI is an index of the prevailing direction of economic trends in the manufacturing and service sectors.
It is an economic indicator, which is derived after monthly surveys of different companies.
There are two types of PMI — Manufacturing PMI and Services PMI.
In India, PMI is compiled by IHS Markit (acquired by S&P Global).

5. Shares of capital goods companies have widely outperformed the benchmark Sensex, as a revival in the cap-ex cycle has driven an increase in order flows to capital goods companies.
Which of the following statements is correct regarding capital goods?
(1) Capital goods can be both tangible and intangible assets.
(2) They can be used more than once.
(3) Natural resources can be capital goods.
(4) Capital goods can be used by end-consumers.
(5) All of the above

Ans: 2
Solution:
Capital goods are physical assets that a company uses in the production process to manufacture products and services that consumers will later use. Capital goods include buildings, machinery, equipment, vehicles, and tools. Capital goods are not finished goods, instead, they are used to make finished goods.
They are generally durable goods that can be used more than once. The most common capital goods are property, plants, and equipment (PPE). Natural resources not modified by human hands are not considered capital goods.

Mint StatiCA MCQs – 1st Dec, 2022

1. _(A)____occurs when, following a recession, different parts of the economy recover at different rates, times, or magnitudes. This contrasts with an even, uniform recovery across sectors, industries, or groups of people.
Identify (A) in the above excerpt.
(1) Z-shaped recovery
(2) V-shaped recovery
(3) U-shaped recovery
(4) K-shaped recovery
(5) L-shaped recovery.

Ans: 4
Solution:
A K-shaped recovery occurs when, following a recession, different parts of the economy recover at different rates, times, or magnitudes. This contrasts with an even, uniform recovery across sectors, industries, or groups of people. A K-shaped recovery leads to changes in the structure of the economy or the broader society as economic outcomes and relations are fundamentally changed before and after the recession.
A K-shaped recovery is one in which the performance of different parts of the economy diverges like the arms of the letter “K.”
The economic performance of different sectors, industries, and groups within an economy always differ to some extent, but in a K-shaped recovery, some parts of the economy may see strong growth while others continue to decline.
Unlike other letter-shaped descriptors, which focus on large aggregates, a K-shaped recovery is described in terms of data broken out across different parts of the economy.
The meaning of a K-shaped recovery really depends on the choice of how to disaggregate data across the economy.
Article

2. Index of Eight Core Industries measures combined and individual performance of production in selected eight core industries viz. Coal, Crude Oil, Natural Gas, Refinery Products, Fertilizers, Steel, Cement and Electricity. What is the weightage of the eight core industries in the Index of Industrial Production?
(1) 10.56 %
(2) 20.45 %
(3) 40.27 %
(4) 50.55 %
(5) 60.33 %

Ans: 3
Solution:
Index of Core Industries measures combined and individual performance of production in selected eight core industries viz. Coal, Crude Oil, Natural Gas, Refinery Products, Fertilizers, Steel, Cement and Electricity. 
The Eight Core Industries comprise 40.27 percent of the weight of items included in the Index of Industrial Production (IIP).
Article

3. India’s fiscal deficit widened to ₹7.58 trillion between April and October, accounting for 45.6% of the budget target for FY23, official data released showed. The government’s estimate of the difference between expenditure and revenue for the entire FY23 is ₹16.61 trillion or 6.4% of the GDP.
Which of the following statements is true about Fiscal Deficit?
A. The fiscal deficit can be calculated by marking out the difference between the total income and the total expenditure by the government.
B. Fiscal Deficit = (Revenue Expenditure + Capital Expenditure) – (Revenue Receipts + Capital Receipts)
C. NK Singh Committee (set up in 2016) recommended that the government should target a fiscal deficit of 3% of the GDP in years up to 31st March 2020, cut it to 2.8% in 2020-21 and to 2.0% by 2023.
Choose the correct option from the following:
(1) Only A and B
(2) Only A
(3) Only C
(4) Only A and C
(5) All of the above

Ans: 1
Solution:
Fiscal Deficit is the difference between the total revenue and total expenditure of the government. The fiscal balance of a country is calculated by its government’s revenue followed by its expenditure in the provided financial year, the situation where the government expenses increase more than the revenue in a year is a fiscal deficit.
The fiscal deficit can be calculated by marking out the difference between the total income and the total expenditure by the government.
The total income of the government is calculated by including all taxes, non-debt capital receipts, and other ways of revenue except for borrowings. To calculate the fiscal deficit
Fiscal Deficit = (Revenue Expenditure + Capital Expenditure) – (Revenue Receipts + Capital Receipts)
In the simplified form, the formula reads out as: Fiscal Deficit = Total expenditure — Total receipts excluding borrowings
Most of the economies around the globe, including India, run under a fiscal deficit, which means the expenditure by the government is more than its income.
NK Singh Committee (set up in 2016) recommended that the government should target a fiscal deficit of 3% of the GDP in years up to 31st March 2020, cut it to 2.8% in 2020-21 and to 2.5% by 2023.
Article

4. The Sensex settled above the 63,000 level for the first time recently, extending its winning momentum to the seventh day amid a largely positive trend in global markets and continuous foreign fund inflows. SENSEX is the benchmark index of the __ in India.
Fill in the Blanks.
(1) BSE
(2) NSE
(3) NCDEX
(4) MCX
(5) ICEX

Ans: 1
Solution:
SENSEX is the benchmark index of the Bombay Stock Exchange (BSE) in India. The SENSEX index is made up of 30 of the BSE’s largest and most actively traded stocks, and it serves as a gauge for the Indian economy. It is a market capitalization-weighted and float-adjusted. The Sensex is India’s oldest stock index, having been established in 1986 by Standard & Poor’s (S&P).
Article

5. Under Section 45ZB of the amended (in 2016) RBI Act, 1934, the central government is empowered to constitute a six-member Monetary Policy Committee (MPC).
Which is the Quorum for the meeting of the Monetary Policy Committee (MPC)?
(1) 1
(2) 2
(3) 3
(4) 4
(5) 5

Ans: 4
Solution:
Under Section 45ZB of the amended (in 2016) RBI Act, 1934, the central government is empowered to constitute a six-member Monetary Policy Committee (MPC).
Composition: Section 45ZB says the MPC shall consist of 6 members:
RBI Governor as its ex officio chairperson,
Deputy Governor in charge of monetary policy,
An officer of the Bank to be nominated by the Central Board,
Three persons to be appointed by the central government.
The MPC is required to meet at least four times in a year. The quorum for the meeting of the MPC is four members. Each member of the MPC has one vote, and in the event of an equality of votes, the Governor has a second or casting vote.
Article

Mint StatiCA MCQs – 29th Nov, 2022

1. Consider the following statements and choose the incorrect one(s)!
A. Tax collection is a part of the revenue receipt.
B. The highest tax slab under the GST regime is 24%
C. Income up to 3 lakh per annum is exempted from income tax.
(1) A and B
(2) A and C
(3) B and C
(4) A, B and C
(5) None of the above

Ans: 3
Solution:
The money received by the government through taxes is known as tax revenue (a part of revenue receipt) and may be utilized for a broad spectrum of purposes such as infrastructure development in the form of roads, railways, bridges, dams etc., public healthcare and education, defence and civil services, to name a few.
Indirect taxes are collected indirectly by the government and are charged on availing or purchasing goods and services. They are paid by consumers to service providers who then have to pay the same amount to the government or local bodies. The system previously included a wide array of taxes including the Value Added Tax, excise duty, customs charges, service tax and sales tax. With the enforcement of the Goods and Services Tax Bill in July 2017, many of these were consolidated into a single system of taxation with five slabs of tax rates, namely – 0%, 5%, 12%, 18% and 28%.

Link to the article

2. Farmers in China are now growing a perennial variety of rice which does not need to be planted every year. The invention could transform rice farming by making it climate-friendly, besides using less of labour and other inputs. 
Which of the following statements is correct concerning rice production and trade?
(1) India is the largest producer in the world, followed by China.
(2) China accounts for almost 40% of global rice export.
(3) Indian Institute of Rice Research is located at Cuttack, Odisha.
(4) Supply of fortified rice in every concerned Indian Government scheme was announced in 2021.
(5) All of the above

Ans: 4
Solution:
India is the world’s second-largest rice producer, after China, and the largest exporter with a 40% share in global trade.
The Hon’ble PM in his address on the 75th Independence Day (15th August, 2021), announced the supply of Fortified Rice in every scheme of the Government of India throughout the country by 2024 in a phased manner. Since then, the initiative has made good progress during the last one year.
Link to the article

3. Many Indians with H-1B visas have been living in the US for years, awaiting permanent residency (the coveted ‘green card’).
What is the H-1 B visa?
(1) Study visa
(2) Work visa
(3) Tourist visa
(4) Diplomatic visa
(5) All of the above

Ans: 2
Solution:
In 1952, after the US started expanding its presence in the science, technology, engineering and mathematics disciplines, it felt the need to hire quality workers who could help the country achieve innovation in these areas at reasonable costs. The need to hire workers paved way for the introduction of the H-1 work visa system in US.
-This work visa system was further subdivided into H-1B, H-2B, L1, O1, and E1 visas, depending on the qualification required and the area for which workers were sought.
-Of these, the H-1B visa remains the most popular due to the relatively better wage chance it offers.
Link to the article

4. The larger companies in India are a fraction of their global counterparts8 in terms of both revenues and market cap, thus presenting a sizeable opportunity. This will come about as growth in India’s per capita GDP likely leads to an inflection point across various sectors.
How does SEBI identify large-cap companies?
(1) Top 100 listed companies in terms of market cap
(2) Top 50 listed companies in terms of market cap
(3) Top 150 listed companies in terms of market cap
(4) Top 75 listed companies in terms of market cap
(5) Top 25 listed companies in terms of market cap

Ans: 1
Solution:
As per market regulator SEBI’s categorization, the top 100 out of 5000 companies by market cap are classified as large-cap stocks. They are just 2% of the total listed universe but contribute approximately 70% of total equity market capitalization. Large-cap companies are typically market or sector leaders with an established and proven track record, and have better access to resources like capital and talent pool. Their businesses and balance sheets are quite stable, compared to small- and mid-cap companies. These characteristics have led to more stable operational performance by them during economic downturns.
Link to the article

5. What is long-term capital gains tax rate for equity share in India?
(1) 5%
(2) 10%
(3) 15%
(4) 20%
(5) 25%

Ans: 2
Solution:
Since 01 April 2018, long-term capital gains on the sale of listed equity shares have been taxable at the rate of 10% if the capital gain is more than Rs. 1 lakh in a financial year. A surcharge is also applicable on this LTCG that varies from 10% to 37%, depending on the investor’s income.
Link to the article

Mint StatiCA MCQs – 28th Nov, 2022

1. Consider the following statements and choose the incorrect one(s)!
A. Large Cap companies are those which have a market share of Rs 5,000 Core or more.
B. Large cap stocks provide more liquidity than mid cap and small cap stocks.
C. Mid cap stocks are more volatile than small cap stocks.
(1) A and B
(2) A and C
(3) B and C
(4) A, B and C
(5) None of the above

Ans: 2
Solution:
Market cap—or market capitalisation—refers to the total value of all a company’s shares of stock. It is calculated by multiplying the price of a stock by its total number of outstanding shares.
Large-cap companies are businesses that are well-established and have a significant market share. Large-cap companies have market caps of Rs 20,000 crore or more. These stocks are less volatile in comparison to mid-cap (Market cap: 5,000 Crore – 20,000 crore) and small-cap (Market cap: less than 5,000 Crore) stocks. The lower volatility makes them less risky. Likewise, Mid cap stocks are less volatile than small-cap stocks.
Large-cap funds invest mainly in blue-chip companies. Such funds inherently have certain advantages: The companies they invest in are large and stable businesses with the capability to weather market volatility. There is a high demand for these stocks, which makes them highly liquid. By similar logic, mid cap stocks are more liquid than small cap stocks.
Link to the article

2. In nine of the past 17 years, the mid-cap index has outdone the Sensex. The small-cap index has done so eight times. Bad years tend to be followed by years of outsized returns for mid-caps and small-caps, even greater than the Sensex.
Which of the statements is correct?
(1) Alpha and beta measure the profitability of an investment
(2) Beta measures the profitability of an investment while Alpha measures the volatility of a stock with respect to the benchmark index.
(3) Alpha measures the profitability of an investment while Beta measures the volatility of a stock with respect to the benchmark index.
(4) Alpha and beta measure the Volatility of an investment
(5) None of the above

Ans: 3
Solution:
Alpha is a measure of the success of your investment. It calculates how much a stock or fund has outperformed the general market. This follows the principle that when the market rises over time, it adds value to most of the stocks. This is called market return, and is often adjusted with risk. However, there are many stocks which outperform, usually because of higher earnings. Their return is higher than the market. Alpha calculates this difference, by comparing your stock or fund with a benchmark index. It, thus, represents how much value has added or subtracted to total returns.
While alpha deals with the rewards of the investment, beta gives perspective about the volatility or risks involved. Just like alpha, beta too established by comparing with a benchmark index. It measures the volatility in a stock’s price and is denoted in positive or negative figures. A positive beta value means that the stock moves in the same direction as the index. A negative value indicates an opposite direction, that is, the stock rises when the market falls and vice versa. Also, a beta value over 1 means the stock is more volatile than the market. For example, if the beta value is 1.1, the share price is like to swing more by 10% than the index. A value lower than 1 means the stock price does not fluctuate as much.
Link to the article

3. Consider the following statements and choose the incorrect one(s).
A. A microfinance loan is defined as a collateral-free loan given to a household having annual household income up to ₹3,00,000.
B. For this purpose, the household shall mean an individual family unit, i.e., husband, wife and their unmarried children.
C. All collateral-free loans, irrespective of end use and mode of application/ processing/ disbursal (either through physical or digital channels), provided to low-income households, i.e., households having annual income up to ₹3,00,000, shall be considered as microfinance loans.
(1) None of the statements
(2) A and B
(3) B and C
(4) A and C
(5) A, B, and C

Ans: 1
Solution:
A microfinance loan is defined as a collateral-free loan given to a household having annual household income up to ₹3,00,000. For this purpose, the household shall mean an individual family unit, i.e., husband, wife and their unmarried children.
All collateral-free loans, irrespective of end use and mode of application/ processing/ disbursal (either through physical or digital channels), provided to low-income households, i.e., households having annual income up to ₹3,00,000, shall be considered as microfinance loans.
To ensure collateral-free nature of the microfinance loan, the loan shall not be linked with a lien on the deposit account of the borrower.
The Regulated Entity for microfinance shall have a board-approved policy to provide the flexibility of repayment periodicity on microfinance loans as per borrowers’ requirement.
Link to the article

4. In FY23, the Centre sought to spend ₹4.45 trillion under centrally sponsored schemes, up from ₹4.15 trillion in FY22. The spending is higher in the case of central sector schemes, which are fully funded by the union government, at ₹11.8 trillion this fiscal.
Which Article of the Indian Constitution provides the legal premise for the Union Budget?
(1) Article 380
(2) Article 110
(3) Article 280
(4) Article 112
(5) Article 157

Ans: 4
Solution:
According to Article 112 of the Indian Constitution, the Union Budget of a year is referred to as the Annual Financial Statement (AFS).
It is a statement of the estimated receipts and expenditure of the Government in a Financial Year (which begins on 1st April of the current year and ends on 31st March of the following year).
Link to the article

5. Every bull market needs a theory. And two months can be a long time in stock markets. The Dow Jones Industrial Average, America’s premier stock market index, was at 28,726 points as of September-end. It has rallied by around 20% to close at 34,347 points as of Friday.
This is in an environment where the US Federal Reserve has been raising the federal funds rate, its key short interest rate, to rein in decadal high retail inflation in the US. In October, the inflation stood at 7.8%.
Usually, what happens when a central bank raises interest rate?
(1) Stock markets witness a sharp growth.
(2) Newly issued bonds are offered at a lower interest rate.
(3) Prices of old bonds go down.
(4) The corporate profits increase.
(5) All of the above

Ans: 3
Solution:
The interest rate is a percentage that a lender of capital charges on the borrower’s debt repayment, usually paid out over a specified period. For example, when you take a loan from a bank, you’ll be charged interest on the principal amount you borrowed.
When the market interest rate rises, the interest rate being offered on new bonds is kept higher than on the old bonds. So you need the yield to match in order for people to be willing to buy the old bonds. Otherwise, they’re just going to stick with the new bonds. For that to happen, the price of the old bonds has to go down. Likewise, when the market interest rate falls, new bonds are issued at a lower cost of debt. So, the old bonds seem a more valuable proposition and their prices increase.
A fall in interest rates lowers the yield on bonds and increases it for equities. This is because corporate profits improve due to a drop in interest cost and an improvement in the future outlook, led by better CAPEX opportunities. Likewise, a rise in interest rates causes a decline in the stock market.
Link to the article

Mint StatiCA MCQs – 25th Nov, 2022

1. Consider the following statements and choose the incorrect one(s)!
A. NBFCs are registered under Section 3 of the Companies Act 2013.
B. NBFCs carry out activities related to financial services, agriculture and industrial services.
C. The Minimum investment limit for AIFs is Rs 20 Lakh.
(1) A and B
(2) A and C
(3) B and C
(4) A, B and C
(5) None of the above

Ans: 3
Solution:
A Non-Banking Financial Company (NBFC) is a company registered under Section 3 of the Companies Act, 1956 engaged in the business of loans and advances, acquisition of shares/stocks/bonds/debentures/securities issued by Government or local authority or other marketable securities of a like nature, leasing, hire-purchase, insurance business, chit business but does not include any institution whose principal business is that of agriculture activity, industrial activity, purchase or sale of any goods (other than securities) or providing any services and sale/purchase/construction of immovable property.
AIF: The minimum investment limit is Rs. 1 crore for investors, whereas the minimum investment amount for directors, employees, and fund managers are Rs. 25 lakh.
AIFs come with a minimum lock-in period of three years.
The number of investors in every scheme is restricted to 1000, except angel funds. Where the number of investors goes up to 49.
Link to the article

2. Hopes of slower US rate hikes and cooling crude prices cheered Indian stocks, with the benchmark indices closing at record highs on Thursday. All sectoral indices gained, and the Sensex and the Nifty rose 1.24% and 1.19%, respectively, to close at all-time highs of 62,272.68 and 18,484.10. During the day, the Sensex hit a new high of 62,412.33, while the Nifty was just a few points away from its record high of 18,604.45.
What is the base year for NIFTY Index?
(1) 1991
(2) 1992
(3) 1993
(4) 1994
(5) 1995

Ans: 5
Solution:
Nifty 50 is primarily used for benchmarking index funds, index-based derivatives, and fund portfolios. Index Services and Products Limited (IISL), a subsidiary of NSE, owns and manages Nifty. Furthermore, the base value of Nifty 50 is 1000 and 1995 is considered as its base year for calculation of the index per float-adjusted market capitalisation methodology.
Link to the article

3. Consider the following statements and choose the incorrect one(s).
A. The Chief Economic Adviser enjoys a Union Cabinet Minister’s Rank.
B. The CEA reports directly to the Prime Minister.
C. The CEA is head of the Economic Division of the Department of Economic Affairs, Ministry of Finance
(1) None of the statements
(2) A and B
(3) B and C
(4) A and C
(5) A, B, and C

Ans: 2
Solution:
About CEA –
-The Chief Economic Adviser (CEA) is a post in Government of India and is equivalent to rank of Secretary to the Government of India.
-The CEA is the ex-officio cadre controlling authority of the Indian Economic Service.
-The CEA is head of Economic Division of the Department of Economic Affairs, Ministry of Finance, Government of India.
-Until 2009, the CEA’s position was a Union Public Service Commission appointment and until the 1970s almost all CEAs were members of the Indian Economic Service.
-The CEA reports directly to the Minister of Finance.
Link to the article

4. India’s green hydrogen plans would play a major role in achieving its goal of net zero emissions and becoming a developed nation by 2047.
Green Hydrogen Policy was unveiled by _ in ___ respectively.
(1) Ministry of Power, 2021
(2) Prime Minister, 2021
(3) Ministry of Power, 2022
(4) Prime Minister, 2022
(5) None of the above

Ans: 3
Solution:
The Ministry of Power (MoP) recently unveiled the first part of India’s much awaited Green Hydrogen Policy on February 17, 2022. The policy is one of the key outcomes of the National Hydrogen Mission which was launched by the Hon’ble Prime Minister, Shri Narendra Modi, on India’s 75th Independence Day last year. It marks the culmination of months of efforts across multiple ministries and stakeholder groups, and affirms India’s intent to be a global green hydrogen hub.
Most large economies including India have committed to net zero targets. Transition to Green Hydrogen and Green Ammonia is one of the major requirements for reduction of emissions, especially in the hard to abate sectors. Government of India have had under consideration a number of policy measures in order to facilitate the transition from fossil fuel based feed stocks to Green Hydrogen / Green Ammonia both as energy carriers and as chemical feed stock for different sectors.
Green Hydrogen / Green Ammonia shall be defined as Hydrogen / Ammonia produced by way of electrolysis of water using Renewable Energy; including Renewable Energy which has been banked and the Hydrogen/Ammonia produced from biomass.
Link to the article

5. Speaking at a panel discussion on ‘India’s Energy Transition Pathways and its Global Impact’, Suman Sharma, managing director, Solar Energy Corp. of India (SECI) noted that government intervention is important to achieve energy transition, and, on this front, the government machinery is working in tandem to facilitate this transition.
Which of the following statements is/are correct?
(1) SECI is under the administrative control of Ministry of Power
(2) It was set up in 2015
(3) It was set up to implement National Solar Mission
(4) It is one of the four Central Public Sector Undertakings dedicated to solar energy sector.
(5) All of the above

Ans: 3
Solution:
-About:
oSolar Energy Corporation of India ltd” (SECI) is a CPSU under the administrative control of the Ministry of New and Renewable Energy (MNRE), set up on 20th Sept, 2011.
-Vision:
1.To build ‘Green India’ through harnessing abundant solar radiation and to achieve energy security for the country.
-Objective:
1.It was set up to facilitate the implementation of National Solar Mission (NSM) and achievement of targets set therein. It is the only CPSU dedicated to the solar energy sector.
2.It is engaged in promotion and development of various renewable energy resources, especially solar energy, trading of power, R&D etc.
Link to the article

Mint StatiCA MCQs – 24th Nov, 2022

1. Consider the following statements and choose the incorrect one(s)!
A. Planning Commission worked with the Bottom-Up approach.
B. Jawaharlal Nehru is called the Father of Five-Year Plans.
C. Guljari Lal Nanda was the first Deputy-Chairman of the Planning Commission.
(1) A and B
(2) A and C
(3) B and C
(4) A, B and C
(5) None of the above

Ans: 1
Solution:
The Planning Commission was established on 15 March 1950, with Prime Minister Jawaharlal Nehru as the chairman. The Planning Commission did not derive its creation from either the Constitution or statute but was an arm of the Central/Union Government. The first Deputy Chairman was Guljari Lal Nanda.
Five-Year Plans were laid out by the Planning Commission as a set of policies to be implemented over a period of five years. The first Five-year Plan was launched in 1951. M Vishveshvaraiya is known as the Father of Five-Year Plans.
The main problem with Planning Commission was that it worked as per the top-down approach. It means that states were not necessarily consulted over policy formulation. This limited the potential of the Planning Commission with respect to the country’s economic development.
Link to the article

2. After 10 years of negotiations, India and Australia have finally agreed an interim free trade deal. The India-Australia Economic Cooperation and Trade Agreement (ECTA) is expected to double trade between the two countries in the next five years.
Which of the following statements is/are correct?
(1) ECTA is more comprehensive than CEPA.
(2) A Free Trade Agreement is more comprehensive than CEPA.
(3) Australia is the first country with whom India will sign a CECA.
(4) CEPA looks deeper at the regulatory aspects of trade than an FTA.
(5) All of the above

Ans: 4
Solution:
The free trade pact, officially called the Australia-India Economic Cooperation and Trade Agreement, was signed in the presence of Prime Minister Narendra Modi and Australian Prime Minister Scott Morrison. It was signed in just a little over six months after the re-launch of negotiations in September 2021. The countries are now expected to towards a wider Comprehensive Economic Cooperation Agreement to include more elements such as agriculture trade.
FTAs are arrangements between two or more countries or trading blocs that primarily agree to reduce or eliminate customs tariff and non tariff barriers on substantial trade between them. FTAs, normally cover trade in goods (such as agricultural or industrial products) or trade in services (such as banking, construction, trading etc.). FTAs can also cover other areas such as intellectual property rights (IPRs), investment, government procurement and competition policy, etc.
CECA is a kind of free trade pact that aims to provide an institutional mechanism to encourage and improve trade between the two countries.
CECA generally covers negotiation on trade tariffs and TRQ (tariff rate quota) rates only. It is not as comprehensive as CEPA. India has signed CECA with Malaysia and Singapore.
As the name suggests, CEPA is more comprehensive and ambitious than an FTA in terms of partnership across a wider coverage of areas and the type of commitments. While a traditional FTA focuses mainly on goods; a CEPA is more ambitious in terms of a holistic coverage of many areas like services, investment, IPR, government procurement, disputes etc. Secondly, CEPA looks deeper at the regulatory aspects of trade than an FTA.
Link to the article

3. Consider the following statements and choose the incorrect one(s).
A. ICMR was established in 1949.
B. Earlier, it was known as Indian Research Fund Association.
C. It is funded by the Department of Biotechnology.
(1) None of the statements
(2) A and B
(3) B and C
(4) A and C
(5) A, B, and C

4
Solution:
The Indian Council of Medical Research (ICMR), New Delhi, the apex body in India for the formulation, coordination and promotion of biomedical research, is one of the oldest medical research bodies in the world. It was formed in 1911 when it was known as Indian Research Fund Association (IRFA). In 1949, IRFA was renamed ICMR.
The ICMR has always attempted to address itself to the growing demands of scientific advances in biomedical research on the one hand, and to the need of finding practical solutions to the health problems of the country, on the other. The ICMR has come a long way from the days when it was known as the IRFA, but the Council is conscious of the fact that it still has miles to go in pursuit of scientific achievements as well as health targets.
It is funded by the Government of India through the Department of Health Research, Ministry of Health & Family Welfare.
Link to the article

4. Paul Brody, global blockchain leader at EY, believes that this might be the last crypto winter ever. In an interview, Brody also spoke about government-based central bank digital currencies (CBDCs), and how they don’t serve a purpose.
Which is the second most traded cryptocurrency?
(1) Dogecoin
(2) Steemcoin
(3) Ethereum
(4) Tether
(5) Ripple

3
Solution:
Bitcoin: Created in 2009 by someone under the pseudonym Satoshi Nakamoto, Bitcoin (BTC) is the original cryptocurrency. As with most cryptocurrencies, BTC runs on a blockchain, or a ledger logging transaction distributed across a network of thousands of computers. Because additions to the distributed ledgers must be verified by solving a cryptographic puzzle, a process called proof of work, Bitcoin is kept secure and safe from fraudsters.
Ethereum: Both a cryptocurrency and a blockchain platform, Ethereum is a favourite of program developers because of its potential applications, like so-called smart contracts that automatically execute when conditions are met and non-fungible tokens (NFTs).
Ethereum is historically the second most popular cryptocurrency however it is very different from Bitcoin.
Link to the article

5. To bring in operational efficiency, Indian stock exchanges said that all stocks, on which derivatives contracts are available, will be transitioned to the T+1 (trade plus one day) settlement cycle from January 2023.
Which is the first country to implement T+1 settlement cycle?
(1) United States
(2) United Kingdom
(3) India
(4) China
(5) Japan

4
Solution:
From February 25, 2022, India will become only the second country after China to move to the T+1 stock settlement mechanism in a phased manner.
T+1 (trading+1day) means settlement of equity transactions in less than 24 hours from the day of transaction. It will make India the fastest stock market in the world to settle equity trades. Currently, India follows T+2 cycle, where it takes 48 hours or even more for the shares to be transferred into the client account in case of purchases.Mint StatiCA

Mint StatiCA MCQs – 23rd Nov, 2022

1. Consider the following statements and choose the incorrect one(s)!
A. Justice K. S. Puttaswamy Case was related to the Constitutional validity of the Aadhar Card.
B. BN Krishna Panel was constituted to improve Aadhar Card features.
C. Right to Nominate means the provision to nominate somebody to collect data from others on one’s behalf.
(1) A and B
(2) A and C
(3) B and C
(4) A, B and C
(5) None of the above

Ans: 3
Solution:
In August 2017, a nine-judge bench of the Supreme Court in Justice K. S. Puttaswamy (Retd) Vs Union of India unanimously held that Indians have a constitutionally protected fundamental right to privacy that is an intrinsic part of life and liberty under Article 21. The petition filed by Justice Puttaswamy was a case which sought to challenge the constitutional validity of the Aadhaar card scheme.
The data protection Bill has been in the works since 2018 when a panel led by Justice B N Srikrishna had prepared a draft version of the Bill.  It is India’s first attempt to domestically legislate on the issue of data protection.
In the latest Draft of the Digital Personal Data Protection Bill 2022, there is right to nominate an individual who will exercise the rights to consent, erase and access to information in the event of somebody’s death or incapacity.
Link to the article

2. The government is working on a production-linked incentive (PLI) scheme worth ₹10,000 crore for chemicals and petrochemicals industries, two people aware of the matter said, as the country aims to triple its capacity to manufacture these key ingredients by 2040.
When the PLI scheme was launched, which of the sectors it did not cover?
(1) Mobile manufacturing
(2) Electric manufacturing
(3) Pharmaceuticals
(4) Medical devices
(5) Fast moving consumer goods

Ans: 5
Solution:
Production linked incentive (PLI) schemes were first introduced in India in March 2020, targeting three industries – mobile manufacturing and electric components, pharmaceutical (critical key starting materials/active pharmaceutical ingredients), and medical device manufacturing. The PLI concept has since expanded with schemes rolled out for multiple sectors to boost India’s manufacturing capabilities and encourage export-oriented production. The PLI schemes aim to develop capacities in the local supply chain, introduce new downstream operations, and incentivize investments into high-tech production.
Link to the article

3. Consider the following statements and choose the incorrect one(s).
A. IPO and Stock markets come under the primary market.
B. FPO comes under the secondary market.
C. Rights Issues and Private Placement come under the primary market.
(1) None of the statements
(2) A and B
(3) B and C
(4) A and C
(5) A, B, and C

Ans: 2
Solution:
The Capital market represents the “Primary Market” and the “Secondary Market. The capital market has two interdependent and inseparable segments, the new issuers (the primary market) and stock (secondary) market. The primary market is used by issuers for raising fresh capital from the investors by making initial public offers or rights issues or offers for sale of equity or debt. An active secondary market promotes the growth of the primary market and capital formation, since the investors in the primary market are assured of a continuous market where they have an option to liquidate their investments.
A corporate may raise capital in the primary market by way of an initial public offer, rights issue or private placement. An Initial Public Offer (IPO) is the selling of securities to the public in the primary market. It is the largest source of funds with long or indefinite maturity for the company.
An IPO is an important step in the growth of a business. It provides a company access to funds through the public capital market. An IPO also greatly increases the credibility and publicity that a business receives. In many cases, an IPO is the only way to finance quick growth and expansion. In terms of the economy, when a large number of IPOs are issued, it is a sign of a healthy stock market and economy.
When the company makes its first IPO to the public, the relationship is directly between the company and investors, and the money flows to the Company as its “Share Capital”. Shareholders thus become owners of the Company through their participation in the Company’s IPO and have ownership rights over the company. This is the largest source of funds for a company, which enables the company to create “Fixed Assets” which will be employed in the course of the business. The shareholders of the Company are free to exit their investment through the secondary market.
Link to the article

4. Telcos have asked the government to provide a clear definition of adjusted gross revenue based on which the industry’s levies and charges are calculated, besides seeking refund of ₹32,000 crore input tax credit on goods and service tax, as part of its key demands ahead of the Union Budget.  Which of the statements is/are correct regarding input tax credit (ITC)?
(1) Input tax is charged when the goods reaches the end consumer.
(2) ITC helps avoid cascading of taxes.
(3) ITC is available as a deduction from Gross profit.
(4) ITC is not applicable to services.
(5) All of the above

Ans: 2
Solution:
Input Tax Credit refers to the tax already paid by a person at the time of purchase of goods or services and which is available as a deduction from tax payable. So, ITC is a mechanism to avoid cascading of taxes. Cascading of taxes, in simple language, is ‘tax on tax’.
For eg-  A trader purchases a good worth rs 100 and pay a tax of 10% on it. And now this trader sold such goods at Rs. 150 and collect tax of Rs. 15 from the buyer. Now the trader has to pay Rs. 15 to government but he had already paid Rs. 10, so this Rs. 10 is ITC of the trader and will be allowed as deduction from tax payable and he has to pay net Rs. 5 as tax.
Link to the article

5. In the OECD’s estimation, the world economy will grow just 3.1% this year, down sharply from a robust 5.9% in 2021. OECD is made up of _ members.
(1) 32
(2) 34
(3) 36
(4) 38
(5) 40

Ans: 4
Solution:
The Organisation for Economic Co-operation and Development (OECD) is an international organisation that works to build better policies for better lives. Our goal is to shape policies that foster prosperity, equality, opportunity and well-being for all. We draw on 60 years of experience and insights to better prepare the world of tomorrow.
The OECD, made up of 38 member countries and headquartered in Paris, works to promote international trade and prosperity and issues periodic reports and analyses.
Link to the article

Mint StatiCA MCQs – 22nd Nov, 2022

1. Consider the following statements and choose the incorrect one(s)!
A. Friendshoring means setting up supply chain facilities in own country with the help of friendly countries.
B. According to the last World Bank’s Logistics Performance Index published in 2018, India was ranked 44th.
C. When launched, PLI Scheme covered four sectors.
(1) A and B
(2) A and C
(3) B and C
(4) A, B and C
(5) None of the above

Ans: 2
Solution:
Friendshoring is a strategy where a country sources the raw materials, components and even manufactured goods from countries that share its values. The dependence on the countries considered a “threat” to the stability of the supply chains is slowly reduced. So, it is shorthand for the practice of relocating supply chains to countries where the risk of disruption from political chaos is low.
In 2018, India was ranked 44th in the World Bank’s Logistics Performance Index.
Production linked incentive (PLI) schemes were first introduced in India in March 2020, targeting three industries – mobile manufacturing and electric components, pharmaceutical (critical key starting materials/active pharmaceutical ingredients), and medical device manufacturing. The PLI concept has since expanded with schemes rolled out for multiple sectors to boost India’s manufacturing capabilities and encourage export-oriented production.
Link to the article

2. If India’s top food safety regulator has its way, you may soon be watching out for “star ratings” on the packets of chips or chocolate when you step into the market. This regulator comes under the administrative control of __.
(1) Ministry of Health & Family Welfare
(2) Ministry of Corporate Affairs
(3) Ministry of Home Affairs
(4) Ministry of Food Processing Industries
(5) Ministry of Commerce & Industry

Ans: 1
Solution:
Food Safety and Standards Authority of India is a statutory body established under the Ministry of Health & Family Welfare, Government of India. The FSSAI has been established under the Food Safety and Standards Act, 2006, which is a consolidating statute related to food safety and regulation in India.
Link to the article

3. Consider the following statements concerning fiscal deficit and choose the incorrect one(s).
A. NARCL is one of the three pillars of the bad debt resolution plan of the Union Government, the other two are IDRCL and IBA.
B. IDRCL will pick up the bad loans above the prescribed threshold and NARCL will manage these assets acquired by the IDRCL.
C. Public sector banks will have up to 49% stake in the IDRCL.
(1) None of the statements
(2) A and B
(3) B and C
(4) A and C
(5) A, B, and C

Ans: 2
Solution:
A.NARCL is one of the two legs of the proposed Bad Bank by the government.
B.The other one is an asset management company called – India Debt Resolution Company Ltd (IDRCL).
C.The NARCL will pick up bad loans above a certain threshold from banks and would aim to sell them to prospective buyers of distressed debt.
.The NARCL will also be responsible for valuing bad loans to determine at what price they would be sold.
.The bad bank would provide government receipts to banks as it takes on non-performing assets from their books.
.State-owned banks will hold 51% stake, while FIs or debt management companies will hold 49%.
D.The IDRCL will be a service company or operational entity, which will manage the assets acquired by NARCL.
.It will enrol market professionals and turnaround experts in managing the bad assets in an attempt to turn them around.
.Public Sector Banks (PSBs) and Public FIs will hold a maximum of 49% stake in IDRCL, and the rest will be held by private-sector lenders.
Link to the article

4. E-commerce companies and eateries will have to ensure that the online reviews of their products and services are genuine, and their authors verifiable, under a new government framework to counter fake reviews released on Monday. The new guidelines, outlined by the Bureau of Indian Standards (BIS) will take effect on 25 November. In that context, choose the incorrect statement.
(1) The Central Consumer Protection Authority is established as per the provisions laid out in the Consumer Protection Act, 2019
(2) These provisions are mentioned in Section 10 of the Act.
(3) The Commission has three members.
(4) The Investigation Wing of the Commission is headed by the Chief Commissioner.
(5) At the district level, the District Magistrate is the Supreme authority, not the Superintendent of police.

Ans: 4
Solution:
The Consumer Protection Act, 2019 has come into force from 20th July, 2020. As provided in section 10 of the Act, the Central Consumer Protection Authority (CCPA) has been established w.e.f. 24th July, 2020.
The objective of the Central Consumer Protection Authority (CCPA) is to promote, protect and enforce the rights of consumers as a class. It will be empowered to conduct investigations into violation of consumer rights and institute complaints / prosecution, order recall of unsafe goods and services, order discontinuation of unfair trade practices and misleading advertisements, impose penalties on manufacturers/endorsers/publishers of misleading advertisements.
Composition of the Commission
.It will have a Chief Commissioner as head, and only two other commissioners as members — one of whom will deal with matters relating to goods while the other will look into cases relating to services.
.The CCPA will have an Investigation Wing that will be headed by a Director General.
.District Collectors too, will have the power to investigate complaints of violations of consumer rights, unfair trade practices, and false or misleading advertisements.
Link to the article

5. The adoption of modern machines and equipment by the farmers depends on various factors such as socioeconomic conditions, geographical conditions, crop grown, irrigation facilities etc. However, the Government of India supports and facilitates the State Governments through various Centrally Sponsored and Central Sector Schemes to promote agriculture throughout the country and infuse modern technologies into the agriculture sector.
Sub-Mission on Agricultural Mechanization was launched in __.
(1) 2012-13
(2) 2013-14
(3) 2014-15
(4) 2015-16
(5) 2016-17

Ans: 3
Solution:
Ministry of Agriculture and Farmers Welfare has launched a Sub-Mission on Agricultural Mechanization (SMAM) in 2014-15 with the objectives of increasing the reach of farm mechanization to small and marginal farmers and to the regions & difficult area where farm power availability is low. To boost up mechanization in the agriculture sector improved agricultural implements and machinery are essential inputs for modern agriculture that enhance the productivity of crops besides reducing human drudgery and cost of cultivation.
Link to the article

Mint StatiCA MCQs – 21st Nov, 2022

1. Consider the following statements and choose the incorrect one(s)!
A. FDIs take a short-term position in the financial market.
B. The maximum investment that can be made through FPI is 20% of the post-issue share capital of a company.
C. FPIs are more volatile than FDIs.
(1) A and B
(2) A and C
(3) B and C
(4) A, B and C
(5) None of the above

Ans: 1
Solution:
FDI refers to a foreign investment in which the investor holds a long-term interest in an enterprise in another country. It involves establishing a direct business interest in a foreign country, such as buying or establishing a manufacturing business, building warehouses, or buying buildings.
Foreign Direct Investment (FDI) is the investment through capital instruments by a person resident outside India:
(a) in an unlisted Indian company; or
(b) in 10 percent or more of the post-issue paid-up equity capital on a fully diluted basis of a listed Indian company.
Foreign Portfolio Investment is any investment made by a person resident outside India in capital instruments where such investment is:
(a) less than 10 percent of the post-issue paid-up equity capital on a fully diluted basis of a listed Indian company or
(b) less than 10 percent of the paid-up value of each series of capital instruments of a listed Indian company.
The nature of liquidity not only determines the amount of trade but also the volatility involved. The volatility associated with FPI is greater than that of FDI. This is because FDI needs a longer investment horizon and narrows down the scope of liquidity and further reduces risks in the investment. 
On the other hand, the FPI can be traded from time to time and leads to heavy liquidity. As a result, this causes heavy volatility as traders enter and exit the market swiftly.
Link to the article

2. Vivad se Vishwas scheme is related to which of the following options?
(1) Judicial wrongdoings
(2) Social injustice
(3) Direct Tax disputes
(4) Real estate property disputes
(5) All of the above

Ans: 3
Solution:
Vivad se Vishwas Scheme is a well-thought scheme brought out by the Government of India to put an end to pending direct tax disputes. The benefit under the Scheme is available for all the taxpayers who are in appeal at different levels of appeal matrix. The taxpayer can simply pay the disputed tax and can save the interest levied and leviable, further, any penalties levied and leviable. It is also available for the taxpayers in whose case the Department is in appeal against the revenue adverse orders.
Link to the article

3.Consider the following statements concerning fiscal deficit and choose the incorrect one(s).
A. RBI recommends the fiscal deficit to be kept up to 3% of the GDP.
B. Natrajan Committee recommended that the Government should target to limit the fiscal deficit to 2.5% of the GDP by 2023.
C. Primary deficit = Fiscal deficit – Interest payments
(1) Only A
(2) A and B
(3) B and C
(4) A and C
(5) A, B, and C

Ans: 2
Solution:
The government describes fiscal deficit of India as “the excess of total disbursements from the Consolidated Fund of India, excluding repayment of the debt, over total receipts into the Fund (excluding the debt receipts) during a financial year”. In simple words, it is a shortfall in a government’s income compared with its spending.
Fiscal Deficit = Total expenditure of the government (capital and revenue expenditure) – Total income of the government (Revenue receipts + recovery of loans + other receipts).
The Fiscal Responsibility and Budget Management Act, 2003 provides that the Centre should take appropriate measures to limit the fiscal deficit up to 3% of the GDP by 31st March, 2021. RBI handles only the monetary policy, not fiscal policy.
The NK Singh committee (set up in 2016) recommended that the government should target a fiscal deficit of 3% of the GDP in years up to 31st March, 2020, cut it to 2.8% in 2020-21 and to 2.5% by 2023.
Primary deficit is defined as fiscal deficit minus interest payments on previous borrowings. Primary deficit shows the borrowing requirements of the govt. for meeting expenditure excluding interest payment. Primary deficit = Fiscal deficit – Interest payments
Link to the article

4. EPFO comes under the administrative control of __.
(1) Ministry of Social Justice
(2) Ministry of Finance
(3) RBI
(4) Ministry of Labour & Employment
(5) Ministry of Commerce & Industry, 2007

Ans: 4
Solution:
The EPFO is under the administrative control of the Ministry of Labour and Employment.
Link to the article

5. In October, the number of economic transactions through the unified payments interface (UPI) touched 7.3 trillion, a record high. This is an increase of nearly a three-fourth from last October.
Which of the following statements is incorrect concerning NPCI.
(1) NPCI is initiated by RBI and IBA.
(2) NPCI is formed under the provisions of Payment and Settlement Systems Act, 2007.
(3) It is a not-for-profit company under section 18 of the Companies Act 2013.
(4) The Company is focused on bringing innovations in the retail payment systems through the use of technology.
(5) None of the above

Ans: 3
Solution:
National Payments Corporation of India (NPCI), an umbrella organisation for operating retail payments and settlement systems in India, is an initiative of Reserve Bank of India (RBI) and Indian Banks’ Association (IBA) under the provisions of the Payment and Settlement Systems Act, 2007, for creating a robust Payment & Settlement Infrastructure in India.
Considering the utility nature of the objects of NPCI, it has been incorporated as a “Not for Profit” Company under the provisions of Section 25 of Companies Act 1956 (now Section 8 of Companies Act 2013), with an intention to provide infrastructure to the entire Banking system in India for physical as well as electronic payment and settlement systems. The Company is focused on bringing innovations in the retail payment systems through the use of technology for achieving greater efficiency in operations and widening the reach of payment systems.
Link to the article

Mint StatiCA MCQs – 18th Nov, 2022

1. Consider the following statements and choose the incorrect one(s)!
A. The tax collections come under revenue receipt.
B. The provision to present Union Budget every year is given in the FRBM Act.
C. Salaries to government employees come under revenue expenditure.
(1) A and B
(2) A and C
(3) B and C
(4) A, B and C
(5) None of the above

Ans: 3
Solution:
Revenue Budget– It consists of the Revenue Expenditure and Revenue Receipts.
•Revenue Receipts are receipts which do not have a direct impact on the assets and liabilities of the government. It consists of the money earned by the government through tax (such as excise duty, and income tax) and non-tax sources (such as dividend income, profits, and interest receipts).
•Revenue Expenditure is the expenditure by the government which does not impact its assets or liabilities. For example, this includes salaries, interest payments, pension, and administrative expenses.
Just like how a company manages expenditures and receipts, the Governments also have to manage these two.
•The provision for Union Budget is enshrined in Article 112 of the Indian Constitution, which refers to the Union Budget as the Annual Financial Statement.
•Union Budget is presented on 1st February, or the first working day or February, every year. Earlier it used to be presented on the last working day of February.
•The budget can be divided into Capital Budget and Revenue Budget or Expenditure Budget Receipt Budget.
Link to the article

2. Which of the following statements is correct?
(1) A call option is an obligation to buy a security at a predetermined date and price.
(2) A put option is a right to buy a security at a predetermined date and price.
(3) A put option is an obligation to sell a security at a predetermined date and price.
(4) A call option is a right to buy a security at a predetermined date and price.
(5) All of the above

Ans: 4
Solution:
Call option and Put option are the two main types of options available in the derivatives market. A Call option is used when you expect the prices to increase/rise. A Put option is used when you expect the prices to decrease/fall. 
A call option gives the buyer the right but not the obligation to buy the underlying asset at a particular price (strike price) on or before the expiration date. 
A put option gives the buyer the right but not the obligation to sell the underlying asset at a particular price (strike price) on or before the expiration date. 
Link to the article

3.Consider the following statements concerning mutual funds and choose the incorrect one(s).
A. NAV is New Asset Value.
B. UTI was the first mutual fund in independent India.
C. ETFs can be traded on stock exchanges.
(1) Only A
(2) A and B
(3) B and C
(4) A and C
(5) A, B, and C

Ans: 3
Solution:
A mutual fund is a pool of money managed by a professional Fund Manager. A SEBI-registered trust through an announcement (Called Scheme Information Document) offers to manage a pool of funds in accordance with an investment objective and collects money from the public (investors). The money so collected will be invested in equities, bonds, money market instruments and/or other securities as per the investment objectives. The income/gains generated from these investments are apportioned proportionately amongst the investors after deducting applicable expenses and levies, by calculating a scheme’s “Net Asset Value” or NAV.
Unit Trust of India was the first mutual fund set up in India in the year 1963. In the late 1980s, the Government allowed public sector banks and institutions to set up mutual funds. In the year 1992, the Securities and Exchange Board of India (SEBI) Act was passed. The objectives of SEBI are – to protect the interest of investors in securities and to promote the development of and regulate the securities market.
ETFs are mutual fund units that investors can buy or sell at the stock exchange. This is in contrast to a normal mutual fund unit that an investor buys or sells from the AMC (directly or through a distributor). In the ETF structure, the AMC does not deal directly with investors or distributors. Units are issued to a few designated large participants called Authorised Participants (APs). The APs provide buy and sell quotes for the ETFs on the stock exchange, which enable investors to buy and sell the ETFs at any given point of time when the stock markets are open for trading.
Link to the article

4. KYC was made compulsory for all Indian Financial Institutions by _ in _;
(1) SEBI, 2014
(2) Ministry of Finance, 2005
(3) RBI, 2004
(4) Ministry of Home Affairs, 2006
(5) Ministry of Commerce & Industry, 2007

Ans: 3
Solution:
Since 2004, the Reserve Bank of India made it compulsory for all Indian financial institutions to verify both the identity and address of all customers carrying out financial transactions with them. Thus, the KYC process was introduced by the RBI as the only mode of verification.
KYC means to ‘know your customer’ which is an effective way for an institution to confirm and thereby verify the authenticity of a customer. For this, the customer is required to submit all KYC documentation before investing in various instruments. All financial institutions are mandated by the RBI to do the KYC process for all customers before giving them the right to carry out any financial transactions. Whether the customer uses KYC online verification or opts for offline KYC, this is a simple one-time process.
Link to the article

5. Estimates of inflation released last week may have given the Reserve Bank of India (RBI) some breathing space in its fight to contain inflation. Retail inflation based on the consumer price index (CPI) moderated to 6.77% in October, compared to 7.41% in September. Even inflation based on the wholesale price index (WPI) declined to 8.4% in October, snapping a double-digit inflationary spell that lasted 18 months. Which of the following options is the one which publishes WPI data?
(1) RBI
(2) Labour Bureau
(3) National Statistical Office
(4) Office of Economic Adviser
(5) NITI Aayog

Ans: 4
Solution:
Office of the Economic Adviser (OEA) is an attached office of the Department for Promotion of Industry and Internal Trade, Ministry of Commerce & Industry.
The main functions of the Office of Economic Adviser include, inter alia the following:
Policy Functions
-Economic policy inputs on industrial development.
-Rendering advice relating to formulation of Industrial Policy, Foreign Trade Policy with respect to industrial sector in general with thrust on manufacturing, issues relating to bilateral and multilateral trade, as well as taxes and duties related to industry, including but not restricted to safeguard and anti-dumping duties.
-Analysis of trends of industrial production and growth.
-Examination of multilateral and bilateral issues and processing Policy Notes with economic implications referred to the Office.
Statistical Functions
-Compiling and releasing monthly Wholesale Price Indices
-Compiling and releasing monthly Index of Core Industries Production
-Developing other Indices on experimental basis, e.g. select business service price indices
-Supervising as a ‘source agency’, compilation of monthly production statistics for identified industrial items, their validation, and onward transmission for computation of the monthly Index of Industrial Production (IIP) by Central Statistics Office.
-Monthly Statistical compilation of macro indicators (secondary information).
Link to the article

Mint StatiCA MCQs – 17th Nov, 2022

1. Consider the following statements and choose the incorrect one(s)!
A. The benchmark NIFTY index is managed by S&P.
B. This benchmark index has 50 stocks.
C. Over the Counter trading can be done on NSE.
(1) A and B
(2) A and C
(3) B and C
(4) A, B and C
(5) None of the above

Ans: 2
Solution:
A stock exchange serves as a market where financial instruments like stocks, bonds and commodities are traded. It is a platform where buyers and sellers come together to trade financial tools during specific hours of any business day while adhering to SEBI’s well-defined guidelines. However, only those companies that are listed on a stock exchange are allowed to trade in it.
Stocks which are not listed on a reputed stock exchange can still be traded in an ‘Over The Counter Market’. But such shares would not be held high in esteem in the stock exchange market. So, Over The Counter trading is not done on stock exchanges.
Apart from SENSEX, Nifty is another widely used term in the stock market. It is the index of the National Stock Exchange (NSE), another popular stock exchange in India. It’s a collection of 50 stocks, and is known as Nifty 50. It’s managed by India Index Services and Products Ltd (IISL).
Link to the article

2. Primary deficit includes all fiscal deficits excluding __.
(1) Money borrowed from the central bank
(2) Long-term loans
(3) Short-term loans
(4) Interest payment
(5) All of the above

Ans: 4
Solution:
Deficits:  Revenue deficit is targeted at 3.8% of GDP, and fiscal deficit is targeted at 6.4% of GDP in 2022-23.  The target for primary deficit (which is fiscal deficit excluding interest payments) in 2021-22 is 2.8% of GDP.
Link to the article

3. Consider the following statements and choose the incorrect one(s).
A. M1 and M3 are broad money.
B. M2 and M4 are narrow money.
C. M1 is the most liquid money.
(1) Only A
(2) A and B
(3) B and C
(4) A and C
(5) A, B, and C

Ans: 2
Solution:
Total stock of money in circulation with the public at any point of time is called Money Supply.
Measures of money supply:
M1 = currency in circulation with public [CU] + demand deposits in commercial banks [DD]
M2 = M1 + saving deposits held by post office banks
M3 = M1 + net time deposits held in commercial banks.
M4 = M3 + total deposits with post office banks[except national savings certificate]
M1, M2 are narrow money and M3, M4 are broad money. Liquidity increases from M4 to M1. M3 is the most popular measure of money supply known as aggregate monetary resource.
Link to the article

4. Promotion of Bulk Drug Parks initiative is implemented by;
(1) Department of Economic Affairs
(2) Department for Promotion of Industry and Internal Trade
(3) Department of Pharmaceuticals
(4) Department of Chemicals and Fertilisers
(5) National Health Authority

Ans: 3
Solution:
The Department of Pharmaceuticals has conveyed ‘in-principle’ approval to the proposals of the three States Viz, Himachal Pradesh, Gujarat and Andhra Pradesh under the Scheme for “Promotion of Bulk Drug Parks”, a key initiative to support Bulk Drugs manufacturing in the country.  The Scheme, with a financial outlay of Rs. 3,000 crores notified in 2020, provides financial assistance to three States for establishing Bulk Drug Parks and aims to bring down the cost of manufacturing bulk drugs by the creation of world-class common infrastructure facilities supported by the Central Government and thereby increase the competitiveness of the domestic bulk drug industry.
Link to the article

5. The Securities and Exchange Board of India (Sebi) on Wednesday floated a consultation paper seeking public comments on the Sebi (Buyback of Securities) Regulations 2018. Comments will be sought till 1 December 2022.
(1) 40%
(2) 50%
(3) 60%
(4) 70%
(5) 80%

Ans: 5
Solution:
Buy-back is the process by which a Company buy-back its Shares from the existing Shareholders usually at a price higher than the market price. When the Company buy-back the Shares, the number of Shares outstanding in the market reduces/falls. It is the option available to shareholders to exit from the Company business. It is governed by section 68 of the Companies Act, 2013.
Reasons of Buy-back:-
-To improve Earning per Share;
-To use ideal cash;
-To give confidence to the Shareholders at the time of falling prices;
-To increase promoters’ shareholding to reduce the chances of the takeover;
-To improve return on capital and return on net worth;
-To return surplus cash to the Shareholder.
Link to the article

Mint StatiCA MCQs – 16th Nov, 2022

1. Consider the following statements and choose the incorrect one(s)!
A. GST Bill was passed in Parliament in 2017.
B. GST Council is headed by the RBI Governor.
C. GST has four tax slabs.
(1) A and B
(2) A and C
(3) B and C
(4) A, B and C
(5) None of the above

Ans: 1
Solution:
The GST rate slabs are decided by the GST Council. The GST Council revises the rate slab of goods and services periodically. The GST rates are usually high for luxury supplies and low for essential needs. GST rates in India for various goods and services are divided into four slabs: they are 5% GST, 12% , 18%, & 28%.
Members:
•The members of the Council include the Union Finance Minister (chairperson), the Union Minister of State (Finance) from the Centre.
•Each state can nominate a minister in-charge of finance or taxation or any other minister as a member.
Link to the article

2. The monthly and annual Export Import Databanks are maintained by;
(1) RBI
(2) EXIM Bank
(3) Ministry of Commerce & Industry
(4) Ministry of Corporate Affairs
(5) Ministry of Finance

Ans: 3
Solution:
Link to the article

3. Consider the following statements and choose the incorrect one(s).
A. Indian Institute of Insolvency Professionals (IIIP) is administered by RBI.
B. It is a section 8 company
C. The Chartered Accountants Act was enacted in 1956.
(1) Only A
(2) A and B
(3) B and C
(4) A and C
(5) A, B, and C

Ans: 4
Solution:
The Indian Institute of Insolvency professionals of ICAI ‘IIIP-I’ or IIIPI, a wholly owned subsidiary of the Institute of Chartered Accountants of India ‘ICAI’, is a section 8 Company promoted by ICAI to enrol and regulate insolvency professionals as its members in accordance with the Insolvency and Bankruptcy code 2016 and read with regulations.
The parent body of IIIPI i.e. ICAI is a Statutory body established under the Chartered Accountants Act, 1949 for regulation of the profession of Chartered Accountants in India. ICAI is the second largest accounting body in the world with a strong tradition of service to the public interest and to the Indian economy.
IIIPI has been awarded with the registration certificate as the First Insolvency Professional Agency of India by Hon’ble Union Finance Minister Shri Arun Jaitley on 28th November 2016 at Delhi. IIIPI is the largest IPA with nearly two-third of members in India, has attracted members from a diverse stream including Chartered Accountant, Company Secretary, Cost Accountant, Advocates and Management Professionals.
Link to the article

4. Which of the following options is correct concerning SENSEX?
(1) It is an index of the National Stock Exchange.
(2) It comprises the 70 most actively traded stocks
(3) SENSEX was launched in 1985.
(4) SENSEX uses Free-Float Market Capitalisation method to calculate the Index.
(5) This Free-Float Market Capitalisation method was first used in 2002.

Ans: 4
Solution:
The Bombay Stock Exchange (BSE) Sensex, the barometer of India’s stock market fortunes, was launched on January 2, 1986 with 30 most actively traded stocks of that period, with 1979 as the base year. Today, the 30 stocks on the BSE Sensex, account for around 1/5th of the market capitalization of the Bombay Stock Exchange.
Like all the other major financial indices of the world the Sensex has also shifted to the ‘Free Float market capitalization’ methodology to determine its figures with effect from September 1, 2003. The level of the index is a direct reflection of the performance of the 30 selected key stocks in the market.
Link to the article

5. Prime Minister Narendra Modi and Chinese President Xi Jinping shook hands and spoke briefly at a formal dinner for G20 leaders in Bali on Tuesday—their first face-to-face encounter in public since the start of the military standoff in Ladakh more than two years ago. How much percentage (approx.) of the global GDP the G-20 countries account for?
(1) 40%
(2) 50%
(3) 60%
(4) 70%
(5) 80%

Ans: 5
Solution:
Some two-thirds of the world’s population live in a G20 country. Together, the G20 produces approximately 80 % of global GDP.
Link to the article

Mint StatiCA MCQs – 15th Nov, 2022

1. Consider the following statements and choose the correct one(s)!
A. CPI – combined index is compiled by the Labour Bureau.
B. The base year for CPI-IW is 2016
C. The current base year for CPI-Rural and Urban was changed in 2015
(1) A and B
(2) A and C
(3) B and C
(4) A, B and C
(5) None of the above

Ans: 3
Solution:
The Labour Bureau, an attached office of the Ministry of Labour & Employment, has been compiling Consumer Price Index for Industrial Workers every month on the basis of retail prices collected from 317 markets spread over 88 industrially important centres in the country. The index is compiled for 88 centres and All-India and is released on the last working day of succeeding month. The base year for Consumer Price Index for Industrial Workers is 2016.
The Price Statistics Division (PSD) of the National Statistical Office (NSO), Ministry of Statistics and Programme Implementation (MoSPI) started compiling Consumer Price Index (CPI) separately for rural, urban, and combined sectors on monthly basis with Base Year (2010=100) for all India and States/UTs with effect from January 2011. It revised the Base Year of the CPI from 2010=100 to 2012=100, incorporating many methodological improvements in consonance with the international practices with effect from January, 2015.
Link to the article

2. Viability Gap Funding Scheme was introduced in;
(1) 2001
(2) 2003
(3) 2004
(4) 2005
(5) 2006

Ans: 5
Solution:
The Department of Economic Affairs, Ministry of Finance introduced “the Scheme for Financial Support to PPPs in Infrastructure” (Viability Gap Funding Scheme) in 2006 with a view to support infrastructure projects undertaken through PPP mode that are economically justified but commercially unviable due to large capital investment requirements, long gestation periods and the inability to increase user charges to commercial levels, in this existing Scheme, VGF up to 40% of the Total Project Cost (TPC) is provided by the Government, of India (Gol) and the sponsoring authority in the form of capital grant at the stage of project construction (20%+20%).
Link to the article

3. Consider the following statements and choose the incorrect one(s).
A. In a floating exchange rate system, the central banks are at the centre stage in determining the value of a currency.
B. Currency appreciation of the domestic currency means the increase in a foreign currency’s value with respect to the domestic currency.
C. Devaluation of a currency is done by the administrative action of a country.
(1) Only A
(2) A and B
(3) B and C
(4) A and C
(5) A, B, and C

Ans: 2
Solution:
In a floating exchange rate system, market forces (based on the demand and supply of a currency) determine the value of a currency.
Currency Appreciation: It is an increase in the value of one currency in relation to another currency.
Currency depreciation is a fall in the value of a currency in a floating exchange rate system.
Currency depreciation can occur due to factors such as economic fundamentals, interest rate differentials, political instability or risk aversion among investors.
If the value of the Indian Rupee is weakened through administrative action (by the union government and RBI), it is devaluation.
Link to the article

4. Which of the following options is incorrect concerning the National Hydrogen Mission?
(1) The aim is to develop India as a global hub for manufacturing of hydrogen and fuel cells technology across the value chain.
(2) The mission would put forward specific strategy for the short term (four years), and broad strokes principles for long term (10 years and beyond).
(3) The Government of India will facilitate demand creation in identified segments. Possible areas include suitable mandates for use of green hydrogen in industry such as fertilizer, steel, petrochemicals etc
(4) The mission aims to aid the government in meeting its climate targets and making India a green hydrogen hub.
(5) This will help in meeting the target of production of twenty million tonnes of Green hydrogen by 2030 and the related development of renewable energy capacity.

Ans: 5
Solution:
-The proposed National Hydrogen Energy Mission aims to lay down the vision, intent and direction for harnessing hydrogen energy by the Government of India.
-The aim is to develop India as a global hub for manufacturing of hydrogen and fuel cell technology across the value chain.
-The mission would put forward a specific strategy for the short term (four years), and broad strokes principles for the long term (10 years and beyond).
-It will provide the necessary flexibility to capture benefits from the advances that are taking place in the technological landscape.
-The Government of India will facilitate demand creation in identified segments. Possible areas include suitable mandates for use of green hydrogen in industries such as fertilizer, steel, petrochemicals etc.
-Major activities envisaged under the mission include creating volumes and infrastructure; demonstrations in niche applications including transport and industry; goal-oriented research & development; facilitative policy support; and putting in place a robust framework for standards and regulations for hydrogen technologies.
-The mission aims to aid the government in meeting its climate targets and making India a green hydrogen hub. This will help in meeting the target of production of five million tonnes of Green hydrogen by 2030 and the related development of renewable energy capacity.
Link to the article

5. The RBI aims to keep inflation in the range of __.
(1) 1-4%
(2) 2-5%
(3) 1-5%
(4) 2-6%
(5) 3-7%

Ans: 4
Solution:
In February 2015, the RBI entered an agreement on the monetary policy framework with the central government. As per this agreement, the RBI needs to target an inflation of 4% as measured by the consumer price index (CPI), with a leeway of 2% on either side. This means that inflation needs to be within a band of 2-6%, for the RBI to ensure that it doesn’t break its agreement with the government.
Link to the article

Mint StatiCA MCQs – 14th Nov, 2022

1. Consider the following statements and choose the incorrect one(s)!
A. ASEAN was established in 1972.
B. The relation with ASEAN is in line with India’s “Neighbourhood First Policy”.
C. India became a summit-level partner with ASEAN in 2002.
(1) A, B and C
(2) A and B
(3) A and C
(4) B and C
(5) None of the above

Ans: 2
Solution:
The Association of Southeast Asian Nations, or ASEAN, was established on 8 August 1967 in Bangkok, Thailand, with the signing of the ASEAN Declaration (Bangkok Declaration) by the Founding Fathers of ASEAN: Indonesia, Malaysia, Philippines, Singapore and Thailand. Currently, it has 10 member states.
India’s relationship with ASEAN has emerged as a key cornerstone of our foreign policy. The relationship has evolved from the ‘Look East Policy’ enunciated in early 1990s which led India to become a Sectoral Partner of ASEAN in 1992, a Dialogue Partner in 1996 and a Summit-level Partner in 2002.
Link to the article

2. Preparations are underway ahead of Prime Minister Narendra Modi’s visit to attend the 17th G20 Summit which is to be held from November 14-16, in Bali. When did India join the G-20 Group?
(1) 1999
(2) 2000
(3) 2002
(4) 2003
(5) 2004

Ans: 1
Solution:
The Group of Twenty, or G20, is the premier forum for international cooperation on the most important aspects of the international economic and financial agenda. It brings together the world’s major advanced and emerging economies.
India has been a member of the G20 since its inception in 1999.
Link to the article

3. Consider the following statements concerning G-20 Group and choose the correct one(s).
A. The first Leaders’ Summit was held in Washington.
B. in 2023, the Summit is to be held in the UT of Ladakh.
C. The current Troika is Italy-Indonesia-India.
(1) Only A
(2) A and B
(3) B and C
(4) A and C
(5) A, B, and C

Ans: 4
Solution:
The First G-20 Leaders’ Summit was hosted by the US President in Washington in November 2008 to develop a coordinated response to the global financial crisis.
As a Troika member, India (future President) will work closely with Indonesia (present President) and Italy (previous President) to ensure consistency and continuity of the G20’s agenda.
India will assume the G20 Presidency on December 1, 2022 from Indonesia, and will convene the G20 Leaders’ Summit for the first time in India in 2023, in Jammu and Kashmir.
Link to the article

4. The National Asset Reconstruction Co. Ltd (NARCL) and India Debt Resolution Co. Ltd (IDRCL), the two key constituents of India’s so-called bad bank, have, along with banks, set up a weekly review mechanism to monitor progress of the proposed bad loan sale exercise following direct intervention by Union finance minister Nirmala Sitharaman. When was NARCL established?
(1) 2018
(2) 2019
(3) 2020
(4) 2021
(5) 2022

Ans: 4
Solution:
NARCL has been incorporated, in 2021, under the Companies Act as an Asset Reconstruction Company (ARC). NARCL has been set up by banks to aggregate and consolidate stressed assets for their subsequent resolution. PSBs will maintain51% ownership in NARCL.
Link to the article

5. Central banks around the world continue to contemplate issuing their own digital currencies (CBDCs). Some have already taken steps in this direction. Which was the first country to issue a nationwide CBDC?
(1) Ecuador
(2) China
(3) Bahamas
(4) Brazil
(5) Portugal

Ans: 3
Solution:
After a successful 2019 pilot on the island conglomeration of Exuma which reflects the configuration of the Bahamas; the digital B$ was launched by the Central Bank of the Bahamas in 2020. The sand dollar is the first-ever nationwide Central Bank Digital Currency in the world. As a pilot location, Exuma is a smaller version of the Bahamas.
Link to the article

Mint StatiCA MCQs – 11th Nov, 2022

1. Consider the following statements and choose the correct one(s)!
A. The National Action Plan on climate change (NAPCC) was launched in 2008.
B. The NPCC has 6 component schemes.
C. National Mission for Sustainable Agriculture is one of the components.
(1) A, B and C
(2) A and B
(3) A and C
(4) B and C
(5) None of the above

Ans: 3
Solution:
The National Action Plan on Climate Change (NAPCC) was launched in 2008 by the Prime Minister’s Council on Climate Change.
The National Action Plan on climate change (NAPCC) encompasses eight missions in the specific areas of Solar Energy, Enhanced Energy Efficiency, Sustainable Habitat, Water, Sustaining the Himalayan Eco-system, Strategic Knowledge for Climate Change, Green India, and Sustainable Agriculture.
Link to the article

2. The government proposes to launch the world’s longest luxury river cruise—from Varanasi to Dibrugarh via Bangladesh— next year in what it hopes will give a push to inland waterways development in India. Which is the longest waterway in India?
(1) NW-1
(2) NW-2
(3) NW-3
(4) NW-4
(5) NW-5

Ans: 1
Solution:
TO promote Inland Water Transport (IWT) in the country, 111 waterways (including 5 existing and 106 new) have been declared as National Waterways (NWs) under the National Waterways Act, 2016 which came into effect from 12th April, 2016.
The National waterway 1 (Ganga-Bhagirathi-Hooghly) is the longest waterway in India stretching for 1620 km.
Link to the article

3. Consider the following statements concerning New Pension Scheme and choose the incorrect one(s).
A. NPS Diwas is celebrated on 2nd October every year.
B. The regulatory body for the NPS scheme was established in 2013
C. The NPS scheme was open to all Indian citizens in 2009
(1) Only A
(2) A and B
(3) B and C
(4) A and C
(5) A, B, and C

Ans: 2
Solution:
National Pension System (NPS) is a defined contribution pension scheme. NPS enables an individual to undertake retirement planning while in employment. With systematic savings and investments, NPS facilitates the accumulation of a pension corpus during their working life. NPS is designed to deliver sustainable solution of having adequate retirement income at old age or upon superannuation. While the scheme was initially designed for government employees only, it was opened up for all citizens of India between the age of 18 and 65 in 2009
Pension Fund Regulatory and Development Authority is the regulatory body under the jurisdiction of the Ministry of Finance, Government of India for overall supervision and regulation of pension in India. The Pension Fund Regulatory & Development Authority Act was passed on 19 September 2013 and the same was notified on 1 February 2014. PFRDA is regulating NPS, subscribed by employees of Govt. of India, State Governments and by employees of private institutions/organizations & unorganized sectors. The PFRDA is ensuring the orderly growth and development of the pension market.
PFRDA observes 1st October as the National Pension System Diwas (NPS Diwas).

4. If an IPO is launched by a company as an offer for sale, which of the following receives the proceeds collected through the IPO?
(1) Only the Company
(2) Only the promotors
(3) Both the company and promotors
(4) The lenders to the company
(5) The company, promotors and lenders

Ans: 2
Solution:
Offer-for-sale (OFS) is made by a company’s existing promoters such as venture capital firms or individuals to sell their stakes to the general public. The company will not receive any proceeds from the offer as all of it will go to the promoters.
Link to the article

5. The Securities and Exchange Board of India (Sebi) recently floated a consultation paper seeking public comments on protection of public equity shareholders in case listed entities undergo the corporate insolvency resolution process. What is the minimum debt amount (in crore) for which this process can initiate?
(1) 1
(2) 10
(3) 100
(4) 1000
(5) 50

Ans: 1
Solution:
The Insolvency and Bankruptcy Code, 2016 (‘IBC’) lays down the provisions for conducting insolvency or bankruptcy of individuals, partnership firms, LLP and companies. However, the process of insolvency and liquidation of corporate debtors under the IBC applies where the minimum default amount is Rs.1 crore only.
Link to the article

Mint StatiCA MCQs – 10th Nov, 2022

1. Consider the following statements and choose the correct one(s)!
A. RBI is the regulatory authority for AT-1 Bonds in the primary and secondary markets.
B. AT-1 Bonds are subordinate to equity shares.
C. The Issuers can not skip interest payouts.
(1) A, B and C
(2) A and B
(3) A and C
(4) B and C
(5) None of the above

Ans: 5
Solution:
These bonds, as a primary market offering, are regulated under the directions of the Reserve Bank of India. However, when these bonds are traded on the secondary market, they are governed under SEBI Circular – 2021.
These bonds are subordinate to all other debt and senior only to equity. Banks have a call option that permits them to redeem these bonds after a certain period.
Link to the article

2. The central government announced a record capital expenditure to offer much-needed support to the economy, which struggled to find its feet following the disruptions caused by the covid-19 pandemic. What is the proposed amount for the capital expenditure in the Union Budget 2022-23?
(1) 2.5 lakh crore
(2) 5 lakh crore
(3) 7.5 lakh crore
(4) 8.5 lakh crore
(5) 9.5 lakh crore

Ans: 3
Solution:
Capital expenditure is the amount spent by the government on the development of infrastructure, facilities, equipment, etc.
It also includes the expenditure incurred on acquiring fixed assets such as land and investment by the government, which will give profits or dividends in the future.
The central government’s budget for 2022-23 showed a sharp jump in capital expenditure to a planned Rs 7.5 lakh crore.
Link to the article

3. Consider the following statements concerning disinvestment and choose the correct one(s).
A. Current fiscal year’s disinvestment target is ₹65,000 crore.
B. DPIIT is the nodal department for disinvestment in PSUs.
C. The nodal department for disinvestment comes under the Ministry of Finance.
(1) Only A
(2) A and B
(3) B and C
(4) A and C
(5) A, B, and C

Ans: 4
Solution:
In FY22, the government set a disinvestment target of ₹1.75 trillion but could garner only ₹13,530 crore due to the continued impact of the pandemic. The target was reduced to ₹78,000 crore in the revised estimates of Union Budget 2022, presented in February this year. For FY23, the government set a much lower target of ₹65,000 crore.
The Department of Investment and Public Asset Management (DIPAM) under the Ministry of Finance is the nodal department for the strategic stake sale in the Public Sector Undertakings (PSUs).
Link to the article

4. The implementing agency for setting 100 fodder-centric Farmer Producer Organisations during this fiscal year is headquartered in_.
(1) Gujarat
(2) Telangana
(3) Karnataka
(4) Uttar Pradesh
(5) Haryana

Ans: 1
Solution:
The government has designated the National Dairy Development Board (NDDB) as the implementing agency for setting up of 100 fodder-centric Farmer Producer Organizations (FPOs) during this fiscal to address the fodder deficit in the country.
The National Dairy Development Board (NDDB) was founded in 1965 to replace exploitation with empowerment, tradition with modernity, and stagnation with growth, transforming dairying into an instrument for the development of India’s rural people. NDDB is headquartered in Anand. Gujarat.
Link to the article

5. Ahead of the assembly elections in Gujarat and Himachal Pradesh, the 23rd tranche of electoral bonds opened for sale. Electoral bonds are not issued in which of the following denominations?
(1) Rs 1,000
(2) Rs 5,000
(3) Rs 10, 000
(4) Rs 1,00,000
(5) Rs 10,00,000

Ans: 2
Solution:
Electoral bonds have been pitched as an alternative to cash donations made to political parties as part of efforts to bring transparency to political funding.
State Bank of India is authorised to issue and encash these bonds. The bonds are issued by SBI in denominations of Rs 1,000, Rs 10,000, Rs 1 lakh, Rs 10 lakh and Rs 1 crore.
Link to the article

Mint StatiCA MCQs – 9th Nov, 2022

1. Consider the following statements and choose the incorrect one(s)!
A. FAME-I was launched by the Ministry of Road Transport and Highways
B. Phase II of the FAME Scheme has a budget outlay of 10,000 Crore.
C. FAME Phase II does not cover electric buses.
(1) Only B
(2) A and B
(3) A and C
(4) B and C
(5) A, B and C

Ans: 3
Solution:
FAME was launched in 2015, by the Ministry of Heavy Industries, with the objective to support hybrid/electric vehicles market development and manufacturing ecosystem.
Phase-II of FAME India Scheme for a period of five years commencing from 1st Apri1, 2019 with a total budgetary support of Rs. 10,000 crore.
This phase mainly focuses on supporting electrification of public & shared transportation, and aims to support through demand incentive 7090 eBuses, 5 lakh e-3Wheelers, 55000 e-4 Wheeler Passenger Cars and 10 lakh e-2 Wheelers.  
Link to the article

2. State-run Indian Renewable Energy Development Agency Ltd (IREDA) is eyeing the status of a development financial institution (DFI) to access funds at lower rates to boost funding for India’s ambitious climate goals. Currently, how many DFI are regulated as All-India Financial Institutions by the RBI?
(1) 3
(2) 4
(3) 5
(4) 6
(5) 7

Ans: 3
Solution:
The National Bank for Financing Infrastructure and Development (NaBFID) Act, 2021 received the assent of the President on March 28, 2021 and has come into force w.e.f. April 19, 2021. Accordingly, NaBFID has been set up as a Development Financial Institution (DFI) to support the development of long-term infrastructure financing in India.
NaBFID shall be regulated and supervised as an All-India Financial Institution (AIFI) by the Reserve Bank under Sections 45L and 45N of the Reserve Bank of India Act, 1934. It shall be the fifth AIFI after EXIM Bank, NABARD, NHB and SIDBI.
Link to the article

3.Consider the following statements concerning the Foreign Portfolio Investment (FPI) and choose the incorrect one(s).
A. FPI is a part of a country’s current account.
B. FPI is not a part of the Balance of Payment.
C. FPI is more volatile than FDI.
(1) Only A
(2) A and B
(3) B and C
(4) A and C
(5) A, B, and C

Ans: 2
Solution:
FPI is part of a country’s capital account and is shown on its Balance of Payments (BOP). The BOP measures the amount of money flowing from one country to other countries over one monetary year.
FPI is frequently referred to as “hot money” due to its propensity to leave at the first indication of economic hardship. FPI is riskier than FDI since it is more volatile, liquid, and liquid.
Link to the article

4. The National Stock Exchange held its first conference call with investors on Monday, aiming to win back the trust of potential investors after the country’s largest exchange was rocked by corporate governance issues and scams in the past few years. When did NSE commence its operations?
(1) 1991
(2) 1992
(3) 1993
(4) 1994
(5) 1995

Ans: 4
Solution:
NSE was incorporated in 1992. It was recognised as a stock exchange by SEBI in April 1993 and commenced operations in 1994 with the launch of the wholesale debt market, followed shortly after by the launch of the cash market segment.
Link to the article

5. Vikram-S will be the first space launch vehicle manufactured and operated entirely by a private company in the country— making this the first launch of its kind since the government opened up the space sector for private industry participation in June 2020. How much amount was given to the Department of Space in the Union Budget 2022-23?
(1) 11,200 Crore
(2) 13,700 Crore
(3) 14,800 Crore
(4) 17,300 Crore
(5) 18,500 Crore

Ans: 2
Solution:
The total budgetary allocation for FY 2022-23 towards the Department of Space is INR 13,700 Crore. 
Link to the article

Mint StatiCA MCQs – 8th Nov, 2022

1. Consider the following statements and choose the incorrect one(s)!
A. Coupon rate is the rate of interest paid by bond issuers on the bond’s face value.
B. Coupon rate is the annual payment or the coupon payment divided by the price of the bond.
C. For a 10-year bond with a face value of Rs 2,000 and a price of Rs 2,500, the coupon amount will be Rs 250 per year.
(1) Only B
(2) A and B
(3) A and C
(4) B and C
(5) A, B and C

Ans: 4
Solution:
Coupon: Coupon rate is the rate of interest paid by bond issuers on the bond’s face value. It is the periodic rate of interest paid by bond issuers to their purchasers. The coupon rate is calculated on the bond’s face value (or par value), not on the issue price or market value. For example, if you have a 10-year- Rs 2,000 bond with a coupon rate of 10 per cent, you will get Rs 200 every year for 10 years.
Coupon Rate= Annual payment or the coupon/Face value of the bond.
Link to the article

2. The newly introduced Windfall tax impacts which of the following entities in India?
(1) Crude oil refineries
(2) Commercial banks
(3) IT companies
(4) Fast-moving consumer goods producers
(5) All of the above

Ans: 1
Solution:
Nearly four months after introducing a windfall tax on refiners and local crude oil producers, the government has managed to garner only ₹2,500-3,000 crore a month from the levy, far less than it needs to fully make up for the losses in revenue due to excise cuts, government officials said.
Windfall taxes are designed to tax the profits a company derives from an external, sometimes unprecedented event— for instance, the energy price-rise as a result of the Russia-Ukraine conflict.
These are profits that cannot be attributed to something the firm actively did, like an investment strategy or an expansion of business.
A windfall is defined as an “unearned, unanticipated gain in income through no additional effort or expense”.
Link to the article

3. Consider the following statements concerning the Revamped Distribution Sector Scheme and choose the incorrect one(s).
A. The nodal agency for the Scheme is the Ministry of Consumer Affairs & Food and Public Distribution.
B. The scheme is valid till FY 2025-26.
C. Its financial outlay is Rs.3,03,758 crore till its validity.
(1) Only A
(2) A and B
(3) B and C
(4) A and C
(5) A, B, and C

Ans: 1
Solution:
The Prime Minister, in September 2022, launched the Ministry of Power’s flagship Revamped Distribution Sector Scheme which is aimed at improving the operational efficiencies and financial sustainability of Distribution Companies. With an outlay of Rs.3,03,758 crore over a period of five years from FY 2021-22 to FY 2025-26, the scheme aims to provide financial assistance to DISCOMs for modernization and strengthening of distribution infrastructure, aiming at improvement of the reliability and quality of supply to end consumers. It is also proposed to provide 25 crore Smart Prepaid meters to consumers all over the country.
Link to the article

4. The Supreme Court by a majority view of 3:2 on 7th November, 2022 upheld the validity of the 10 percent reservation to people belonging to the economically weaker sections (EWS) in admissions and government jobs. This reservation was brought in by which of the following constitutional amendments?
(1) 101st Constitutional Amendment
(2) 102nd Constitutional Amendment
(3) 103rd Constitutional Amendment
(4) 104th Constitutional Amendment
(5) 105th Constitutional Amendment

Ans: 3
Solution:
Five-judge Constitution bench of the Supreme Court upholds the validity of the Constitution’s 103rd Amendment Act 2019, which provides for the 10 per cent EWS reservation amongst the general category.
Link to the article

5. In February this year, small saving deposits stood at ₹9.9 trillion, which is only a small fraction of Scheduled Commercial Banks deposits that totalled to ₹170.2 trillion, as per a report by Bank of Baroda. Which Panel had suggested a market-linked interest rate system on small savings scheme?
(1) Usha Thorat Panel
(2) Shyamala Gopinath Panel
(3) Urjit Patel Panel
(4) Debojit Ray Panel
(5) K Subramania Panel

Ans: 2
Solution:
Interest rates on small savings schemes are reset on a quarterly basis, in line with the movement in benchmark government bonds of similar maturity. The rates are reviewed periodically by the Ministry of Finance.
The Shyamala Gopinath panel (2010) constituted on the Small Saving Scheme had suggested a market-linked interest rate system for small savings schemes.
We will cover Small Savings Scheme in detail in PIB Sutra (a new segment by PracticeMock) of this month.
Link to the article

Mint StatiCA MCQs – 7th Nov, 2022

  1. Consider the following statements and choose the incorrect one(s)!
    A. COP-26 was organised in Paris.
    B. UNFCCC entered into force in 1992.
    C. India announced its plans to achieve the Net-Zero Target by 2070 in COP-26.
    (1) Only B
    (2) A and B
    (3) A and C
    (4) B and C
    (5) A, B and C

    Ans: 2
    COP-26 was organised in Glasgow, UK.
    The UNFCCC, signed in 1992 at the United Nations Conference on Environment and Development also known as the Earth Summit, the Rio Summit or the Rio Conference. The UNFCCC entered into force on March 21, 1994.
    India has set itself a target to achieve net zero by 2070.
    Link to the article

  2. Where was the first Conference of Parties to the UNFCCC conducted?
    (1) Paris
    (2) Geneva
    (3) Berlin
    (4) Kyoto
    (5) Buenos Aires

    Ans: 3
    Solution:
    The COP meets every year, unless the Parties decide otherwise. The first COP meeting was held in Berlin, Germany in March, 1995. The COP meets in Bonn, the seat of the secretariat, unless a Party offers to host the session. Just as the COP Presidency rotates among the five recognized UN regions – that is, Africa, Asia, Latin America and the Caribbean, Central and Eastern Europe and Western Europe and Others – there is a tendency for the venue of the COP to also shift among these groups.

  3. Consider the following statements and choose the correct one(s).
    A. The nodal agency for UNFCCC in India is the Ministry of Environment, Forests and Climate Change (MoEFCC).
    B. India ratified the UNFCCC in 1992.
    C. India became a part of the COP in 1993.
    (1) Only C
    (2) A and B
    (3) B and C
    (4) A and C
    (5) A, B, and C

    Ans: 4
    Solution:
    Ratification: India ratified the UNFCCC in 1993, and hence, became a part of the COP in 1993.
    Nodal Agency: The nodal agency for the UNFCCC in India is the Ministry of Environment, Forests and Climate Change (MoEFCC).
    CBDR – RC: India strongly supports the principles of Equity and Common But Differentiated Responsibilities and Respective Capability (CBDR-RC) at the Paris Convention in 2015, COP-21.

  4. The insolvency system in India is regulated by the __.
    (1) RBI
    (2) SEBI
    (3) Ministry of Finance
    (4) IBPS
    (5) IBBI

    Ans: 5
    Solution:
    A key pillar of the insolvency ecosystem is the regulator, namely, the Insolvency and Bankruptcy Board of India (IBBI). It is responsible for professionalising insolvency services through the regulation and development of service providers, namely, insolvency professionals, insolvency professional agencies, insolvency professional entities, information utilities, registered valuers, and registered valuers’ organisations. It has been the endeavour of the IBBI to ensure that the service providers are fit-and-proper persons and technically competent, and also have the motivation and drive to uphold the highest standards of ethics and professionalism. In pursuance of this responsibility, the IBBI has been carrying out and promoting a variety of activities to build the capacity of service providers and monitoring their conduct and performance closely.

  5. What is the minimum capital requirement for entry of small insurers?
    (1) 100 crore
    (2) 200 crore
    (3) 250 crore
    (4) 500 crore
    (5) 740 crore

    Ans: 1
    Solution:
    The Insurance Regulatory and Development Authority of India (IRDAI) has suggested the government ease the minimum capital requirement of ₹100 crore for insurers and permit the regulator to fix the amount, depending on the business plans of the prospective company.
    Doing away with the minimum capital requirement of ₹100 crore would allow entry of small, specialized and niche players, which would help in increased insurance penetration and density in the country.

This brings us to the end of the article. Sign Up at PracticeMock for more of these MCQs.

    Free Mock Tests for the Upcoming Exams

Leave a Reply

Your email address will not be published. Required fields are marked *